SlideShare a Scribd company logo
1 of 237
Download to read offline
Bas 111 Mathematics 3 Dr. Fatma Zedan El Emam
1
CHAPTER 1
Partial Derivatives
1. 1 Functions of Several Variables
Many functions depend on more than one
independent variable. The function h
r
V 2


calculates the volume of a circular cylinder from its
radius and height. In this section, we define
functions of more than one independent variable.
Definition 1
A function f of two variables, x and y, is a rule that
assigns a unique real number f (x, y) to each point
(x, y) in some set D in the xy-plane. The set D is
the domain of f and its range is the set of values
that f takes on, that is, {f (x, y) (x, y)  D}.
We often write z = f (x, y) to make explicit the
value taken on by f at the general point (x, y). The
Bas 111 Mathematics 3 Dr. Fatma Zedan El Emam
2
variables x and y are independent variables and z
is the dependent variable.
A function of two variables is
just a function whose domain
is a subset of 2
R and whose
range is a subset of .
R (see
Fig. 1.1.1), where the domain
D is represented as a subset
Fig. 1.1.1
of the xy -plane and the range is a set of numbers
on a real line, shown as a z-axis.
Definition 2
A function f of three variables, x, y, and z, is a rule
that assigns a unique real number f ( x, y, z ) to
each point (x, y, z) in some set D in three
dimensional space.
Example 1 Let ).
(
ln
1
)
,
( 2
y
x
y
y
x
f 



Find f (e, 0) and sketch the domain of f.
Bas 111 Mathematics 3 Dr. Fatma Zedan El Emam
3
Solution. By substitution
3
2
1
)
(
ln
1
)
0
(
ln
1
0
)
0
,
(
2
2









e
e
e
f
To find the domain of f , we
note that 1

y is defined
only when y ≥ −1,
Fig. 1.1.2
While )
(
ln 2
y
x  is defined only when
y
x 
 2
0 or
2
x
y  . Thus, the domain of f
consists of all points in they-plane for which
2
1 x
y 

 . To sketch the domain, we first sketch
the parabola
2
x
y  and the line y = −1. The
domain of f is then the region lying above or on
the line y = −1 and below the parabola
2
x
y  (See
Fig.1.1.2).
Example 2 Let
1
1
)
,
(




x
y
x
y
x
f Find f (3, 2)
and sketch the domain of f.
Bas 111 Mathematics 3 Dr. Fatma Zedan El Emam
4
Solution. By substitution
2
6
1
3
1
2
3
)
2
,
3
( 




f
The domain of f is Fig. 1.1.3
}
0
1
,
0
1
)
,
{( 




 x
y
x
y
x
D
}
1
,
0
1
)
,
{( 



 x
y
x
y
x . The inequality
,
0
1

 y
x or ,
1


 x
y describes the points
that lie on or above the line ,
1


 x
y while
,
1

x means that the points on the line ,
1

x
must be excluded from the domain. (See Fig.1.1.3)
Example 3 Find the domain and the range of
.
9
)
,
( 2
2
y
x
y
x
g 


Solution. The domain of g is
}
9
)
,
{(
}
0
9
)
,
{(
2
2
2
2







y
x
y
x
y
x
y
x
D
Bas 111 Mathematics 3 Dr. Fatma Zedan El Emam
5
which is the disk with center (0,0) and radius 3.
(See Fig. 1.1.4) The range of g is
}.
)
,
(
,
9
{ 2
2
D
y
x
y
x
z
z 


 Since z is a
positive square root, 0

z . Also, because
,
9
9 2
2


 y
x we have 3
9 2
2


 y
x . So
the range is ]
3
,
0
[
}
3
0
{ 

 z
z
Domain of )
,
( y
x
g
Fig.1.1.4
Graph of )
,
( y
x
g
Example 4 Let
2
2
2
1
)
,
,
( z
y
x
z
y
x
f 


 .
Find )
2
1
,
2
1
,
0
( 
f and the domain of f.
Solution. By substitution
Bas 111 Mathematics 3 Dr. Fatma Zedan El Emam
6
2
1
2
1
2
1
)
0
(
1
)
2
1
,
2
1
,
0
(
2
2
2



















f
The domain of f is
}
0
1
)
,
,
{( 2
2
2




 z
y
x
z
y
x
D
}
1
)
,
,
{( 2
2
2



 z
y
x
z
y
x , we see that the
domain of f consists of all points on or within the
sphere 1
2
2
2


 z
y
x
1.2 Limits
For a function of one variable there are two one-
sided limits at a point 0
x , namely, )
(
lim
0
x
f
x
x


and
)
(
lim
0
x
f
x
x


reflecting the fact that there are only
two directions from which x can approach 0
x , the
right or the left. If 


)
(
lim
0
x
f
x
x
)
(
lim
0
x
f
x
x


, then
Bas 111 Mathematics 3 Dr. Fatma Zedan El Emam
7
)
(
lim
0
x
f
x
x
does not exist. For functions of two
variables the
situation is more complicated
because there are infinitely
many different curves (paths)
along which (x, y) can
Fig. 1.2.1
approach )
,
( 0
0 y
x (See Fig. 1.2.1).
Theorem 1
(a) If L
y
x
f 
)
,
( as )
,
(
)
,
( 0
0 y
x
y
x  , then
L
y
x
f 
)
,
( as )
,
(
)
,
( 0
0 y
x
y
x  along any path
(b) If 1
)
,
( L
y
x
f  as )
,
(
)
,
( 0
0 y
x
y
x  along a
path 1
C and 2
)
,
( L
y
x
f  as )
,
(
)
,
( 0
0 y
x
y
x 
along a path 2
C , where 2
1 L
L  , then
)
,
(
lim
)
,
(
)
,
( 0
0
y
x
f
y
x
y
x 
does not exist.
Bas 111 Mathematics 3 Dr. Fatma Zedan El Emam
8
• To show a limit does not exist, it is still enough to
find two paths along which the limits are not
equal. We give some suggestions here. You can try
the following paths:
1) Horizontal line through )
0
,
0
( , the equation of
such a path is 0

y .
2) Vertical line through )
0
,
0
( , the equation of such
a path is 0

x .
3) Any straight line through )
0
,
0
( , the equation of
the line with slope m through )
0
,
0
( is .
mx
y 
4) Quadratic paths. For example, a typical
quadratic path through )
0
,
0
( is
2
x
y  or
2
y
x 
Definition 3
We say that a function ƒ(x, y) approaches the limit
L as (x, y) approaches ),
,
( 0
0 y
x and write
)
,
(
lim
)
,
(
)
,
( 0
0
y
x
f
y
x
y
x 
= L
if, for every number ,
0

 there exists a
Bas 111 Mathematics 3 Dr. Fatma Zedan El Emam
9
corresponding number 0

 such that for all (x, y)
in the domain of ƒ, 

 L
y
x
f )
,
( whenever





 2
0
2
0 )
(
)
(
0 y
y
x
x .
Remark: We remark that the usual limit theorems
hold for limits of functions of two variables. For
example
)
,
(
)
,
( 0
0
lim
y
x
y
x 
[f (x, y)  g (x, y)]
=
)
,
(
)
,
( 0
0
lim
y
x
y
x 
f (x, y) 
)
,
(
)
,
( 0
0
lim
y
x
y
x 
g (x, y)
Example 5 Calculating Limits
a) 3
2
)
1
,
0
(
)
,
( 5
3
lim
y
xy
y
x
xy
x
y
x 




b) 2
2
)
4
,
3
(
)
,
(
lim y
x
y
x



Solution. Direct substitution gives:
a)
3
)
1
(
)
1
)(
0
(
5
)
1
(
)
0
(
3
)
1
)(
0
(
0
5
3
lim 3
2
3
2
)
1
,
0
(
)
,
(











 y
xy
y
x
xy
x
y
x
b) 5
25
)
4
(
)
3
(
lim 2
2
2
2
)
4
,
3
(
)
,
(








y
x
y
x
Bas 111 Mathematics 3 Dr. Fatma Zedan El Emam
10
Example 6
Find .
lim
2
)
0
,
0
(
)
,
( y
x
xy
x
y
x 


Solution. Since the denominator y
x 
approaches 0 as )
0
,
0
(
)
,
( 
y
x , we multiply
numerator and denominator by ,
y
x 
however, we produce an equivalent fraction
whose limit we can find:
)
)(
(
)
)(
(
lim
lim
2
)
0
,
0
(
)
,
(
2
)
0
,
0
(
)
,
( y
x
y
x
y
x
xy
x
y
x
xy
x
y
x
y
x 








y
x
y
x
y
x
x
y
x 




)
)(
(
lim
)
0
,
0
(
)
,
(
)
(
lim
)
0
,
0
(
)
,
(
y
x
x
y
x



0
)
0
0
(
0 


Example 7
Show that 2
2
2
2
)
0
,
0
(
)
,
(
lim
y
x
y
x
y
x 


does not exist.
Solution. Direct substitution gives:
Bas 111 Mathematics 3 Dr. Fatma Zedan El Emam
11
0
0
lim 2
2
2
2
)
0
,
0
(
)
,
(



 y
x
y
x
y
x
(indeterminate). More
investigation is needed!
Let 2
2
2
2
)
,
(
y
x
y
x
y
x
f


 . First let’s approach (0, 0)
along the x-axis. Then 0

y gives 1
)
0
,
( 2
2


x
x
x
f
for all ,
0

x so, 1
)
,
( 
y
x
f as )
0
,
0
(
)
,
( 
y
x
along the x-axis.
Next let’s approach (0, 0) along the y-axis by
putting .
0

x Then 1
)
,
0
( 2
2




y
y
y
f for all
,
0

y so 1
)
,
( 

y
x
f as )
0
,
0
(
)
,
( 
y
x along the
y-axis.
(See Fig. 1.2.2) Since f has
two different limits along 2
different paths, the given
limit does not exist.
Fig. 1.2.2
Bas 111 Mathematics 3 Dr. Fatma Zedan El Emam
12
Example 8
Show that 2
2
)
0
,
0
(
)
,
(
lim
y
x
xy
y
x 

does not exist.
Solution. Let 2
2
)
,
(
y
x
xy
y
x
f

 . First let’s approach
(0, 0) along the x-axis.
If ,
0

y then ,
0
0
)
0
,
( 2


x
x
f so
0
)
,
( 
y
x
f as )
0
,
0
(
)
,
( 
y
x along the x-axis.
Next let’s approach (0, 0) along the y-axis.
If ,
0

x then ,
0
0
)
,
0
( 2


y
y
f so
0
)
,
( 
y
x
f as )
0
,
0
(
)
,
( 
y
x along the y-axis.
Although we have obtained identical limits along
the axes, that does not show that the given limit is
0. Now let’s approach )
0
,
0
( along another line, say
y=x. For all ,
0

x .
2
1
)
,
( 2
2
2



x
x
x
y
x
f So,
2
1
)
,
( 
y
x
f as )
0
,
0
(
)
,
( 
y
x along y=x.
Bas 111 Mathematics 3 Dr. Fatma Zedan El Emam
13
Since f has two different limits along 2 different
paths, the given limit does not exist.
Example 9
If ,
)
,
( 4
2
2
y
x
xy
y
x
f

 does )
,
(
lim
)
0
,
0
(
)
,
(
y
x
f
y
x 
exist?
Solution. Let’s approach (0, 0) along the line y=mx
where m is the slope, and
.
0
1
)
(
)
(
)
,
(
)
,
(
2
4
2
4
4
2
3
2
4
2
2









x
m
x
m
x
m
x
x
m
mx
x
mx
x
mx
x
f
y
x
f
So 0
)
,
( 
y
x
f as )
0
,
0
(
)
,
( 
y
x along y=mx.
Thus, the limit as (x, y) approaches to the origin
along any straight line is zero. But that does not
show that the given limit is 0, if we now let
)
0
,
0
(
)
,
( 
y
x along the parabola
2
y
x  , we have
  2
1
2
)
,
(
)
,
( 4
4
4
2
2
2
2
2






y
y
y
y
y
y
y
y
f
y
x
f .
So
2
1
)
,
( 
y
x
f as )
0
,
0
(
)
,
( 
y
x along
2
y
x 
Bas 111 Mathematics 3 Dr. Fatma Zedan El Emam
14
Since different paths lead to different limiting
values, the given limit does not exist.
Example 10 Find 2
2
2
)
0
,
0
(
)
,
(
3
lim
y
x
y
x
y
x 

if it exists
Solution. We could show that the limit along any
line through the origin is 0. This doesn’t prove that
the given limit is 0, but the limits along the
parabolas
2
x
y  and
2
y
x  also turn out to be 0,
so we begin to suspect that the limit does exist
and is equal to 0.
Bas 111 Mathematics 3 Dr. Fatma Zedan El Emam
15
Hence, by Definition 3,
0
3
lim 2
2
2
)
0
,
0
(
)
,
(


 y
x
y
x
y
x
1. 3 Continuity
Definition 4
A function )
,
( y
x
f is continuous at the point
)
,
( 0
0 y
x if
1. f is defined at )
,
( 0
0 y
x ,
2. )
,
(
lim
)
,
(
)
,
( 0
0
y
x
f
y
x
y
x 
exists,
3. ).
,
(
)
,
(
lim 0
0
)
,
(
)
,
( 0
0
y
x
f
y
x
f
y
x
y
x


A function is continuous if it is continuous at every
point of its domain.
Example 11 Every polynomial in x, y is continuous
on
2
R . Each rational function is continuous in its
Bas 111 Mathematics 3 Dr. Fatma Zedan El Emam
16
domain. For instance, the rational function
y
x
y
x
x
y
x
f



3
2
)
,
( is continuous on
}.
0
)
,
{( 2



 y
x
R
y
x
D
Example 12
let .
3
)
,
( 2
2
2
y
x
y
x
y
x
f

 Where is f continuous?
Example 13 let









).
0
,
0
(
)
(
,
0
)
0
,
0
(
)
(
,
3
)
,
( 2
2
2
x,y
x,y
y
x
y
x
y
x
f
Show that f is continuous on
2
R .
Example 14 Show that









)
0
,
0
(
)
(
,
0
)
0
,
0
(
)
(
,
2
)
,
( 2
2
x,y
x,y
y
x
xy
y
x
f
is continuous at every point except the origin.
Solution. The function f is continuous at any
point )
0
,
0
(
)
( 
x,y because its values are then
Bas 111 Mathematics 3 Dr. Fatma Zedan El Emam
17
given by a rational function of x and y. At (0, 0),
the value of f is defined, but f has no limit as
)
0
,
0
(
)
,
( 
y
x , as we will now see. For every
value of m, the function f has a constant value on
the line y=mx, ,
0

x because
.
1
2
2
)
(
)
(
2
2
)
,
(
2
2
2
2
2
2
2
2
2
m
m
x
m
x
mx
mx
x
mx
x
y
x
xy
y
x
f
mx
y
mx
y











Therefore, f has this number as its limit as (x, y)
approaches (0, 0) along the line:
  .
1
2
)
,
(
lim
)
,
(
lim 2
)
0
,
0
(
)
,
(
along
)
0
,
0
(
)
,
( m
m
y
x
f
y
x
f mx
y
y
x
mx
y
y
x 

 



This limit changes with m. There is therefore no
single number we may call the limit of f as (x, y)
approaches the origin. Then, the limit does not
exist and the function is not continuous.
Bas 111 Mathematics 3 Dr. Fatma Zedan El Emam
18
Example 15 Where is the function
2
2
2
2
)
,
(
y
x
y
x
y
x
f


 continuous?
Solution. The function is discontinuous at (0, 0)
because it is not defined at (0, 0). Since f (x, y) is a
rational function, it is continuous on its domain,
which is the set )}.
0
,
0
(
)
,
(
)
,
{( 
 y
x
y
x
D
Example 16 Let










).
0
,
0
(
)
(
,
0
)
0
,
0
(
)
(
,
)
,
( 2
2
2
2
x,y
x,y
y
x
y
x
y
x
f
Here, f is defined at (0,0) but is still discontinuous
there because 2
2
2
2
)
0
,
0
(
)
,
(
lim
y
x
y
x
y
x 


does not exist (see
Example 7).
Remark: One may compute limits using polar
coordinates. This is especially convenient for limits
at the origin and for those expressions that are
independent of θ.
Bas 111 Mathematics 3 Dr. Fatma Zedan El Emam
19
Example 17 Find ).
ln(
)
(
lim 2
2
2
2
)
0
,
0
(
)
,
(
y
x
y
x
y
x



Solution. Let )
,
( 
r be polar coordinates of the
point )
,
( y
x with 0

r . Then we have
2
2
2
,
sin
,
cos y
x
r
r
y
r
x 


 
 .
Moreover, since 0

r we have ,
2
2
y
x
r 
 so
that 
 0
r if and only if )
0
,
0
(
)
,
( 
y
x . Thus,
0
)
(
lim
rule)
s
Hopital’
L’
(
/
2
/
2
lim
/
1
ln
2
lim
ln
lim
)
ln(
)
(
lim
2
0
3
0
2
0
2
2
0
2
2
2
2
)
0
,
0
(
)
,
(


















r
r
r
r
r
r
r
y
x
y
x
r
r
r
r
y
x
Remark: The graph of
)
ln(
)
(
)
,
( 2
2
2
2
y
x
y
x
y
x
f 


in Example 17 is a surface
with a hole at the origin
(Figure 1.3.1). We can
remove this discontinuity by
defining f (0, 0) to be 0. Figure 1.3.1
Bas 111 Mathematics 3 Dr. Fatma Zedan El Emam
20
Exercise 1
1-5 Find and sketch the domain of the function.
6-11 Use limit laws to evaluate the
12-13 Show that the limit does not exist by
considering the limits as (x, y)→(0, 0) along the
coordinate axes.
14-17 Determine whether the limit exists. If so,
find its value.
Bas 111 Mathematics 3 Dr. Fatma Zedan El Emam
21
18-20 Evaluate the limit using the substitution
2
2
y
x
z 
 and observing that 
 0
z if and only if (x,
y)→(0, 0).
21- 22 Evaluate the limits by converting to polar
coordinates
24-27 Determine the set of points at which the
function is continuous.
Bas 111 Mathematics 3 Dr. Fatma Zedan El Emam
22
1. 4 Partial Derivatives
Definition 5
If z = f (x, y), then the first partial derivative of f
with respect to x (w.r.t. x) is denoted by
x
f


or x
f
and is given by
h
y
x
f
y
h
x
f
x
f
h
)
,
(
)
,
(
lim
0






Similarly, the partial derivative of f w.r.t. y is
denoted by
y
f


or y
f and is given by
k
y
x
f
k
y
x
f
y
f
k
)
,
(
)
,
(
lim
0






The slope of the curve )
,
( 0
y
x
f at the point
))
,
(
,
,
( 0
0
0
0 y
x
f
y
x
P in the plane 0
y
y  is the value
of the partial derivative of ƒ w.r.t. x at )
,
( 0
0 y
x .
Bas 111 Mathematics 3 Dr. Fatma Zedan El Emam
23
Rule for Finding Partial Derivatives of z = f (x, y)
1. To find x
f , regard y as a constant and
differentiate w.r.t. x.
2. To find y
f , regard x as a constant and
differentiate w.r.t. y.
Example 1
Find the values of
x
f


and
y
f


at the point (4, -5) if
.
1
3
)
,
( 2



 y
xy
x
y
x
f
Bas 111 Mathematics 3 Dr. Fatma Zedan El Emam
24
Solution. To find
x
f


, we regard y as a constant
and differentiate w.r.t. x:
.
3
2
0
0
1
3
2
)
1
3
( 2
y
x
y
x
y
xy
x
x
x
f
















the value of
x
f


at (4, -5) is 2(4) + 3(-5) = -7.
To find
y
f


, we regard x as a constant and
differentiate w.r.t. y:
.
1
3
0
1
1
3
0
)
1
3
( 2
















x
x
y
xy
x
y
y
f
the value of
y
f


at (4, -5) is 3(4) + 1 = 13.
Example 2 Find
y
f


if .
sin
)
,
( xy
y
y
x
f 
Solution. We treat x as a constant and ƒ as a
product of y and sin xy:
Bas 111 Mathematics 3 Dr. Fatma Zedan El Emam
25
.
sin
cos
)
(sin
)
(
)
cos
(
)
(
)
(sin
sin
)
sin
(
xy
xy
xy
xy
xy
y
xy
y
y
y
xy
xy
y
y
xy
y
y
y
f

















Example 3 Find x
f and y
f if .
cos
2
)
,
(
x
y
y
y
x
f


Solution. We treat x as a quotient. With y held
constant, we get
.
)
cos
(
sin
2
)
cos
(
)
sin
(
2
)
0
(
)
cos
(
)
cos
(
)
cos
(
2
)
2
(
)
cos
(
cos
2
2
2
2
x
y
x
y
x
y
x
y
x
y
x
y
x
y
x
y
y
x
x
y
x
y
y
x
fx


























With x held constant, we get
2
)
cos
(
)
cos
(
2
)
2
(
)
cos
(
cos
2
x
y
x
y
y
y
y
y
x
y
x
y
y
y
fy



















Bas 111 Mathematics 3 Dr. Fatma Zedan El Emam
26
.
)
cos
(
cos
2
)
cos
(
)
1
(
2
)
2
(
)
cos
(
2
2
x
y
x
x
y
y
x
y






Example 4
If ,
1
sin
)
,
( 







y
x
y
x
f calculate
x
f


and
y
f


.
Solution. Using the Chain Rule for functions of one
variable, we have
y
y
x
y
x
x
y
x
x
f






























1
1
1
cos
1
1
cos
2
)
1
(
1
cos
1
1
cos
y
x
y
x
y
x
y
y
x
y
f































• Implicit differentiation works for partial
derivatives the way it works for ordinary
derivatives, as the next example illustrates
Example 5 Find x
z 
 / if z is defined implicitly as a
function of x and y by the equation
y
x
z
yz 

 ln
Bas 111 Mathematics 3 Dr. Fatma Zedan El Emam
27
Solution. We differentiate both sides of the w.r.t.
x, holding y constant and treating z as a
differentiable function of x:
.
1
1
1
0
1
1
ln
)
(



































yz
z
x
z
x
z
z
y
x
z
z
x
z
y
x
y
x
x
z
x
yz
x
Functions Of More Than Two Variables
The definitions of the partial derivatives of
functions of more than two independent variables
are ordinary derivatives with respect to one
variable, taken while the other independent
variables are held constant.
Example 6
Calculate
z
f


, if x, y, and z are independent
variables and ).
3
(
sin
)
,
,
( z
y
x
z
y
x
f 

Bas 111 Mathematics 3 Dr. Fatma Zedan El Emam
28
Solution.
).
3
(
cos
3
)
3
(
)
3
(
cos
)
3
(
sin
)]
3
(
sin
[
z
y
x
z
y
z
z
y
x
z
y
z
x
z
y
x
z
z
f

















Example 7
If ,
sin
3
2 xz
e
z
y
x
w 
 find ,
/
,
/ y
w
x
w 


 and
.
/ z
w 

Solution. xz
ze
z
xy
x
w




sin
2 3
z
y
x
y
w
sin
3 2
2



xz
xe
z
y
x
x
w




cos
3
2
Second-Order Partial Derivatives
When we differentiate a function ƒ(x, y) twice, we
produce its second-order derivatives. These
derivatives are usually denoted by
Bas 111 Mathematics 3 Dr. Fatma Zedan El Emam
29
2
2
)
(
x
f
x
f
x
f
f
f
x
xx
x
x
x


















x
y
f
x
f
y
f
f
f
y
xy
y
x
x


















 2
)
(
y
x
f
y
f
x
f
f
f
x
yx
x
y
y


















 2
)
(
2
2
)
(
y
f
y
f
y
f
f
f
y
yy
y
y
y


















Theorem 2 Let f be a function of x and y If
xy
y
x f
f
f
f ,
,
, and yx
f are continuous on an open
region R, then
yx
xy f
f 
throughout R.
If w = f (x, y, z) and f has continuous second partial
derivatives, then the following equalities hold for
mixed partials:
.
;
;
2
2
2
2
2
2
z
y
w
y
z
w
z
x
w
x
z
w
y
x
w
x
y
w





















Bas 111 Mathematics 3 Dr. Fatma Zedan El Emam
30
Example 8 Find the Second Partial Derivatives of
f if .
3
2
)
,
( 2
2
3
x
y
x
y
x
y
x
f 


Solution. Since the first Partial of f are
3
4
3
)
,
( 2
2


 xy
y
x
y
x
fx
and ,
2
2
)
,
( 2
3
x
y
x
y
x
fy 
 then
y
xy
xy
y
x
x
y
x
fxx 4
6
)
3
4
3
(
)
,
( 2
2
2







x
y
x
xy
y
x
y
y
x
fxy 4
6
)
3
4
3
(
)
,
( 2
2
2







x
y
x
x
y
x
x
y
x
fyx 4
6
)
2
2
(
)
,
( 2
2
3






.
2
)
2
2
(
)
,
( 3
2
3
x
x
y
x
y
y
x
fyy 




Example 9 Find the Second Partial Derivatives of
f if .
)
,
( 4
3
2
y
x
y
x
y
x
f 

Solution. Since the first Partial of f are
y
x
xy
y
x
fx
3
3
4
2
)
,
( 

and ,
3
)
,
( 4
2
2
x
y
x
y
x
fy 
 then
Bas 111 Mathematics 3 Dr. Fatma Zedan El Emam
31
y
x
y
y
x
xy
x
y
x
fxx
2
3
3
3
12
2
)
4
2
(
)
,
( 





3
2
3
3
4
6
)
4
2
(
)
,
( x
xy
y
x
xy
y
y
x
fxy 





3
2
4
2
2
4
6
)
3
(
)
,
( x
xy
x
y
x
x
y
x
fyx 





.
6
)
3
(
)
,
( 2
4
2
2
y
x
x
y
x
y
y
x
fyy 




Third and higher partial derivatives are defined in
like manner. For example,
xxx
f
x
f
x
x
f
















2
2
3
3
,
yyyy
f
y
f
y
y
f
















3
3
4
4
xyy
f
x
y
f
y
x
y
f

















 2
2
3
,
xxyy
f
x
y
f
y
x
y
f


















2
3
2
2
4
.
Bas 111 Mathematics 3 Dr. Fatma Zedan El Emam
32
and so on. If first, second, and third partial
derivatives are continuous, then
xxy
yxx
xyx f
f
f 
 and .
yyx
xyy
yxy f
f
f 

Example 9 Let .
)
,
( 2
y
e
y
y
x
f x

 Find .
xyy
f
Solution.  
x
xyy e
y
y
x
f
y
x
y
f
f 2
2
2
2
2
2
3


















  x
x
e
ye
y
2
2 



1. 5 The Chain Rule
Recall that the Chain Rule for functions of a single
variable gives the rule for differentiating a
composite function: If )
(x
f
y  and )
(t
g
x  ,
where f and g are differentiable functions, then y
is indirectly a differentiable function of t and
dt
dx
dx
dy
dt
dy

Bas 111 Mathematics 3 Dr. Fatma Zedan El Emam
33
We will now derive a version of the chain rule for
functions of two variables.
The Chain Rule for Derivative
Suppose that )
,
( y
x
f
z  is
a differentiable function of x
and y, where )
(t
g
x  and
)
(t
h
y  are both
differentiable functions of t.
Then z is a differentiable Fig. 1.5.1
function of t and
dt
dy
y
z
dt
dx
x
z
dt
dz





 .
See “tree diagram” Fig. 1.5.1
Example 11 Suppose that
3
2
2
,
, t
y
t
x
y
x
z 


Bas 111 Mathematics 3 Dr. Fatma Zedan El Emam
34
Use the chain rule to find dt
dz/ , and check the
result by expressing z as a function of t and
differentiating directly.
Solution. By the chain rule
6
2
4
5
2
2
7
)
3
)(
(
)
2
)(
2
(
)
3
)(
(
)
2
)(
2
(
t
t
t
t
t
t
x
t
xy
dt
dy
y
z
dt
dx
x
z
dt
dz











Alternatively, we can express z directly as a
function of t ,
7
3
2
2
2
)
(
)
( t
t
t
y
x
z 


and then differentiate to obtain 6
7
/ t
dt
dz  .
Example 12 If ,
3 4
2
xy
y
x
z 
 where t
x 2
sin

and ,
cost
y  find
dt
dz
when .
0

t
Solution. The Chain Rule gives
)
sin
)(
12
(
)
2
cos
2
)(
3
2
( 3
2
4
t
xy
x
t
y
xy
dt
dy
y
z
dt
dx
x
z
dt
dz











Bas 111 Mathematics 3 Dr. Fatma Zedan El Emam
35
when t=0, we have x = sin 0 = 0 and y = cos 0 = 1.
Therefore
6
)
0
sin
)(
0
0
(
)
0
cos
2
)(
3
0
(
0







t
dt
dz
The Chain Rules for Partial Derivatives
Suppose that )
,
( y
x
f
z  is a differentiable
function of x and y, where )
,
( v
u
g
x  and
)
,
( v
u
h
y  are both differentiable functions of
u and v. Then
u
y
y
z
u
x
x
z
u
z












v
y
y
z
v
x
x
z
v
z












Example 13 Given that
v
u
y
v
u
x
e
z xy
/
,
2
, 



Bas 111 Mathematics 3 Dr. Fatma Zedan El Emam
36
find ∂z/∂u and ∂z/∂v using the chain rule.
Solution.
,
1
4
2
2
2
1
)
(
)
2
)(
(
)
/
)(
2
(
)
/
)(
2
(
v
u
v
u
v
u
v
u
xy
xy
xy
e
v
u
e
v
v
u
v
u
e
v
x
y
v
xe
ye
u
y
y
z
u
x
x
z
u
z















 






























)
/
)(
2
(
2
2
)
/
)(
2
(
2
2
2
2
)
2
(
)
(
)
1
)(
(
v
u
v
u
v
u
v
u
xy
xy
xy
e
v
u
e
v
u
v
u
v
u
e
v
u
x
y
v
u
xe
ye
v
y
y
z
v
x
x
z
v
z






















































Example 14 If y
e
z x
sin
 , where
2
st
x  and
t
s
y 2
 , find s
z 
 / and t
z 
 / using the chain
rule
Solution.
Bas 111 Mathematics 3 Dr. Fatma Zedan El Emam
37
),
cos(
2
)
sin(
)
2
)(
cos
(
)
)(
sin
(
2
2
2
2
2
2
t
s
ste
t
s
e
t
st
y
e
t
y
e
s
y
y
z
s
x
x
z
s
z
st
st
x
x
















)
cos(
)
sin(
2
)
)(
cos
(
)
2
)(
sin
(
2
2
2
2
2
2
t
s
e
s
t
s
ste
s
y
e
st
y
e
t
y
y
z
t
x
x
z
t
z
st
st
x
x
















1.6 Implicit Differentiation
Partial derivatives can be used to find derivatives
of functions that are defined implicitly. Suppose,
an equation F(x, y) = 0 defines a differentiable
function f of one variable x such that F(x, f (x)) = 0
for all x in the domain D of f. If we let
w = F(u, y), where u = x and y = f (x)
then applying Chain Rule and using the fact that w
=F(x, f(x)) =0 for all x in D,
,
0







dx
dy
y
w
dx
du
u
w
dx
dw
Bas 111 Mathematics 3 Dr. Fatma Zedan El Emam
38
that is, .
0
)
(
)
1
( 






x
f
y
w
u
w
If ,
0
/ 

 y
w then (since u = x)
.
)
,
(
)
,
(
/
/
)
(
y
x
F
y
x
F
y
w
x
w
x
f
y
x









We may summarize this discussion as follows.
Theorem 3 If the equation F(x, y) = 0 defines
implicitly as a differentiable function x and if
0
)
,
( 
y
x
Fy one variable, then
.
)
,
(
)
,
(
y
x
F
y
x
F
dx
dy
y
x


Example 15 Find
dx
dy
if .
0
3
2
3


 x
y
x
Solution. Take .
3
)
,
( 2
3


 x
y
x
y
x
F Then
.
2
3 2
2
yx
y
x
F
F
dx
dy
y
x 




Alternatively, differentiating implicitly yields
Bas 111 Mathematics 3 Dr. Fatma Zedan El Emam
39
0
0
2
3 2
2










dx
dy
y
x
y
x or
yx
y
x
dx
dy
2
3 2
2



which agrees with the result obtained first.
Example 16 Find
dx
dy
if .
0
2
sin
2


 y
y
x
Solution. Take .
2
sin
)
,
( 2
y
y
x
y
x
F 

 Then
.
2
cos
2





y
x
F
F
dx
dy
y
x
Theorem 4 If the equation F(x, y, z) = 0 defines
implicitly as a differentiable function of x and y,
and if ,
0
)
,
( 
y
x
Fz then
z
x
F
F
x
z




and .
z
y
F
F
y
z




Example 17 Consider the sphere .
1
2
2
2


 z
y
x
Find ∂z/∂x and ∂z/∂y at the point .
3
2
,
3
1
,
3
2






Solution. By Theorem 4 with
Bas 111 Mathematics 3 Dr. Fatma Zedan El Emam
40
,
1
)
,
,
( 2
2
2



 z
y
x
z
y
x
F
z
x
z
x
F
F
x
z
z
x








2
2
and
z
y
z
y
F
F
y
z
z
y








2
2
At the point ,
3
2
,
3
1
,
3
2





 evaluating these
derivatives gives 1
/ 


 x
z and
2
1
/ 


 y
z .
Bas 111 Mathematics 3 Dr. Fatma Zedan El Emam
41
Exercise 2
22-26 Use the chain rule to find dz/dt.
Bas 111 Mathematics 3 Dr. Fatma Zedan El Emam
42
Use appropriate forms of the chain rule to find ∂z/∂u and
∂z/∂v.
30. Find the value of x
z 
 / at the point (1,1,1) if z is
defined implicitly as a function of x and y by the
equation
31. Find x
z 
 / and y
z 
 / if z is defined implicitly as
a function of x and y by the equation
32. Find dx
dy / if .
0
1
5
4
3 3
4




 x
x
y
y
33-35 Find dy/dx
36-37 Find ∂z/∂x and ∂z/∂y
Bas 111 Mathematics 3 Dr. Fatma Zedan El Emam
43
1.7 Directional Derivatives and Gradient Vectors
1.7.1 The Directional Derivative
Suppose that the function ƒ(x, y) is defined
throughout a region R in the xy-plane, that
)
,
( 0
0
0 y
x
P is a point in R, and that j
u
i
u 2
1
u 
 is a
unit vector. Then the equations
2
0
1
0 , hu
y
y
hu
x
x 



parameterize the line through 0
P parallel to u. If
the parameter h measures arc length from 0
P in
the direction of u, we find the rate of change of ƒ
at 0
P in the direction of u by calculating
)
,
( 0
0 y
x
f
Du from the following definition (Fig. 1).
Fig. 1
Bas 111 Mathematics 3 Dr. Fatma Zedan El Emam
44
Definition 1
The directional derivative of f at )
,
( 0
0 y
x in the
direction of a unit vector j
u
i
u 2
1
u 
 is
h
y
x
f
hu
y
hu
x
f
y
x
f
D
h
u
)
,
(
)
,
(
)
,
( 0
0
2
0
1
0
0
0
0 lim





(1)
if this limit exists.
Note If i

u ( 1
1 
u and 0
2 
u ), then Equation (1)
gives
)
,
(
)
,
(
)
,
(
)
,
(
0
0
0
0
0
0
0
0
0 lim
y
x
f
h
y
x
f
y
h
x
f
y
x
f
D
x
h
i





That is, the directional derivative of f in the x-
direction is the partial derivative of f in the x-
direction, as expected. Similarly, we can show that
).
,
(
)
,
( 0
0
0
0 y
x
f
y
x
f
D y
j 
Bas 111 Mathematics 3 Dr. Fatma Zedan El Emam
45
If a is any vector having the same direction as u
we shall also refer to )
,
(
u y
x
f
D as the directional
derivative of f in the direction of a.
Theorem 1 If f is a differentiable function of x and
y, then f has a directional derivative in the
direction of any unit vector j
u
i
u 2
1
u 
 and
2
1
u )
,
(
)
,
(
)
,
( u
y
x
f
u
y
x
f
y
x
f
D y
x 
 (2)
Example 1 Find the directional derivative of
2
3
)
,
( y
x
y
x
f  at the point ( - 1, 2) in the direction
of the vector a = 4i - 3j.
Solution We wish to find )
2
,
1
(
u 
f
D , where u is a
unit vector having the direction of a. Since
j
i
j
i
a
a
5
3
5
4
3
4
)
3
4
(
u 2
2





 ,
2
2
3
)
,
( y
x
y
x
fx  and ,
2
)
,
( 3
y
x
y
x
fy 
it follows that
2
1
u )
,
(
)
,
(
)
,
( u
y
x
f
u
y
x
f
y
x
f
D y
x 

Bas 111 Mathematics 3 Dr. Fatma Zedan El Emam
46
.
5
3
2
5
4
3 3
2
2














 y
x
y
x
Hence
.
12
5
12
5
48
5
3
)
2
(
)
1
(
2
5
4
)
2
(
)
1
(
3
)
2
,
1
( 3
2
2
u





















f
D
• If the unit vector u makes an angle  with the
positive x-axis (as in Fig. 2), then we can write
j
i 
 sin
cos
u 
 and the formula in Theorem 1
becomes

 sin
)
,
(
cos
)
,
(
)
,
(
u y
x
f
y
x
f
y
x
f
D y
x 
 (3)
Fig. 2
Bas 111 Mathematics 3 Dr. Fatma Zedan El Emam
47
Example 2 Find the directional derivative of
xy
e
y
x
f 
)
,
( at t point ( - 2, 0) in the direction of
the unit vector that makes an angle of 3
/
 with
the positive x-axis
Solution The partial derivatives of f are
The unit vector u that makes an angle of 3
/
 with
the positive x-axis is
Thus, from (3)
1.7.2 The Gradient
The directional derivative )
,
(
u y
x
f
D can be
written as the dot product of the unit vector
Bas 111 Mathematics 3 Dr. Fatma Zedan El Emam
48
j
u
i
u 2
1
u 

and the vector
j
y
x
f
i
y
x
f y
x )
,
(
)
,
( 
Thus,
2
1
2
1
u
)
,
(
)
,
(
]
)
,
(
)
,
(
[
)
(
)
,
(
u
y
x
f
u
y
x
f
j
y
x
f
i
y
x
f
j
u
i
u
y
x
f
D
y
x
y
x






The vector j
y
x
f
i
y
x
f y
x )
,
(
)
,
(  plays an important
role in many other computations and is given a
special name.
Definition 2 Let f be a function of x and y. The
gradient of f is the vector function
j
y
x
f
i
y
x
f
y
x
f y
x )
,
(
)
,
(
)
,
( 

 (4)
The symbol f
 (read “nabla”)
Example 3 If ,
sin
)
,
( xy
e
x
y
x
f 
 then
j
xe
i
ye
x
j
y
x
f
i
y
x
f
y
x
f
xy
xy
y
x






)
(cos
)
,
(
)
,
(
)
,
(
Bas 111 Mathematics 3 Dr. Fatma Zedan El Emam
49
and i
f 2
)
1
,
0
( 

Example 4 Find the gradient of
x
y
y
x
y
x
f ln
sin
)
,
( 
 at the point )
,
( 
e .
Solution Since
we have
So the gradient of f at )
,
( 
e is
Formula (3) can now be written as
u
)
,
(
)
,
(
u 

 y
x
f
y
x
f
D (5)
Example 5 Find the directional derivative of the
function y
y
x
y
x
f 4
)
,
( 3
2

 at the point (2, -1) in
the direction of the vector .
5
2
v j
i 

Bas 111 Mathematics 3 Dr. Fatma Zedan El Emam
50
Solution We first compute the gradient vector at
(2, -1):
j
y
x
i
xy
y
x
f )
4
3
(
2
)
,
( 2
2
3




j
i
f 8
4
)
1
,
2
( 




Note that v is not a unit vector, but since
29
v  , the unit vector in the direction of v is
j
i
j
i
29
5
29
2
5
2
)
5
2
(
v
v
u 2
2






Therefore, by Equation 5, we have
 
     
29
32
29
5
8
2
4
29
5
29
2
8
4
u
)
1
,
2
(
)
1
,
2
(
u




















j
i
j
i
f
f
D
• Properties of the Gradient
Theorem 2 Suppose f is differentiable at the point
(x, y).
1. If 0,
)
,
( 
 y
x
f then 0
)
,
(
u 
y
x
f
D for every u.
Bas 111 Mathematics 3 Dr. Fatma Zedan El Emam
51
2. The maximum value of )
,
(
u y
x
f
D is )
,
( y
x
f
 ,
and this occurs when u has the same direction as
)
,
( y
x
f
 .
3. The minimum value of )
,
(
u y
x
f
D is )
,
( y
x
f

 ,
and this occurs when u has the direction of
)
,
( y
x
f

 .
Example 6 Let .
)
,
( 2 y
e
x
y
x
f  Find the maximum
value of a directional derivative at (-2, 0) and find
the unit vector in the direction in which the
maximum value occurs.
Solution Since
j
e
x
i
xe
j
y
x
f
i
y
x
f
y
x
f y
y
y
x
2
2
)
,
(
)
,
(
)
,
( 




the gradient of f at (−2, 0) is
j
i
f 4
4
)
0
,
2
( 




By Theorem 2, the maximum value of the
directional derivative is
2
4
32
4
)
4
(
)
0
,
2
( 2
2






f
Bas 111 Mathematics 3 Dr. Fatma Zedan El Emam
52
This maximum occurs in the direction of ∇f (−2, 0).
The unit vector in this direction is
j
i
j
i
f
f
2
1
2
1
)
4
4
(
2
4
1
0)
2,
(
0)
2,
(
u 









• Functions of Three Variables
Theorem 3 If f is a differentiable function of x, y
and z, then f has a directional derivative in the
direction of any unit vector k
u
j
u
i
u
u 3
2
1 

 and
3
2
1 )
,
,
(
)
,
,
(
)
,
,
(
)
,
,
(
u
z
y
x
f
u
z
y
x
f
u
z
y
x
f
z
y
x
f
D
z
y
x
u



(6)
The gradient of f is
k
z
y
x
f
j
z
y
x
f
i
z
y
x
f
z
y
x
f z
y
x )
,
,
(
)
,
,
(
)
,
,
(
)
,
,
( 



We also write
u
)
,
,
(
)
,
,
(
u 

 z
y
x
f
z
y
x
f
D (7)
Example 7 If z
yz
y
x
z
y
x
f 

 3
2
)
,
,
(
(a) Find the gradient of f
Bas 111 Mathematics 3 Dr. Fatma Zedan El Emam
53
(b) Find the directional derivative of f at the point
)
0
,
2
,
1
(  in the direction of the vector
.
2
2 k
j
i
a 


Solution (a) The gradient of f is
k
yz
j
z
x
xyi
k
z
y
x
f
j
z
y
x
f
i
z
y
x
f
z
y
x
f z
y
x
)
1
3
(
)
(
2
)
,
,
(
)
,
,
(
)
,
,
(
)
,
,
(
2
3
2










(b) At (1,−2, 0) we have k
j
i
f 




 4
)
0
,
2
,
1
(
Since a is not a unit vector, we normalize it, getting
k
j
i
k
j
i
a
a
3
2
3
1
3
2
)
2
2
(
9
1
u 






Therefore Equation 7 gives
 
  3
3
2
3
1
3
2
4
3
2
3
1
3
2
4
u
)
0
,
2
,
1
(
)
0
,
2
,
1
(
u






























k
j
i
k
j
i
f
f
D
Bas 111 Mathematics 3 Dr. Fatma Zedan El Emam
54
Exercise 3
1-4 Find f
Du at P.
5-9 Find the directional derivative of f at P in the
direction of a.
10-11 Find the directional derivative of f at P in the
direction of a vector making the counterclockwise
angle θ with the positive x-axis.
Bas 111 Mathematics 3 Dr. Fatma Zedan El Emam
55
12-13 Find ∇z and ∇w.
14-15 Find the gradient of f at the indicated point.
16-18 Find a unit vector in the direction in which f
increases most rapidly at P, and find the rate of
change of f at P in that direction.
19-20 Find a unit vector in the direction in which f
decreases most rapidly at P, and find the rate of
change of f at P in that direction.
Bas 111 Mathematics 3 Dr. Fatma Zedan El Emam
56
CHAPTER 2
Multiple Integrals
The notion of a definite integral can be extended
to functions of two or more variables. In this
section we will discuss the double integral, which
is the extension to functions of two variables.
• Review of the Definite Integral
We know that the area A between the graph of f
and the interval [a, b] is

 








n
i
i
i
n
x
b
a
x
x
f
dx
x
f
A
i 1
0
max
)
(
lim
)
(
2.1 Volumes and Double Integrals over rectangles
In a similar manner, let us consider a function
Bas 111 Mathematics 3 Dr. Fatma Zedan El Emam
57
)
,
( y
x
f of two variable x and y that is continuous
on a rectangular region R in the xy-plane given by
R= {(x, y): d
y
c
b
x
a 


 , }
or ]
,
[
]
,
[ d
c
b
a
R 
 .
We imagine R to be covered by a net work of
lines parallel to the x- and y-axes (Fig. 2.1). These
lines divide the region R into small pieces of area
y
x
A 


 . We number these in some order
n
A
A
A
A 


 ,
,
,
, 3
2
1  , then the volume of
the solid enclosed between the surface
)
,
( y
x
f
z  and the region R is defined by

 









n
k
k
k
k
n
A
R
A
y
x
f
dA
y
x
f
V
k 1
0
)
,
(
lim
)
,
(
Here, 

n indicates the process of increasing
the number of subrectangles of the rectangle R in
such a way that both the lengths and the widths of
the subrectangles approach zero.
Bas 111 Mathematics 3 Dr. Fatma Zedan El Emam
58
Fig. 2.1

R
dA
y
x
f )
,
( is called the double integral of
)
,
( y
x
f over R.
• Properties Of Double Integrals
If ƒ(x, y) and g(x, y) are continuous, then
Bas 111 Mathematics 3 Dr. Fatma Zedan El Emam
59
(i) ,
)
,
(
)
,
( 
 
R
R
dA
y
x
f
c
dA
y
x
cf if c is any real
number.






R
R
R
dA
y
x
g
dA
y
x
f
dA
y
x
g
y
x
f
)
,
(
)
,
(
)]
,
(
)
,
(
[
(ii)
(iii) If R is the union of two nonoverlapping regions
1
R and 2
R


 

2
1
)
,
(
)
,
(
)
,
(
R
R
R
dA
y
x
f
dA
y
x
f
dA
y
x
f
(iv) 0
)
,
( 

R
dA
y
x
f if 0
)
,
( 
y
x
f on R.
• Evaluating Double Integrals
Fubini’s Theorem
If )
,
( y
x
f is continuous on ]
,
[
]
,
[ d
c
b
a
R 
 then,
dy
dx
y
x
f
dx
dy
y
x
f
dA
y
x
f
d
c
b
a
b
a
d
c
R
 
 















)
,
(
)
,
(
)
,
(
Bas 111 Mathematics 3 Dr. Fatma Zedan El Emam
60
These integrals are called iterated or repeated
integrals.
The symbols

b
a
dx
y
x
f )
,
( and 
d
c
dy
y
x
f )
,
(
denote partial definite integrals; the first integral,
called the partial definite integral w.r.t. x, is
evaluated by holding y fixed and integrating w.r.t.
x, and the second integral, called the partial
definite integral w.r.t. y, is evaluated by holding x
fixed and integrating w.r.t. y.
Example 1 Evaluate the iterated integrals.
(a) dx
dy
y
x
 
3
0
2
1
2
(b) dy
dx
y
x
 
2
1
3
0
2
Solution (a) Here we first integrate w. r. t. y:
Bas 111 Mathematics 3 Dr. Fatma Zedan El Emam
61
2
27
2
2
3
2
3
0
3
2
3
0
2
1
2
3
0
2
3
0
2
1
2
3
0
2
1
2

























 



x
dx
x
dx
y
x
dx
dy
y
x
dx
dy
y
x
y
y
(b) Here we first integrate w. r. t. x:
2
27
2
9
9
3
2
1
2
2
1
3
0
3
2
1
2
1
3
0
2
2
1
3
0
2

























 



y
dy
y
dy
x
y
dy
dx
y
x
dy
dx
y
x
x
x
Example 2 Evaluate the iterated integrals.
(a) dx
dy
y
x )
2
40
(
3
1
4
2
  
(b) dy
dx
y
x )
2
40
(
4
2
3
1
  
Solution
Bas 111 Mathematics 3 Dr. Fatma Zedan El Emam
62
 
  112
6
80
)
12
80
(
)]
4
80
(
)
16
160
[(
40
)
2
40
(
)
2
40
(
(a)
3
1
2
3
1
3
1
3
1
4
2
2
3
1
4
2
3
1
4
2























 
 

x
x
dx
x
dx
x
x
dx
xy
y
dx
dy
xy
dx
dy
y
x
y
 
  112
4
80
]
8
80
[
)]
40
(
)
9
120
[(
40
)
2
40
(
)
2
40
(
(b)
4
2
2
4
2
4
2
4
2
3
1
2
4
2
3
1
4
2
3
1























 


y
y
dy
y
dy
y
y
dy
y
x
x
dy
dx
xy
dy
dx
y
x
x
Bas 111 Mathematics 3 Dr. Fatma Zedan El Emam
63
Example 3 Calculate 
R
dA
y
x
f )
,
( for
y
x
y
x
f 2
6
1
)
,
( 
 and R: .
1
1
,
2
0 



 y
x
Solution
 
  .
4
8
2
)
16
2
(
2
)
6
1
(
)
,
(
1
1
2
1
1
2
0
1
1
3
1
1
2
0
2

















 

y
y
dy
y
dy
y
x
x
dy
dx
y
x
dA
y
x
f
x
x
R
Another Sol.
  dx
y
x
y
dx
dy
y
x
dA
y
x
f
y
y
R
1
1
2
0
2
2
2
0
1
1
2
3
)
6
1
(
)
,
(





 





Bas 111 Mathematics 3 Dr. Fatma Zedan El Emam
64









2
0
2
0
2
2
4
2
)]
3
1
(
)
3
1
[(
dx
dx
x
x
Example 4
Use a double integral to find the volume of the
solid that is bounded above by the plane
y
x
z 

 4 and below by the rectangle
]
2
,
0
[
]
1
,
0
[ 

R (Fig. 2.2).
Solution
The volume is the double integral of y
x
z 

 4
over R, this can be obtained from either of the
iterated integrals
  

2
0
1
0
)
4
( dy
dx
y
x or dx
dy
y
x
  

1
0
2
0
)
4
(
Using the first of these, we obtain
Bas 111 Mathematics 3 Dr. Fatma Zedan El Emam
65
 
 





2
0
1
0
)
4
(
)
4
( dy
dx
y
x
dA
y
x
V
R
  

















2
0
2
0
1
0
2
2
7
2
4 dy
y
dy
xy
x
x
x
5
2
2
7
2
0
2









y
y
You can check this result by evaluating the second
integral.
Fig. 2.2
Example 5 Evaluate 
R
dA
x
y2
,
 
 
1
0
,
2
3
:
, 




 y
x
y
x
R .
Bas 111 Mathematics 3 Dr. Fatma Zedan El Emam
66
Solution
6
5
6
5
2
5
2
1
1
0
3
1
0
2
1
0
2
3
2
2
1
0
2
3
2
2




























 
 


y
dy
y
dy
x
y
dy
dx
x
y
dA
x
y
x
R
Fact: If )
(
)
(
)
,
( y
h
x
g
y
x
f  and we are integrating
over the rectangle ]
,
[
]
,
[ d
c
b
a
R 
 then,






















dy
y
h
dx
x
g
dA
y
h
x
g
dA
y
x
f
d
c
b
a
R
R
)
(
)
(
)
(
)
(
)
,
(
Example 6 Evaluate 
R
dA
y
x 2
cos ,
]
2
/
,
0
[
]
3
,
2
[ 



R .
Solution
















 



2
/
0
2
3
2
2
cos
cos

dy
y
dx
x
dA
y
x
R
Bas 111 Mathematics 3 Dr. Fatma Zedan El Emam
67
8
5
2
sin
2
1
2
1
2
5
)
2
cos
1
(
2
1
2
1
2
0
2
/
0
3
2
2





































 

y
y
dy
y
x
Example 7 Evaluate

R
dA
y
xcos
sin , where ]
2
/
,
0
[
]
2
/
,
0
[ 
 

R
Solution
















 


2
/
0
2
/
0
cos
sin
cos
sin


dy
y
dx
x
dA
y
x
R
   
1
1
1
sin
cos
2
/
0
2
/
0







y
x
Bas 111 Mathematics 3 Dr. Fatma Zedan El Emam
68
Exercise 4
1-8 Evaluate the iterated integrals.
9-12 Evaluate the double integral over the
rectangular region R.
Bas 111 Mathematics 3 Dr. Fatma Zedan El Emam
69
13-15 Use a double integral to find the volume.
Bas 111 Mathematics 3 Dr. Fatma Zedan El Emam
70
• Iterated Integrals With Nonconstant Limits Of
Integration
Example 8 Evaluate
(a)  

1
0
2
2
x
x
dx
dy
x
y (b) dy
dx
y
x
y
sin
3
/
0
cos
0
 

Solution
120
13
15
24
3
3
3
)
(
1
0
5
8
1
0
4
7
1
0
3
1
0
2
1
0
2
2
2
2











































 
 




x
x
dx
x
x
dx
x
y
dx
dy
x
y
dx
dy
x
y
a
x
x
y
x
x
x
x
dy
y
x
dy
dx
y
x
dy
dx
y
x
b
y
x
y
y
cos
0
3
/
0
2
3
/
0
cos
0
3
/
0
cos
0
sin
2
sin
sin
)
(


 
 

















Bas 111 Mathematics 3 Dr. Fatma Zedan El Emam
71
48
7
cos
6
1
sin
cos
2
1
3
/
0
3
3
/
0
2















 


y
dy
y
y
2.2 Double Integrals over General Regions D
We will limit our study of double integrals to two
basic types of regions, which we will call type I and
type II; they are defined as follows.
A plane region D is said to be of type I if it lies
between the graphs of two continuous functions
of x, that is,
 
)
(
)
(
,
)
,
( 2
1 x
g
y
x
g
b
x
a
y
x
D 




Bas 111 Mathematics 3 Dr. Fatma Zedan El Emam
72
Some type I regions
Also, the plane region D is said to be of type II if it
lies between the graphs of two continuous
functions of y, that is,
 
)
(
)
(
,
)
,
( 2
1 y
h
x
y
h
d
y
c
y
x
D 




Some type II regions
Bas 111 Mathematics 3 Dr. Fatma Zedan El Emam
73
Theorem 2 Fubini’s Theorem (Stronger form)
Let )
,
( y
x
f is continuous on a regionD.
1. If D is defined by b
x
a 
 ,
)
(
)
( 2
1 x
g
y
x
g 
 with 1
g and 2
g continuous
on [a, b], (D is a type I region) then
dx
dy
y
x
f
dA
y
x
f
b
a
x
g
x
g
D
 
 
)
(
)
(
2
1
)
,
(
)
,
( .
2. If D is defined by ,
d
y
c 
 )
(
)
( 2
1 y
h
x
y
h 

with 1
h and 2
h continuous on [c, d], ], (D is a type
II region) then
dy
dx
y
x
f
dA
y
x
f
d
c
y
h
y
h
D
 
 
)
(
)
(
2
1
)
,
(
)
,
( .
Example 9 Evaluate  
D
dA
y
x )
2
( , whereD
is the region bounded by the parabolas
2
2x
y 
and .
1 2
x
y 

Solution
Bas 111 Mathematics 3 Dr. Fatma Zedan El Emam
74
The parabolas intersect when ,
1
2 2
2
x
x 
 that is,
,
1
2

x so .
1


x We note that the region D,
sketched in Fig.1, is a type I region but not a type II
region and we can write
 
2
2
1
2
,
1
1
)
,
( x
y
x
x
y
x
D 






Since the lower boundary is
2
2x
y  and the upper
boundary is
2
1 x
y 
 , then
 
dx
x
x
x
x
x
x
dx
y
xy
dx
dy
y
x
dA
y
x
x
y
x
y
x
x
D


 



















1
1
2
2
2
2
2
2
1
2
1
1
2
1
1
1
2
]
)
2
(
)
2
(
)
1
(
)
1
(
[
)
2
(
)
2
(
2
2
2
2
dx
x
x
x
x








1
1
2
3
4
)
1
2
3
(
15
32
2
3
2
4
5
3
1
1
2
3
4
5














x
x
x
x
x
.
Bas 111 Mathematics 3 Dr. Fatma Zedan El Emam
75
Fig. 1
Example 10
Find the volume of the solid that lies under the
paraboloid 2
2
y
x
z 
 and above the region D in
the xy-plane bounded by the line x
y 2
 and the
parabola 2
x
y  .
Solution 1 From Fig. 2 we see that D is a type I
region and
 
x
y
x
x
y
x
D 2
,
2
0
)
,
( 2





Therefore the volume under 2
2
y
x
z 
 and
above D is
Bas 111 Mathematics 3 Dr. Fatma Zedan El Emam
76
35
216
6
7
5
21
3
14
3
3
)
(
3
)
2
(
)
2
(
3
)
(
)
(
2
0
4
5
7
2
0
3
4
6
2
0
3
2
2
2
3
2
2
0
2
3
2
2
0
2
2
2
2
2
2
2
















































 



x
x
x
dx
x
x
x
dx
x
x
x
x
x
x
dx
y
y
x
dx
dy
y
x
dA
y
x
V
x
y
x
y
x
x
D
Fig. 2
D as a type I region
Bas 111 Mathematics 3 Dr. Fatma Zedan El Emam
77
Solution 2 From Fig. 3 we see that D can also be
written as a type II region and










 y
x
y
y
y
x
D
2
1
,
4
0
)
,
(
Therefore another expression for V is
35
216
96
13
7
2
15
2
2
24
3
3
)
(
)
(
4
0
4
2
/
7
2
/
5
4
0
3
3
2
/
5
2
/
3
4
0
2
3
4
0
2
2
2
2
2
1
2
1


































 



y
y
y
dy
y
y
y
y
dy
x
y
x
dy
dx
y
x
dA
y
x
V
y
x
y
x
y
y
D
Fig. 3 D as a type II region
Bas 111 Mathematics 3 Dr. Fatma Zedan El Emam
78
Example 11 Evaluate 
R
dA
xy over the region R
enclosed between ,
2
1
x
y  ,
x
y  2

x and
4

x .
Solution We view R as a type I region.
dx
xy
dx
dy
xy
dA
xy
x
x
y
x
x
R

 










4
2 2
/
2
4
2 2
/ 2
4
2
4
3
4
2
3
2
32
6
8
2 














 
x
x
dx
x
x
6
11
32
16
6
8
32
256
6
64

















Bas 111 Mathematics 3 Dr. Fatma Zedan El Emam
79
Example 12 Evaluate  
R
dA
y
x )
2
( 2
over the
triangular region R enclosed between the lines
,
1


 x
y 1

 x
y and 3

y
Solution 1 We view R as a type II region
 
 
3
68
2
3
2
)
2
2
(
)
2
1
(
)
2
2
1
(
)
2
(
)
2
(
3
1
4
3
3
1
3
2
3
1
3
3
2
3
1
1
1
2
2
3
1
1
1
2
2



























 







y
y
dy
y
y
dy
y
y
y
y
y
dy
x
y
x
dy
dx
y
x
dA
y
x
y
x
y
x
y
y
R
Bas 111 Mathematics 3 Dr. Fatma Zedan El Emam
80
Solution 2 We view R as a type I region.




 
 








































 

2
0
3
0
2
3
2
0
3
1
3
0
2
3
1
3
2
0
3
1
2
0
2
3
1
2
2
2
2
)]
3
)
1
(
)
1
(
2
(
)
9
6
[(
)]
3
)
1
(
)
1
(
2
(
)
9
6
[(
]
3
2
[
]
3
2
[
)
2
(
)
2
(
)
2
(
)
2
(
)
2
(
2
1
dx
x
x
x
x
dx
x
x
x
x
dx
y
y
x
dx
y
y
x
dx
dy
y
x
dx
dy
y
x
dA
y
x
dA
y
x
dA
y
x
y
x
y
y
x
y
x
x
R
R
R
3
68
6
52
6
84
)]
)
4
(
3
)
1
(
3
2
9
3
[
]
)
4
(
3
)
1
(
3
2
9
3
[
4
2
3
2
0
2
4
2
3
2

















 
x
x
x
x
x
x
x
x
x
x
Bas 111 Mathematics 3 Dr. Fatma Zedan El Emam
81
Example 13
Use a double integral to find the volume of the
tetrahedron bounded by the coordinate planes
and the plane .
2
4
4 y
x
z 


Solution
The tetrahedron is bounded above by the plane
y
x
z 2
4
4 

 (1)
and below by the triangular region R shown in Fig.
4. Thus, the volume is given by
dA
y
x
V
R
 

 )
2
4
4
(
Bas 111 Mathematics 3 Dr. Fatma Zedan El Emam
82
The region R is bounded by the x-axis, the y-axis,
and the line y = 2 − 2x [set z = 0 in (1)], so that
treating R as a type I region yields
 
3
4
)
4
8
4
(
4
4
)
2
4
4
(
)
2
4
4
(
1
0
2
1
0
2
2
0
2
1
0
2
2
0















 




dx
x
x
dx
y
xy
y
dx
dy
y
x
dA
y
x
V
x
y
x
R
Fig. 4
Bas 111 Mathematics 3 Dr. Fatma Zedan El Emam
83
• Reversing The Order Of Integration
Sometimes the evaluation of an iterated integral
can be simplified by reversing the order of
integration. The next example illustrates how this
is done.
Example 14 Evaluate
 
2
0
1
2
/
2
y
x
dy
dx
e
Solution
[Since there is no elementary antiderivative of
2
x
e ,
the integral cannot be evaluated by performing
the x-integration first. Evaluate this integral by
expressing it as an equivalent iterated integral
with the order of integration reversed, so we must
change the order of integration.] To reverse the
order of integration, we treat R as a type I region,
which enables us to write the given integral as
Bas 111 Mathematics 3 Dr. Fatma Zedan El Emam
84
 
  1
2
1
0
1
0
1
0
2
0
1
0
2
0
2
0
1
2
/
2
2
2
2
2
2









 

 

e
e
dx
e
x
dx
y
e
dx
dy
e
dA
e
dy
dx
e
x
x
x
y
x
x
x
R
x
y
x
Example 15 Evaluate
 
1
0
1
2
.
sin
x
dx
dy
y
Solution
Here, we must change the order of integration
(why). So, we treat D as a type II region
Bas 111 Mathematics 3 Dr. Fatma Zedan El Emam
85
 
)
1
cos
1
(
2
1
cos
2
1
sin
sin
sin
sin
sin
1
0
2
1
0
2
1
0
0
2
1
0 0
2
1
0
2
1
2
















 
 



y
dy
y
y
dy
y
x
dy
dx
y
dA
y
dx
dy
y
y
x
x
y
D
x
D as a type I region D as a type II region
• Area Calculated As A Double Integral
Definition: The area of a closed, bounded plane
region R is


R
dA
A . (2)
To evaluate the integral in (2), we integrate the
constant function f (x, y)=1 over R.
Bas 111 Mathematics 3 Dr. Fatma Zedan El Emam
86
Example 16 Find the area of the region R
bounded by x
y  and 2
x
y  in the first quadrant
Solution We sketch the region (Fig. 5) and
calculate the area as
 
.
6
1
3
2
)
(
1
0
3
2
1
0
2
1
0
1
0
2
2












 

 
x
x
dx
x
x
dx
y
dx
dy
A x
x
x
x
Fig. 5
Example 17 Use a double integral to find the area
of the region R enclosed between the parabola
2
2
1
x
y  and the line y = 2x.
Bas 111 Mathematics 3 Dr. Fatma Zedan El Emam
87
Solution The region R may be treated equally well
as type I (Fig. 6a) or type II (Fig. 6b). Treating R as
type I yields
 
.
3
16
6
2
1
2
of
area
4
0
3
2
4
0
2
4
0
2
2
/
4
0
2
2
/
2
2






















 
 
x
x
dx
x
x
dx
y
dx
dy
dA
R x
x
y
x
x
R
Treating R as type II yields
 
.
3
16
4
3
2
2
2
1
2
of
area
8
0
2
2
/
3
8
0
8
0
2
2
/
8
0
2
2
/






















 
 
y
y
dy
y
y
dy
x
dy
dx
dA
R y
y
x
y
y
R
Bas 111 Mathematics 3 Dr. Fatma Zedan El Emam
88
Exercises 5
1–8 Evaluate the iterated integral.
9-10 Evaluate the double integral in two ways
using iterated integrals: (a) viewing R as a type I
region, and (b) viewing R as a type II region.
11–12 Evaluate the double integral.
Bas 111 Mathematics 3 Dr. Fatma Zedan El Emam
89
13–14 Use double integration to find the area of
the plane region enclosed by the given curves.
15 Use double integration to find the volume of
the solid.
16–17 Evaluate the integral by first reversing the
order of integration.
Bas 111 Mathematics 3 Dr. Fatma Zedan El Emam
90
2.3 Double Integrals in Polar Coordinates
•Converting Double Integrals From Rectangular
To Polar Coordinate
Sometimes a double integral that is difficult to
evaluate in rectangular coordinates can be
evaluated more easily in polar coordinates by
making the substitution x = r cos θ, y = r sin θ
and expressing the region of integration in polar
form; that is, we rewrite the double integral in
rectangular coordinates as





d
dr
r
r
r
f
dA
r
r
f
dA
y
x
f
R
R





limits
e
appropriat
)
sin
,
cos
(
)
sin
,
cos
(
)
,
(
Bas 111 Mathematics 3 Dr. Fatma Zedan El Emam
91
Notice that d x d y is not replaced by dr dθ but by
r dr dθ.
Example 1 Use polar coordinates to evaluate
 



1
1
1
0
2
/
3
2
2
2
)
(
x
dx
dy
y
x .
Solution
5
5
1
)
(
)
(
)
(
0
0
1
0
3
2
/
3
2
2
1
1
1
0
2
/
3
2
2
2












 

 


d
d
dr
r
r
dA
y
x
dx
dy
y
x
R
x
Fig. 1
Bas 111 Mathematics 3 Dr. Fatma Zedan El Emam
92
Example 2 Use polar coordinates to evaluate
,
)
4
3
( 2
 
R
dA
y
x
where R is the region in the upper half-plane
bounded by the circles
1
2
2

 y
x and .
4
2
2

 y
x
Solution The region R can be described as
 
4
1
,
0
)
,
( 2
2




 y
x
y
y
x
R
It is the half-ring shown in Figure 2(b), and in polar
coordinates it is given by .
0
,
2
1 
 


 r
Therefore,
 

 
 






















0
2
1
2
4
3
0
2
1
2
3
2
0
2
1
2
2
2
sin
cos
)
sin
4
cos
3
(
)
sin
4
cos
3
(
)
4
3
(
d
r
r
d
dr
r
r
d
dr
r
r
r
dA
y
x
r
r
R
Bas 111 Mathematics 3 Dr. Fatma Zedan El Emam
93
2
15
)
2
sin
4
15
2
15
sin
7
)
2
cos
1
(
2
15
cos
7
)
sin
15
cos
7
(
0
0
0
2




































d
d
 


 2
0
,
1
0
)
,
(
)
( 



 r
r
R
a
 


 



 0
,
2
1
)
,
(
)
( r
r
R
b
Fig. 2
Bas 111 Mathematics 3 Dr. Fatma Zedan El Emam
94
If )
,
( 
r
f is the constant function whose
value is 1, then the integral of f over R is the area
of R.
Area in Polar Coordinates
The area of a closed and bounded region R in the
polar coordinate plane is
,
.


R
d
dr
r
A 
Example 3
Use a polar double integral to find the area
enclosed by the three-petaled rose r = sin 3θ.
Bas 111 Mathematics 3 Dr. Fatma Zedan El Emam
95
Solution We will calculate the area of the petal R
in the first quadrant and multiply by 3.











 
4
1
6
6
sin
4
3
)
6
cos
1
(
4
3
3
sin
2
3
3
3
3
/
0
3
/
0
3
/
0
2
3
/
0
3
sin
0
















 

d
d
d
dr
r
dA
A
R
2.4 Triple Integral
As in the case of double integrals, a triple integral
may be found by evaluating an appropriate
iterated integral.
Theorem 1 Let G be the rectangular box defined
by the inequalities
a ≤ x ≤ b, c ≤ y ≤ d, k ≤ z ≤ l
If f is continuous on the region G, then
Bas 111 Mathematics 3 Dr. Fatma Zedan El Emam
96
Moreover, the iterated integral on the right can be
replaced with any of the five other iterated
integrals that result by altering the order of
integration.
Example 4 Evaluate the triple integral

G
dV
z
xy 3
2
12
over the rectangular box G defined by the
inequalities
−1 ≤ x ≤ 2, 0 ≤ y ≤ 3, 0 ≤z ≤ 2.
Solution. Of the six possible iterated integrals we
might use, we will choose the one in (1). Thus, we
will first integrate with respect to z , holding x and
y fixed, then with respect to y, holding x fixed, and
finally with respect to x.
Bas 111 Mathematics 3 Dr. Fatma Zedan El Emam
97
Example 5 Evaluate the triple integral
where B is the rectangular box given by
Solution. We could use any of the six possible
orders of integration. If we choose to integrate
with respect to x, then y, and then z, we obtain
Bas 111 Mathematics 3 Dr. Fatma Zedan El Emam
98
• Evaluating Triple Integrals Over More General
Regions
Theorem 2 Let G be a simple xy-solid with upper
surface )
,
(
2 y
x
g
z  and lower surface )
,
(
1 y
x
g
z 
, and let R be the projection of G on the xy -plane.
If f(x, y, z) is continuous on G, then
Bas 111 Mathematics 3 Dr. Fatma Zedan El Emam
99
Example 6 Let G be the wedge in the first octant
that is cut from the cylindrical solid 1
2
2

 z
y by
the planes y = x and x = 0. Evaluate 
G
dV
z
Solution. The solid G and its projection R on the
xy-plane are shown in Figure 4. The upper surface
of the solid is formed by the cylinder and the lower
surface by the xy-plane. Since the portion of the
cylinder 1
2
2

 z
y that lies above the xy-plane
has the equation ,
1 2
y
z 
 and the xy-plane has
the equation z = 0, it follows from (2) that
For the double integral over R, the x- and y-
integrations can be performed in either order,
since R is both a type I and type II region. We will
integrate with respect to x first. With this choice,
(3) yields
Bas 111 Mathematics 3 Dr. Fatma Zedan El Emam
100
Fig. 4
Example 7 Evaluate ,

E
dV
z where E is the
solid tetrahedron bounded by the four planes x =
0, y = 0, z = 0, and .
1


 z
y
x
Bas 111 Mathematics 3 Dr. Fatma Zedan El Emam
101
Solution. When we set up a triple integral it’s wise
to draw two diagrams: one of the solid region E
(see Figure 5) and one of its projection D on the
xy-plane (see Figure 6). The lower boundary of the
tetrahedron is the plane z = 0 and the upper
boundary is the plane 1


 z
y
x (or y
x
z 

1
). Notice that the planes 1


 z
y
x and z= 0
intersect in the line 1

 y
x (or x
y 
1 ) in the
xy-plane. So the projection of E is the triangular
region shown in Figure 6,
and
Bas 111 Mathematics 3 Dr. Fatma Zedan El Emam
102
• Volume Calculated As A Triple Integral
of
volume 

G
dV
G (4)
Example 8 Use a triple integral to find the volume
of the solid within the cylinder 9
2
2

 y
x and
between the planes z = 1 and x + z = 5.
Solution. The solid G and its projection R on the
xy-plane are shown in Fig. 7. The lower surface of
the solid is the plane z = 1 and the upper surface
Bas 111 Mathematics 3 Dr. Fatma Zedan El Emam
103
is the plane x + z = 5 or, equivalently, z = 5 − x.
Thus, from (2) and (4)
For the double integral over R, we will integrate
with respect to y first. Thus, (5) yields
Where,
Bas 111 Mathematics 3 Dr. Fatma Zedan El Emam
104
and the second integral is 0 because the integrand
is an odd function.
Fig. 7
Example 9 Find the volume of the solid enclosed
between the paraboloids 2
2
5
5 y
x
z 
 and
2
2
7
6 y
x
z 

 .
Solution. The solid G and its projection R on the
xy-plane are shown in Figure 7. The projection R is
obtained by solving the given equations
simultaneously to determine where the
paraboloids intersect. We obtain
Bas 111 Mathematics 3 Dr. Fatma Zedan El Emam
105
or
which tells us that the paraboloids intersect in a
curve on the elliptic cylinder given by (6).
The projection of this intersection on the xy-plane
is an ellipse with this same equation. Therefore,
Bas 111 Mathematics 3 Dr. Fatma Zedan El Emam
106
Fig. 7
• Integration In Other Orders
In Formula (2) for integrating over a simple xy-
solid, the 
z integration was performed first.
However, there are situations in which it is
preferable to integrate in a different order. For
example, Figure 8a shows a simple xz-solid, and
Figure 8b shows a simple yz -solid. For a simple xz
-solid it is usually best to integrate with respect to
y first, and for a simple yz -solid it is usually best to
integrate with respect to x first:
Bas 111 Mathematics 3 Dr. Fatma Zedan El Emam
107
A simple xz-solid A simple yz -solid
(a) Fig. 8 (b)
Example 10 In Example 6 we evaluated

G
dV
z
over the wedge in Fig. 4 by integrating first with
respect to z. Evaluate this integral by integrating
first with respect to x.
Solution. The solid is bounded in the back by the
plane x = 0 and in the front by the plane y
x  , so
Bas 111 Mathematics 3 Dr. Fatma Zedan El Emam
108
where R is the projection of G on the yz -plane
(Fig. 9). The integration over R can be performed
first with respect to z and then y or vice versa.
Performing the z-integration first yields
which agrees with the result in Example 6.
Fig. 8
Bas 111 Mathematics 3 Dr. Fatma Zedan El Emam
109
Exercises 6
1-4 Evaluate the iterated integral.
5-7 Evaluate the iterated integral by converting to
polar coordinates.
8-11 Evaluate the iterated integral.
Bas 111 Mathematics 3 Dr. Fatma Zedan El Emam
110
12-13 Evaluate the triple integral.
14-16 Use a triple integral to find the volume of
the solid.
Bas 111 Mathematics 3 Dr. Fatma Zedan El Emam
111
CHAPTER 3
First-Order Differential Equation
3.1 Introduction
A differential equation (DE) is any equation which
contains derivatives, either ordinary derivatives or
partial derivatives.
we shall classify differential equations by type,
order, and linearity.
Classification By Type: An equation involving only
ordinary derivatives w.r.to a single independent
variable is ordinary differential equation (ODE).
For example,
An equation involving partial derivatives w.r.to
more than one independent variables is called a
partial differential equation (PDE). For example,
Bas 111 Mathematics 3 Dr. Fatma Zedan El Emam
112
Classification By Order: The order of a differential
equation (either ODE or PDE) is the order of the
highest derivative in the equation. For example,
is a second-order ODE.
Classification By Linearity: An nth-order ODE
).
(
)
(
)
(
)
(
)
( 0
1
1
1
1 x
g
y
x
a
dx
dy
x
a
dx
y
d
x
a
dx
y
d
x
a n
n
n
n
n
n 



 

 
is said to be linear since it satisfies the following
two conditions
1) It is linear in
)
(
,
,
, n
y
y
y 
 . This means that, the
dependent variable y and all its derivatives
)
(
,
,
, n
y
y
y 
 are of the first degree, that is, the
power of each term involving y is 1.
2) The coefficients n
a
a
a ,
,
, 1
0  of
)
(
,
,
, n
y
y
y 

depend at most on the independent variable x.
Bas 111 Mathematics 3 Dr. Fatma Zedan El Emam
113
For example, the equations
Are linear first, second, and third-order ODE s,
where the first equation can be written of the
form x
y
y
x 


4 . A nonlinear ODE is simply one
that is not linear. Nonlinear functions of the
dependent variable or its derivatives, such as sin y
or , cannot appear in a linear equation. Therefore
nonlinear term:
coefficient depends on y
x
e
y
y
y 




2
)
1
( ,
nonlinear term:
nonlinear function of y
0
sin
2
2



y
dx
y
d
and
nonlinear term:
power not 1
0
2
4
4



y
dx
y
d
Bas 111 Mathematics 3 Dr. Fatma Zedan El Emam
114
are examples of nonlinear first, second, and
fourth-order ordinary differential equations,
respectively.
• The degree of a DE is the degree of the highest
derivative after removing the radical sign and
fraction.
The solution of DE
An equation containing dependent variable y and
independent variable x and free from derivative,
which satisfies the DE, is called the solution of the
DE.
Bas 111 Mathematics 3 Dr. Fatma Zedan El Emam
115
Note: The number of arbitrary constants in the
solution is equal to the order of the differential
equation.
• Initial Condition(s)
initial conditions are values of the solution and/or
its derivative(s) at specific points of the form,
0
0 )
( y
x
y  and/or k
k
y
x
y 
)
( 0
)
(
• Initial Value Problem
An Initial Value Problem ( IVP) is a differential
equation along with an appropriate number of
initial conditions. For example
0
0 )
(
),
,
( y
x
y
y
x
f
dx
dy


and
1
0
0
0
2
2
)
(
,
)
(
),
,
,
( y
x
y
y
x
y
y
y
x
f
dx
y
d





are first and second-order initial-value problems,
respectively.
Bas 111 Mathematics 3 Dr. Fatma Zedan El Emam
116
Note: The number of initial conditions required
will depend on the order of the differential
equation.
3.2 Solutions Of First ODEs
3.2.1 Separable Equations
Definition 3.2.1
A first-order differential equation of the form
dx
x
g
dy
y
h )
(
)
( 
is said to be separable or to have separable
variables.
The solution to the first-order separable
differential equation
dx
x
g
dy
y
h )
(
)
( 
is
C
dx
x
g
dy
y
h 
 
 )
(
)
(
Bas 111 Mathematics 3 Dr. Fatma Zedan El Emam
117
Example 2 Solve the differential equation
0
)
4
(
6
2 2




 y
x
x
xy .
Solution. The equation may be written
.
0
)
4
(
)
3
(
2 2




dx
dy
x
y
x
Assuming that ,
0
)
4
)(
3
( 2


 x
y we may
divide both sides by this product, then
0
3
1
4
2
2




dy
y
dx
x
x
.
Thus the given equation is separable, and
integration of each term gives us
.
3
4
)
3
)(
4
(
)
3
)(
4
(
)
3
)(
4
(
ln
3
ln
4
ln
3
1
4
2
2
2
2
2
1
2
1
2
1
2
1

























 

x
C
y
C
y
x
C
e
y
x
C
y
x
C
y
x
C
dy
y
dx
x
x
C
Bas 111 Mathematics 3 Dr. Fatma Zedan El Emam
118
Example 3 Solve the differential equation
.
0
2
4


dx
dy
e
y x
Solution. Rewrite the equation as
.
3
2
6
where
2
3
6
by
sides
both
multiply
3
1
2
1
1
0
1
0
3
/
1
2
1
3
2
1
3
2
1
4
2
4
2
2
4




























C
e
y
C
C
C
y
e
C
y
e
C
dy
y
dx
e
dy
y
dx
e
dy
dx
e
y
x
x
x
x
x
x
Example 4 Solve
.
4
2

 y
dx
dy
Solution. We put the equation in the form
Bas 111 Mathematics 3 Dr. Fatma Zedan El Emam
119
.
4
2
dx
y
dy


By using partial fractions, we get
dx
dy
y
y













 2
4
1
2
4
1
.
1
1
2
by
replacing
2
2
2
2
2
2
4
2
2
ln
4
2
ln
2
ln
4
2
1
2
1
4
4
4
4
4
1
1
1
1
1
1
1
x
x
C
x
C
x
C
C
x
Ce
Ce
y
C
e
e
e
y
y
e
e
y
y
e
y
y
C
x
y
y
C
x
y
y
C
dx
dy
y
y












































Bas 111 Mathematics 3 Dr. Fatma Zedan El Emam
120
Example 5 Solve the IVP
.
0
)
0
(
,
2
sin
cos
)
( 2


 y
x
e
dx
dy
x
y
e y
y
Solution. Dividing the equation by x
ey
cos gives
dx
x
x
dy
e
y
e
y
y
cos
2
sin
2



 











xdx
dy
ye
e
xdx
dy
ye
e
dx
x
x
x
dy
ye
e
y
y
y
y
y
y
sin
2
)
(
sin
2
)
(
cos
cos
sin
2
)
(
,
cos
2 C
x
e
ye
e y
y
y





 

[ where
y
y
y
e
ye
dy
ye 





 by integrating
by parts]. The initial condition y = 0 when x = 0
implies C= 4. Thus a solution of the IVP is
.
cos
2
4 x
e
ye
e y
y
y



 

Bas 111 Mathematics 3 Dr. Fatma Zedan El Emam
121
3.2.2 Exact Differential Equation
Definition 3.2.2
A first-order differential equation of the form
0
)
,
(
)
,
( 
 dy
y
x
N
dx
y
x
M
is said to an exact differential equation provided
.
x
N
y
M





The solution to the first-order exact differential
equation
0
)
,
(
)
,
( 
 dy
y
x
N
dx
y
x
M
is
C
dy
x
N
dx
M
y

 
 )
containing
not
of
terms
(
const.
as
Example 6 Solve
0
)
5
3
2
(
)
2
3
5
( 4
2
2
3
3
2
2
4





 dy
y
y
x
y
x
dx
xy
y
x
x
Solution. Here,
Bas 111 Mathematics 3 Dr. Fatma Zedan El Emam
122
.
6
6
,
6
6
5
3
2
,
2
3
5
2
2
2
2
4
2
2
3
3
2
2
4
x
N
y
M
xy
y
x
x
N
xy
y
x
y
M
y
y
x
y
x
N
xy
y
x
x
M




















Then, the given equation is exact and the solution
is
C
dy
x
N
dx
M
y

 
 )
containing
not
of
terms
(
const.
as
C
y
y
x
y
x
x
C
dy
y
dx
xy
y
x
x
y









 

5
3
2
2
3
5
4
3
2
2
4
5
)
2
3
5
(
const.
as
Example 7 Solve
0
)
3
(sin
)
1
2
cos
2
( 2
2
2





 dy
x
x
dx
xy
x
xy
Solution. Here,
Bas 111 Mathematics 3 Dr. Fatma Zedan El Emam
123
.
2
cos
2
,
2
cos
2
3
sin
,
1
2
cos
2
2
2
2
2
2
x
N
y
M
x
x
x
x
N
x
x
x
y
M
x
x
N
xy
x
xy
M




















So the given differential equation is exact
differential equation and the solution is
C
dy
x
N
dx
M
y

 
 )
containing
not
of
terms
(
const.
as
C
dy
dx
xy
x
xy
y




 
 3
)
1
2
cos
2
( 2
const.
as
.
3
sin
3
1
2
cos
2
2
2
2
C
y
x
yx
x
y
C
dy
y
dx
dx
x
y
dx
x
x
y









 
 

Example 8 Solve
0
1
)
1
( /
/










dx
dy
y
x
e
e y
x
y
x
Solution. We have,
Bas 111 Mathematics 3 Dr. Fatma Zedan El Emam
124
.
1
1
,
,
1
0
1
0
1
1
2
2
2
x
N
y
M
e
y
x
e
y
x
e
y
e
y
x
N
e
y
x
y
M
y
x
e
e
N
e
M
dy
y
x
e
e
dx
e
dx
dy
y
x
e
e
y
x
y
x
y
x
y
x
y
x
y
x
y
x
y
x
y
x
y
x
y
x
y
x
y
x





























































Then the given equation is exact and its solution is
C
dy
x
N
dx
M
y

 
 )
containing
not
of
terms
(
const.
as
.
0
1
C
ye
x
C
dy
dx
e
y
x
y
y
x














 

const.
as
Bas 111 Mathematics 3 Dr. Fatma Zedan El Emam
125
Equations Reducible to Exact - Integrating Factor
Sometimes a differential equation which is not
exact may become so, on multiplication by a
suitable function known as the integrating factor
I.F.
Rule 1. If N
N
M x
y /
)
(  is a function of x alone,
say f (x), then


dx
x
f
e
)
(
I.F.
Rule 2. If M
M
N y
x /
)
(  is a function of y alone,
say )
(y
f , then


dy
y
f
e
)
(
I.F.
Example 9 Solve
0
2
)
ln
2
( 

 dx
y
dy
xy
x
x (1)
Solution.
xy
x
x
N
y
M
dy
xy
x
x
dx
y






ln
2
,
2
0
)
ln
2
(
2
Bas 111 Mathematics 3 Dr. Fatma Zedan El Emam
126
.
)
ln
1
(
2
,
2 y
x
x
N
y
M








So, equation (1) not exact. Here,
).
(
1
)
ln
2
(
)
ln
2
(
ln
2
ln
2
2
2
x
f
x
y
x
x
y
x
xy
x
x
y
x
N
x
N
y
M

















x
x
e
e
e
e x
x
dx
x
dx
x
f 1
I.F. 1
ln
ln
1
)
( 1





 

 


On multiplying equation (1) by
x
1
, we get the exact
DE.
.
2
1
ln
2
2
0
)
ln
2
(
2
2
C
y
x
y
C
dy
y
dx
x
y
dy
y
x
dx
x
y
y












const.
as
Bas 111 Mathematics 3 Dr. Fatma Zedan El Emam
127
Example 10 Solve
0
)
4
2
(
)
2
( 4
3
4




 dy
x
y
xy
dx
y
y (2)
Solution. Here,
.
4
,
2
4
4
2
,
2
3
3
4
3
4













y
x
N
y
y
M
x
y
xy
N
y
y
M
3
3
ln
ln
3
3
)
(
3
3
4
3
3
1
I.F.
)
(
3
)
2
(
)
2
(
3
2
)
2
4
(
)
4
(
3
y
y
e
e
e
e
y
f
y
y
y
y
y
y
y
y
M
y
M
x
N
x
x
dx
y
dx
y
f

























 


On multiplying equation (2) by
3
/
1 y , we get the
exact DE. 0
)
4
2
(
)
2
( 3
2




 dy
y
x
y
x
dx
y
y
C
y
y
y
x
C
dy
y
dx
y
y
y







 

2
2
2
)
2
(
2
)
2
(
const.
as
Bas 111 Mathematics 3 Dr. Fatma Zedan El Emam
128
Exercises 7
1-4 prove that y is a solution of the indicated
differential equation.
Solve the differential equations
Find the particular solution of the IVP.
Solve the differential equations
Bas 111 Mathematics 3 Dr. Fatma Zedan El Emam
129
Find the particular solution of the IVP.
Solve the differential equations
Bas 111 Mathematics 3 Dr. Fatma Zedan El Emam
130
3.2.3 Homogeneous Differential Equation
Definition 3.2.3
A homogeneous DE is a first order DE that can be
written in the form
)
(
x
y
f
dx
dy

To solve the homogeneous DE, we put
x
y
v  (or
vx
y  ) and
dx
dv
x
v
dx
dy

 . The reduced
differential equation involves v and x only and it
can be solved by variables separable method.
Another Definition
A DE of the form
)
,
(
)
,
(
y
x
y
x
f
dx
dy


is called a homogeneous equation if each term of f
(x, y) and (x, y) is of the same degree.
Bas 111 Mathematics 3 Dr. Fatma Zedan El Emam
131
Example 11 Solve the following differential
equation
xy
y
y
x
xy 2
3
)
2
( 2
2




Solution. We have,
2
2
2
2
2
2
3
2
3
)
2
(
x
xy
xy
y
dx
dy
xy
y
dx
dy
x
xy






 .
Divide numerator and denominator on the right by
2
x to obtain
1
2
2
)
(
3 2



x
y
x
y
x
y
dx
dy
.
Put
x
y
v  vx
y 
 and
dx
dv
x
v
dx
dy

 .
On substituting, the given equation becomes
1
2
2
2
3
1
2
2
3
2
2
2










v
v
v
v
v
dx
dv
x
v
v
v
dx
dv
x
v
Bas 111 Mathematics 3 Dr. Fatma Zedan El Emam
132
x
dx
dv
v
v
v
v
v
v
dx
dv
x 












 2
2
1
2
1
2
C
x
v
v
x
dx
dv
v
v
v
ln
ln
)
(
ln
1
2
2
2













 

Cx
v
v 

 2
Replacing v with
x
y
we get,
3
2
2
2
Cx
xy
y
Cx
x
y
x
y





Example 12 Solve the differential equation
.
0
cot
cot 







 dx
y
x
y
dy
y
x
x
y
Solution. We have,
y
x
y
y
y
x
x
dy
dx
y
y
x
x
dy
dx
y
x
y
cot
cot
cot
)
cot
(




 .
Bas 111 Mathematics 3 Dr. Fatma Zedan El Emam
133
Divide numerator and denominator on the right by
y to obtain
y
x
y
x
y
x
dy
dx
cot
1
cot 
 .
Put
y
x
v  vy
x 
 and
dy
dv
y
v
dy
dx

 .
On substituting, the given equation becomes
v
v
v
v
v
dy
dv
y
v
v
v
dy
dv
y
v
cot
cot
1
cot
cot
1
cot







y
dy
vdv
v
dy
dv
y 




 cot
cot
1
C
y
v
y
dy
dv
v ln
ln
sin
ln
cot 





 

C
v
y 
 sin
Replacing v with
y
x
we get, .
sin C
y
x
y 
Bas 111 Mathematics 3 Dr. Fatma Zedan El Emam
134
Example 13 Solve the differential equation
y
x
y
dx
dy
x 

2
Solution. Rewrite the equation as
x
y
x
y
dx
dy








2
Put
x
y
v  vx
y 
 and
dx
dv
x
v
dx
dy

 .
x
dx
v
dv
v
dx
dv
x
v
v
dx
dv
x
v 






 2
2
2
C
x
v
C
x
v
x
dx
v
dv









 

ln
1
ln
1
2
Replacing v with
x
y
we get,
C
x
x
y



ln
Bas 111 Mathematics 3 Dr. Fatma Zedan El Emam
135
3.2.4 Linear Differential Equations
Definition 3.2.3
A first-order linear DE is an equation of the form
)
(
)
( x
Q
y
x
P
dx
dy


where P and Q are continuous functions.
The solution to the first-order linear DE
)
(
)
( x
Q
y
x
P
dx
dy


is
 
 

 c
dx
x
Q
x
x
y )
(
)
(
)
(
1


where,


dx
x
P
e
x
)
(
)
(

Note. 

dx
x
P
e
x
)
(
)
(
 is called the integrating
factor.
Bas 111 Mathematics 3 Dr. Fatma Zedan El Emam
136
Example 14 Solve the differential equation
2
)
1
(
)
1
( 


 x
e
y
dx
dy
x x
Solution. Rewrite the equation as
)
1
(
1
1



 x
e
y
x
dx
dy x
.
Here,
1
1
)
(



x
x
P and )
1
(
)
( 
 x
e
x
Q x
.
1
1
)
(
1
)
1
(
ln
)
1
(
ln
1
1
)
(
x
e
e
e
e
x
x
x
dx
x
dx
x
P














Then the solution is
 
   
C
e
x
C
dx
e
x
C
dx
x
e
x
x
C
dx
x
Q
x
x
y
x
x
x
























)
1
(
)
1
(
)
1
(
1
1
)
1
(
)
(
)
(
)
(
1


Example 15 Solve the differential equation
0
3
5 5
2



 x
xy
y
x
Bas 111 Mathematics 3 Dr. Fatma Zedan El Emam
137
Solution. Rewrite the equation as
3
3
5
x
y
x
dx
dy


 .
Here,
x
x
P
5
)
(  and
3
3
)
( x
x
Q 

.
)
( 5
5
ln
ln
5
5
)
(
x
e
e
e
e
x x
x
dx
x
dx
x
P








Then the solution is
 
   






















C
x
x
C
dx
x
x
C
dx
x
x
x
C
dx
x
Q
x
x
y
3
1
3
1
)
3
(
1
)
(
)
(
)
(
1
9
5
8
5
3
5
5


• Equations Reducible To The Linear Form
(Bernoulli Equation)
A Bernoulli differential equation has the form
n
y
x
Q
y
x
P
dx
dy
)
(
)
( 
 (1)
Bas 111 Mathematics 3 Dr. Fatma Zedan El Emam
138
where n is a real number. The Bernoulli DE can be
reduced to the linear form on dividing by
n
y and
substituting z
yn

1
1
.
On dividing both sides of (1) by
n
y , we get
)
(
1
)
(
1
1
x
Q
y
x
P
dx
dy
y n
n

  (2)
Put z
yn

1
1
, so that
dx
dz
n
dx
dy
y
dx
dz
dx
dy
y
n
n
n
)
1
(
1
1
)
1
(





.
Then (2) becomes )
(
)
(
)
1
(
1
x
Q
z
x
P
dx
dz
n



or
)
(
)
1
(
)
(
)
1
( x
Q
n
z
x
P
n
dx
dz




which is a linear equation in z and can be solved
easily by the previous method.
Note; Equation (1) is linear if n =0 and separable if
n =1.
Bas 111 Mathematics 3 Dr. Fatma Zedan El Emam
139
Example 16 Solve the differential equation
0
)
(
2


 dx
y
x
y
dy
x
Solution. Rewrite the equation as
2
2
1
1
y
x
y
x
dx
dy


 (Bernoulli DE, n =2)
2
2
1
1
1
1
2
x
y
x
dx
dy
y
y





Put z
y

1
, so that
dx
dz
dx
dy
y
dx
dz
dx
dy
y




 2
2
1
1
The given equation reduces to a linear differential
equation in z.
2
1
1
x
z
x
dx
dz

 .
Here,
x
x
P
1
)
( 
 and 2
1
)
(
x
x
Q 
.
1
)
(
1
ln
ln
1
)
(
x
e
e
e
e
x x
x
dx
x
dx
x
P










Bas 111 Mathematics 3 Dr. Fatma Zedan El Emam
140
Then the solution is
 
 
)
2
1
(
1
2
1
1
)
(
)
(
)
(
1
2
2
3
2
C
x
x
y
C
x
x
C
dx
x
x
C
dx
x
x
x
C
dx
x
Q
x
x
z

































Example 17 Solve the differential equation
3
6
1
2 y
x
y
x
y 

 
Solution. Rewrite the equation as
3
6
2
y
x
y
x
dx
dy

 (Bernoulli DE, n =3)
6
2
3
1
2
1
3
x
y
x
dx
dy
y
y




Put z
y

2
1
, so that
Bas 111 Mathematics 3 Dr. Fatma Zedan El Emam
141
dx
dz
dx
dy
y
dx
dz
dx
dy
y 2
1
1
1
2 2
3





The given equation reduces to a linear differential
equation in z.
6
)
2
(
6
2
4
2
2
1
x
z
x
dx
dz
x
z
x
dx
dz









.
Here,
x
x
P
4
)
( 
 and
6
2
)
( x
x
Q 

.
1
)
( 4
4
ln
ln
4
1
4
)
(
x
e
e
e
e
x x
x
dx
x
dx
x
P







 


Then the solution is
 
  



























C
x
x
C
dx
x
x
C
dx
x
x
x
C
dx
x
Q
x
x
z
3
2
2
)
2
(
1
)
(
)
(
)
(
1
3
4
2
4
6
4
4


)
3
2
(
1 3
4
2
C
x
x
y




Bas 111 Mathematics 3 Dr. Fatma Zedan El Emam
142
or 1
)
3
2
( 2
4
7


 y
Cx
x
Example 18 Solve the following IVP
0



 y
x
y
y , y(1) = 0
Solution. Rewrite the equation as
2
1
1
y
y
x
dx
dy

 (Bernoulli DE,
2
1

n )
1
1 2
1
2
1
2
1





y
x
dx
dy
y
y
. Put z
y 
2
1
, so that
dx
dz
dx
dy
y
dx
dz
dx
dy
y 2
2
1 2
1
2
1





The given equation reduces to a linear differential
equation in z.
2
1
2
1
1
1
2
2






z
x
dx
dz
z
x
dx
dz
.
Here,
x
x
P
2
1
)
(  and
2
1
)
( 
x
Q
Bas 111 Mathematics 3 Dr. Fatma Zedan El Emam
143
.
)
( 2
1
2
1
ln
1
2
1
)
(
x
e
e
e
x x
dx
x
dx
x
P







Then the solution is
 
2
1
2
1
2
3
2
1
2
1
2
1
3
1
3
1
3
2
1
)
(
)
(
)
(
1
2
1































Cx
x
y
Cx
x
C
x
x
C
dx
x
x
C
dx
x
Q
x
x
z 

Applying the initial condition and solving for C
gives,
3
1
3
1
0 



 C
C .
Thus a solution of the IVP is
)
(
3
1 2
1
2
1


 x
x
y or
2
)
1
(
9
1

 x
x
x
y
Bas 111 Mathematics 3 Dr. Fatma Zedan El Emam
144
Exercises 8
1-5 Solve the following differential equations
6-7 Solve the following differential equations by
using the Substitution x = vy.
8 Show that the following differential equation is
both homogeneous and exact and solve it.
9-12 Solve the following differential equations
Bas 111 Mathematics 3 Dr. Fatma Zedan El Emam
145
13 The value of  so that
2
y
e
is an integrating
factor of the differential equation
In Problems 15–20 solve the given differential
equation by using an appropriate substitution.
In Problems 21 and 22 solve the given initial-value
problem.
Bas 111 Mathematics 3 Dr. Fatma Zedan El Emam
146
CHAPTER 4
Second-Order Linear Differential Equation
4.1. Basic concepts
Definition 4.1.1
The second-order linear differential equation with
constant coefficients has the form
)
(
2
2
x
f
cy
dx
dy
b
dx
y
d
a 

 . (1)
If 0
)
( 
x
f for all x, the equation is said to be
homogeneous.
Notice that the meaning of the word
homogeneous is different from that in Section
3.2.3. If 0
)
( 
x
f for some x, the equation is said
to be nonhomogeneous. Thus the form of a
second-order linear homogeneous differential
equation is
0





 cy
y
b
y
a (2)
Bas 111 Mathematics 3 Dr. Fatma Zedan El Emam
147
Theorem 4.3.1.
If 1
y and 2
y are both solutions of equation (2),
then the linear combination
2
2
1
1 y
c
y
c
y 

where 1
c and 2
c are arbitrary constants, is also a
solution.
• Linear Dependence And Linear Independence
Of Solutions
There is a simple test to determine whether two
solutions of equation (2) are linearly independent
or dependent on an open interval I . Define the
Wronskian )
,
( 2
1 y
y
W of two solutions 1
y and 2
y
to be the 2×2 determinant
.
)
,
( 1
2
2
1
2
2
1
1
2
1 y
y
y
y
y
y
y
y
y
y
W 






Bas 111 Mathematics 3 Dr. Fatma Zedan El Emam
148
Often we denote this Wronskian as just W(x). The
solutions 1
y and 2
y are linearly independent on I
if and only if 0
)
( 
x
W on I.
Example 1 Determine whether the functions
x
e2
and
x
xe2
are linearly independent
Solution.
0
2
2
2
2
)
(
4
4
4
4
2
2
2
2
2







x
x
x
x
x
x
x
x
x
e
xe
xe
e
xe
e
xe
e
e
x
W
,
so
x
e2
and
x
xe2
are linearly independent .
Theorem 4.3.2.
If 1
y and 2
y are linearly independent solutions of
equation (2) on an interval, and a is never 0, then
the general solution is given by
2
2
1
1 y
c
y
c
y 

where 1
c and 2
c are arbitrary constants.
Bas 111 Mathematics 3 Dr. Fatma Zedan El Emam
149
• The general solution of Eq. (1) has the form
p
c y
y
y 

where the complementary function )
(x
yc is
general solution of associated homogeneous Eq.
(2) and )
(x
yp is a particular solution of Eq. (1)
i.e. p
y
y
c
y
c
y 

 2
2
1
1
• Differential Operator
Symbol D stands for the operation of differential
i.e.,
2
2
2
,
dx
y
d
y
D
dx
dy
Dy 
 .
For example
x
x
x
x
x
x
e
e
dx
d
e
D
e
e
dx
d
e
D 3
3
2
2
3
2
3
3
3
9
,
3 


 ,
Bas 111 Mathematics 3 Dr. Fatma Zedan El Emam
150
D
1
stands for the operation of integration and
2
1
D
stands for the operation of integration twice.
Some properties of operator D is
)
(
)
(
)]
(
)
(
[
)
1 2
1
2
1 x
f
D
x
f
D
x
f
x
f
D 


)
(
)]
(
[
)
2 x
Df
k
x
f
k
D 
4.2 Method For Finding The Complementary
Function (C.F.) c
y
Steps for solving
1) Starting from: 0





 cy
y
b
y
a
2) Rewrite this Eq. as: 0
)
( 2


 y
c
bD
aD
3) Write the auxiliary equation A.E. (characteristic
equation): 0
2


 c
b
a 

4) Solve the auxiliary equation:
Bas 111 Mathematics 3 Dr. Fatma Zedan El Emam
151
a
ac
b
b
2
4
,
2
2
1






Then form the solution according to the following
possibilities:
Solutions to the
A.E.
general solution
Two real distinct
roots 2
1,

Real repeated roots


 
 2
1
Complex roots

 i

x
x
e
C
e
C
y 2
1
2
1




x
x
x
e
x
C
C
xe
C
e
C
y 


)
( 2
1
2
1 



 
x
C
x
C
e
y x



sin
cos 2
1 

Finally, solve for constants if you have been given
initial conditions.
Example 2 Solve
Bas 111 Mathematics 3 Dr. Fatma Zedan El Emam
152
.
0
15
8
2
2


 y
dx
dy
dx
y
d
Solution. Given equation can be written as
0
)
15
8
( 2


 y
D
D
Here auxiliary equation is
5
,
3
0
)
5
)(
3
(
0
15
8
2
1
2
















Hence, the required solution is
x
x
e
C
e
C
y 5
2
3
1 

Example 3 Solve
.
0
3 2
2


 y
dx
dy
dx
y
d
Solution. Given equation can be written as
0
)
1
3
( 2


 y
D
D
Here auxiliary equation is
0
1
3 2


 

6
13
1
, 2
1



 

The general solution is
Bas 111 Mathematics 3 Dr. Fatma Zedan El Emam
153
x
x
e
C
e
C
y 6
)
13
1
(
2
6
)
13
1
(
1






Example 4 Solve
.
0
9
6
2
2


 y
dx
dy
dx
y
d
Solution. Given equation can be written as
0
)
9
6
( 2


 y
D
D
A.E. is 0
9
6
2


 

3
0
)
3
( 2
1
2





 


Then, the general solution is
x
x
xe
C
e
C
y 3
2
3
1 

Example 5 Solve the equation
.
0
9
12
4 




 y
y
y
Solution. Given equation can be written as
0
)
9
12
4
( 2


 y
D
D
A.E. is 0
9
12
4 2


 
 0
)
3
2
( 2


 
Bas 111 Mathematics 3 Dr. Fatma Zedan El Emam
154
2
3
2
1 


 

Then, the general solution is
x
x
xe
C
e
C
y 2
3
2
2
3
1




Example 6 Solve
.
0
5
4
2
2


 y
dx
dy
dx
y
d
Solution. Here the auxiliary equation is
0
5
4
2


 

i
i










 2
2
2
4
2
20
16
4
, 2
1 

Then, the general solution is
)
sin
cos
( 2
1
2
x
C
x
C
e
y x

 
Example 7 Solve the initial-value problem
0



 y
y 2
)
0
( 
y 3
)
0
( 

y
Bas 111 Mathematics 3 Dr. Fatma Zedan El Emam
155
Solution. The auxiliary equation is 

 0
1
2

1
2


 i


 2
1,
 . Thus ,
1
,
0 
 
 and
since ,
1
0

x
e the general solution is
x
C
x
C
x
y sin
cos
)
( 2
1 

x
C
x
C
x
y cos
sin
)
( 2
1 



The initial conditions become
3
)
0
(
,
2
)
0
( 2
1 



 C
y
C
y
Therefore the solution of the initial-value problem
is
x
x
x
y sin
3
cos
2
)
( 

Bas 111 Mathematics 3 Dr. Fatma Zedan El Emam
156
Exercises 9
1-4 Determine whether the given set of functions
are linearly independent on the interval )
,
( 
 .
1.
x
e
x
f 
)
(
1 and
x
e
x
f 

)
(
2
2. x
x
f 3
sin
)
(
1  and x
x
f 3
cos
)
(
2 
3.
x
e
x
f 

)
(
1 and
x
e
x
f 
 5
)
(
2
4.
3
1 )
( x
x
f  and
4
2 )
( x
x
f 
5-12 Find the general solution of the given second-
order differential equation.
Bas 111 Mathematics 3 Dr. Fatma Zedan El Emam
157
4.3. Methods for Finding The Particular Solution
p
y
4.3.1 Inverse Differential Operator Method
BY using the differential operator D, we can write
the equation
)
(
2
2
x
f
cy
dx
dy
b
dx
y
d
a 


in the form
)
(
)
( 2
x
f
y
c
bD
aD 


or more simply,
)
(
)
( x
f
y
D 
 where c
bD
aD
D 


 2
)
( .
So,
)
(
)
(
1
x
f
D
yp

 .
• Rules To Find p
y
1) If m
x
f 
)
( (m is constant)
a) 0
)
0
(
,
)
0
(
1
)
(
1







c
m
m
m
D
yp
Bas 111 Mathematics 3 Dr. Fatma Zedan El Emam
158
0
)
0
(
,
1
)
(
1
1
)
(
1
b) 2










x
b
m
b
m
D
m
b
aD
D
m
bD
aD
m
D
yp
Example 8 Find the particular solution of the
following DE
2
8
2 




 y
y
y
Solution. Given equation can be written as
2
)
8
2
( 2


 y
D
D
    .
4
1
8
2
2
8
0
0
1
2
8
2
1
2









D
D
yp
Example 9 Find the particular solution of the
following D.E.
2
2 



 y
y
Solution. Given equation can be written as
2
)
2
( 2

 y
D
D
Bas 111 Mathematics 3 Dr. Fatma Zedan El Emam
159
   
  .
)
1
(
1
2
2
0
1
1
2
2
1
1
2
2
1
2
 












x
dx
D
D
D
D
D
D
yp
2) If )
(x
f is a polynomial of degree n and )
(D

take any form of ,
,
)
1
(
),
1
(
),
1
( 2

D
D
D 


then
)
(
]
)
1
(
1
[
)
(
]
1
[
)
(
1
1
2
1
x
f
D
D
D
x
f
D
x
f
D
n
n











)
(
]
1
[
)
(
]
1
[
)
(
1
1
2
1
x
f
D
D
D
x
f
D
x
f
D
n










)
(
]
)
1
(
3
2
1
[
)
(
]
1
[
)
(
)
1
(
1
2
2
2
x
f
D
n
D
D
x
f
D
x
f
D
n











Here we expand
1
)]
(
[ 
 D by the Binomial
theorem in ascending powers of D as far as the
result of operation on
n
x is zero.
Bas 111 Mathematics 3 Dr. Fatma Zedan El Emam
160
Example 10 Solve the following DE
2
x
y
y 




Solution. Given equation can be written as
2
2
)
( x
y
D
D 

A.E. is 0
)
1
(
0
2




 



,
1
,
0
, 2
1
2
1
x
c e
C
C
y 





 

.
2
3
)
2
2
(
1
)
1
(
1
)
1
(
1
)
1
(
1
1
1
2
3
2
2
2
2
1
2
2
2
x
x
x
x
x
D
x
D
D
D
x
D
D
x
D
D
x
D
D
yp














 
Then the general solution is
x
x
x
e
C
C
y
y
y x
p
c 2
3
2
3
2
1 





 
Example 11 Find the particular solution of the
following DE
2
3 x
y
y
y 





Bas 111 Mathematics 3 Dr. Fatma Zedan El Emam
161
Solution. Given equation can be written as
2
2
)
1
3
( x
y
D
D 


2
1
2
2
2
)]
3
(
1
[
1
3
1
x
D
D
x
D
D
yp








2
2
2
2
2
]
8
3
1
[
]
9
)
3
(
1
[ x
D
D
x
D
D
D 






16
6
2


 x
x
3) Exponential shift: If )
(
)
( x
v
e
x
f mx
 , then
)
(
)
(
1
)
(
)
(
1
x
v
m
D
e
x
v
e
D
mx
mx




Example 12 Solve the DE
x
e
x
y
y
y 2
3
4
4 





Solution. Given equation can be written as
x
e
x
y
D
D 2
3
2
)
4
4
( 


A.E. is 0
)
2
(
0
4
4 2
2





 


,
2 2
2
2
1
2
1
x
x
c xe
C
e
C
y 




 

Bas 111 Mathematics 3 Dr. Fatma Zedan El Emam
162
   
x
x
p e
x
D
e
x
D
D
y 2
3
2
2
3
2
)
2
(
1
4
4
1





3
2
2
3
2
2 1
]
2
)
2
[(
1
x
D
e
x
D
e x
x




20
4
1 5
2
4
2 x
e
x
D
e x
x


Then the general solution is
5
2
2
2
2
1
20
1
x
e
xe
C
e
C
y
y
y x
x
x
p
c 




Example 13 Solve the DE
3
3
9
6
x
e
y
y
y
x







Solution. Given equation can be written as
3
3
2
)
9
6
(
x
e
y
D
D
x




A.E. is 0
)
3
(
0
9
6 2
2





 


,
3 3
2
3
1
2
1
x
x
c xe
C
e
C
y 







 

Bas 111 Mathematics 3 Dr. Fatma Zedan El Emam
163



















3
3
2
3
3
2
)
3
(
1
9
6
1
x
e
D
x
e
D
D
y
x
x
p
 
3
2
3
3
2
3 1
1
]
3
)
3
[(
1 





 x
D
e
x
D
e x
x
x
e
x
e
x
D
e
x
x
x
2
)
1
)(
2
(
2
1 3
1
3
2
3

















Then the general solution is
x
e
xe
C
e
C
y
y
y
x
x
x
p
c
2
3
3
2
3
1








Example 14 Solve the IVP.
4
1
)
0
(
,
4
11
)
0
(
,
6
8
3
4 









 y
y
xe
y
y
y x
Solution. Given equation can be written as
6
8
)
3
4
( 2



 x
xe
y
D
D
A.E. is 0
)
3
)(
1
(
0
3
4
2






 



,
3
,
1
, 3
2
1
2
1
x
x
c e
C
e
C
y 







 

 
6
8
3
4
1
2



 x
p xe
D
D
y
Bas 111 Mathematics 3 Dr. Fatma Zedan El Emam
164
    2
1
2
2
6
3
4
1
8
3
4
1
I
I
D
D
xe
D
D
x








 
 
 
 
x
D
D
e
x
D
D
e
x
D
D
e
xe
D
D
I
x
x
x
x
)
4
1
)(
2
1
)(
4
)(
2
(
1
8
)
4
)(
2
(
1
8
]
3
)
1
][(
1
)
1
[(
1
8
8
)
3
)(
1
(
1
1














   
x
D
D
e
x
D
D
e x
x
































4
1
2
1
4
1
2
1
1
1
)
2
1
4
1
(
4
1
2
1 
















 x
e
x
D
e x
x
)
3
4
(
4
1
)
4
3
( 


 x
e
x
e x
x
,
    2
6
3
0
0
1
6
3
4
1
2
2 






D
D
I .
Bas 111 Mathematics 3 Dr. Fatma Zedan El Emam
165
Then 2
)
3
4
(
4
1
2
1 



 x
e
I
I
y x
p
and the general solution is
2
)
3
4
(
4
1
3
2
1 





 

x
e
e
C
e
C
y
y
y x
x
x
p
c ,
)
3
4
(
4
1
)
4
(
4
1
3 3
2
1 





 

x
e
e
e
C
e
C
y x
x
x
x
)
1
4
(
4
1
3 3
2
1 



 

x
e
e
C
e
C x
x
x
,
2
)
3
(
4
1
4
11
4
11
)
0
( 2
1 







 C
C
y

0
2
1 

 C
C (1)
4
1
3
4
1
4
1
)
0
( 2
1 





 C
C
y

0
3 2
1 

 C
C (2)
From (1) and (2) we get 0
,
0 2
1 
 C
C , then
the required solution is
2
)
3
4
(
4
1


 x
e
y x
Bas 111 Mathematics 3 Dr. Fatma Zedan El Emam
166
4) If
mx
e
x
f 
)
( , then
a) 0
)
(
,
)
(
1
)
(
1





 m
e
m
e
D
y mx
mx
p
b) 0
)
(
,
)
(
1
)
(
1






 m
e
m
x
e
D
y mx
mx
p and
0
)
( 
 m .
If 

 0
)
(m
mx
mx
p e
m
x
e
D
y
)
(
1
.
)
(
1 2
 




Example 15 Solve the DE
x
e
y
y
y 4
42
3
2 





Solution. Given equation can be written as
x
e
y
D
D 4
2
42
)
3
2
( 


A.E. is 0
)
3
)(
1
(
0
3
2
2






 



,
3
,
1
, 3
2
1
2
1
x
x
c e
C
e
C
y 





 

 
x
p e
D
D
y 4
2
42
3
2
1



Bas 111 Mathematics 3 Dr. Fatma Zedan El Emam
167
x
x
x
x
e
e
e
e
D
D
4
4
4
4
2
)
7
)(
3
(
1
42
)
3
4
)(
1
4
(
1
42
)
3
)(
1
(
1
42








Then the general solution is
x
x
x
p
c e
e
C
e
C
y
y
y 4
3
2
1 2




 
Example 16 Find the particular solution of the
following DE
x
e
y
y 





Solution.
x
p
x
e
D
D
y
e
y
D
D 





 2
2 1
)
(
We note that 0
)
(
1
2




D
D
D , then
x
x
x
D
x
D
x
p
xe
e
x
e
D
x
e
D
D
x
e
D
D
y





















)
1
2
(
1
)
1
2
(
1
)
(
1
1
1
1
2
2
Mathematics 3.pdf civil engineering concrete
Mathematics 3.pdf civil engineering concrete
Mathematics 3.pdf civil engineering concrete
Mathematics 3.pdf civil engineering concrete
Mathematics 3.pdf civil engineering concrete
Mathematics 3.pdf civil engineering concrete
Mathematics 3.pdf civil engineering concrete
Mathematics 3.pdf civil engineering concrete
Mathematics 3.pdf civil engineering concrete
Mathematics 3.pdf civil engineering concrete
Mathematics 3.pdf civil engineering concrete
Mathematics 3.pdf civil engineering concrete
Mathematics 3.pdf civil engineering concrete
Mathematics 3.pdf civil engineering concrete
Mathematics 3.pdf civil engineering concrete
Mathematics 3.pdf civil engineering concrete
Mathematics 3.pdf civil engineering concrete
Mathematics 3.pdf civil engineering concrete
Mathematics 3.pdf civil engineering concrete
Mathematics 3.pdf civil engineering concrete
Mathematics 3.pdf civil engineering concrete
Mathematics 3.pdf civil engineering concrete
Mathematics 3.pdf civil engineering concrete
Mathematics 3.pdf civil engineering concrete
Mathematics 3.pdf civil engineering concrete
Mathematics 3.pdf civil engineering concrete
Mathematics 3.pdf civil engineering concrete
Mathematics 3.pdf civil engineering concrete
Mathematics 3.pdf civil engineering concrete
Mathematics 3.pdf civil engineering concrete
Mathematics 3.pdf civil engineering concrete
Mathematics 3.pdf civil engineering concrete
Mathematics 3.pdf civil engineering concrete
Mathematics 3.pdf civil engineering concrete
Mathematics 3.pdf civil engineering concrete
Mathematics 3.pdf civil engineering concrete
Mathematics 3.pdf civil engineering concrete
Mathematics 3.pdf civil engineering concrete
Mathematics 3.pdf civil engineering concrete
Mathematics 3.pdf civil engineering concrete
Mathematics 3.pdf civil engineering concrete
Mathematics 3.pdf civil engineering concrete
Mathematics 3.pdf civil engineering concrete
Mathematics 3.pdf civil engineering concrete
Mathematics 3.pdf civil engineering concrete
Mathematics 3.pdf civil engineering concrete
Mathematics 3.pdf civil engineering concrete
Mathematics 3.pdf civil engineering concrete
Mathematics 3.pdf civil engineering concrete
Mathematics 3.pdf civil engineering concrete
Mathematics 3.pdf civil engineering concrete
Mathematics 3.pdf civil engineering concrete
Mathematics 3.pdf civil engineering concrete
Mathematics 3.pdf civil engineering concrete
Mathematics 3.pdf civil engineering concrete
Mathematics 3.pdf civil engineering concrete
Mathematics 3.pdf civil engineering concrete
Mathematics 3.pdf civil engineering concrete
Mathematics 3.pdf civil engineering concrete
Mathematics 3.pdf civil engineering concrete
Mathematics 3.pdf civil engineering concrete
Mathematics 3.pdf civil engineering concrete
Mathematics 3.pdf civil engineering concrete
Mathematics 3.pdf civil engineering concrete
Mathematics 3.pdf civil engineering concrete
Mathematics 3.pdf civil engineering concrete
Mathematics 3.pdf civil engineering concrete
Mathematics 3.pdf civil engineering concrete
Mathematics 3.pdf civil engineering concrete
Mathematics 3.pdf civil engineering concrete

More Related Content

Similar to Mathematics 3.pdf civil engineering concrete

OrthogonalFunctionsPaper
OrthogonalFunctionsPaperOrthogonalFunctionsPaper
OrthogonalFunctionsPaperTyler Otto
 
Fourier 3
Fourier 3Fourier 3
Fourier 3nugon
 
Numarical values
Numarical valuesNumarical values
Numarical valuesAmanSaeed11
 
Numarical values highlighted
Numarical values highlightedNumarical values highlighted
Numarical values highlightedAmanSaeed11
 
Limits and Continuity of Functions
Limits and Continuity of Functions Limits and Continuity of Functions
Limits and Continuity of Functions OlooPundit
 
Convexity in the Theory of the Gamma Function.pdf
Convexity in the Theory of the Gamma Function.pdfConvexity in the Theory of the Gamma Function.pdf
Convexity in the Theory of the Gamma Function.pdfPeterEsnayderBarranz
 
fourierseries-121010133546-phpapp02.pdf
fourierseries-121010133546-phpapp02.pdffourierseries-121010133546-phpapp02.pdf
fourierseries-121010133546-phpapp02.pdfdiehardgamer1
 
Transformation of random variables
Transformation of random variablesTransformation of random variables
Transformation of random variablesTarun Gehlot
 
Digital text book
Digital text bookDigital text book
Digital text bookshareenapr5
 
Coincidence points for mappings under generalized contraction
Coincidence points for mappings under generalized contractionCoincidence points for mappings under generalized contraction
Coincidence points for mappings under generalized contractionAlexander Decker
 

Similar to Mathematics 3.pdf civil engineering concrete (17)

OrthogonalFunctionsPaper
OrthogonalFunctionsPaperOrthogonalFunctionsPaper
OrthogonalFunctionsPaper
 
Sol75
Sol75Sol75
Sol75
 
Sol75
Sol75Sol75
Sol75
 
Fourier 3
Fourier 3Fourier 3
Fourier 3
 
Program on Quasi-Monte Carlo and High-Dimensional Sampling Methods for Applie...
Program on Quasi-Monte Carlo and High-Dimensional Sampling Methods for Applie...Program on Quasi-Monte Carlo and High-Dimensional Sampling Methods for Applie...
Program on Quasi-Monte Carlo and High-Dimensional Sampling Methods for Applie...
 
Numarical values
Numarical valuesNumarical values
Numarical values
 
Numarical values highlighted
Numarical values highlightedNumarical values highlighted
Numarical values highlighted
 
Limits and Continuity of Functions
Limits and Continuity of Functions Limits and Continuity of Functions
Limits and Continuity of Functions
 
Jacobians new
Jacobians newJacobians new
Jacobians new
 
Convexity in the Theory of the Gamma Function.pdf
Convexity in the Theory of the Gamma Function.pdfConvexity in the Theory of the Gamma Function.pdf
Convexity in the Theory of the Gamma Function.pdf
 
Double integration
Double integrationDouble integration
Double integration
 
fourierseries-121010133546-phpapp02.pdf
fourierseries-121010133546-phpapp02.pdffourierseries-121010133546-phpapp02.pdf
fourierseries-121010133546-phpapp02.pdf
 
Transformation of random variables
Transformation of random variablesTransformation of random variables
Transformation of random variables
 
Shareena p r
Shareena p r Shareena p r
Shareena p r
 
Shareena p r
Shareena p r Shareena p r
Shareena p r
 
Digital text book
Digital text bookDigital text book
Digital text book
 
Coincidence points for mappings under generalized contraction
Coincidence points for mappings under generalized contractionCoincidence points for mappings under generalized contraction
Coincidence points for mappings under generalized contraction
 

Recently uploaded

UNIT III ANALOG ELECTRONICS (BASIC ELECTRONICS)
UNIT III ANALOG ELECTRONICS (BASIC ELECTRONICS)UNIT III ANALOG ELECTRONICS (BASIC ELECTRONICS)
UNIT III ANALOG ELECTRONICS (BASIC ELECTRONICS)Dr SOUNDIRARAJ N
 
Gfe Mayur Vihar Call Girls Service WhatsApp -> 9999965857 Available 24x7 ^ De...
Gfe Mayur Vihar Call Girls Service WhatsApp -> 9999965857 Available 24x7 ^ De...Gfe Mayur Vihar Call Girls Service WhatsApp -> 9999965857 Available 24x7 ^ De...
Gfe Mayur Vihar Call Girls Service WhatsApp -> 9999965857 Available 24x7 ^ De...srsj9000
 
Introduction-To-Agricultural-Surveillance-Rover.pptx
Introduction-To-Agricultural-Surveillance-Rover.pptxIntroduction-To-Agricultural-Surveillance-Rover.pptx
Introduction-To-Agricultural-Surveillance-Rover.pptxk795866
 
Software and Systems Engineering Standards: Verification and Validation of Sy...
Software and Systems Engineering Standards: Verification and Validation of Sy...Software and Systems Engineering Standards: Verification and Validation of Sy...
Software and Systems Engineering Standards: Verification and Validation of Sy...VICTOR MAESTRE RAMIREZ
 
IVE Industry Focused Event - Defence Sector 2024
IVE Industry Focused Event - Defence Sector 2024IVE Industry Focused Event - Defence Sector 2024
IVE Industry Focused Event - Defence Sector 2024Mark Billinghurst
 
CCS355 Neural Network & Deep Learning UNIT III notes and Question bank .pdf
CCS355 Neural Network & Deep Learning UNIT III notes and Question bank .pdfCCS355 Neural Network & Deep Learning UNIT III notes and Question bank .pdf
CCS355 Neural Network & Deep Learning UNIT III notes and Question bank .pdfAsst.prof M.Gokilavani
 
Architect Hassan Khalil Portfolio for 2024
Architect Hassan Khalil Portfolio for 2024Architect Hassan Khalil Portfolio for 2024
Architect Hassan Khalil Portfolio for 2024hassan khalil
 
Artificial-Intelligence-in-Electronics (K).pptx
Artificial-Intelligence-in-Electronics (K).pptxArtificial-Intelligence-in-Electronics (K).pptx
Artificial-Intelligence-in-Electronics (K).pptxbritheesh05
 
INFLUENCE OF NANOSILICA ON THE PROPERTIES OF CONCRETE
INFLUENCE OF NANOSILICA ON THE PROPERTIES OF CONCRETEINFLUENCE OF NANOSILICA ON THE PROPERTIES OF CONCRETE
INFLUENCE OF NANOSILICA ON THE PROPERTIES OF CONCRETEroselinkalist12
 
What are the advantages and disadvantages of membrane structures.pptx
What are the advantages and disadvantages of membrane structures.pptxWhat are the advantages and disadvantages of membrane structures.pptx
What are the advantages and disadvantages of membrane structures.pptxwendy cai
 
main PPT.pptx of girls hostel security using rfid
main PPT.pptx of girls hostel security using rfidmain PPT.pptx of girls hostel security using rfid
main PPT.pptx of girls hostel security using rfidNikhilNagaraju
 
Call Us ≽ 8377877756 ≼ Call Girls In Shastri Nagar (Delhi)
Call Us ≽ 8377877756 ≼ Call Girls In Shastri Nagar (Delhi)Call Us ≽ 8377877756 ≼ Call Girls In Shastri Nagar (Delhi)
Call Us ≽ 8377877756 ≼ Call Girls In Shastri Nagar (Delhi)dollysharma2066
 
Concrete Mix Design - IS 10262-2019 - .pptx
Concrete Mix Design - IS 10262-2019 - .pptxConcrete Mix Design - IS 10262-2019 - .pptx
Concrete Mix Design - IS 10262-2019 - .pptxKartikeyaDwivedi3
 
Call Girls Narol 7397865700 Independent Call Girls
Call Girls Narol 7397865700 Independent Call GirlsCall Girls Narol 7397865700 Independent Call Girls
Call Girls Narol 7397865700 Independent Call Girlsssuser7cb4ff
 
Sachpazis Costas: Geotechnical Engineering: A student's Perspective Introduction
Sachpazis Costas: Geotechnical Engineering: A student's Perspective IntroductionSachpazis Costas: Geotechnical Engineering: A student's Perspective Introduction
Sachpazis Costas: Geotechnical Engineering: A student's Perspective IntroductionDr.Costas Sachpazis
 
Biology for Computer Engineers Course Handout.pptx
Biology for Computer Engineers Course Handout.pptxBiology for Computer Engineers Course Handout.pptx
Biology for Computer Engineers Course Handout.pptxDeepakSakkari2
 
Churning of Butter, Factors affecting .
Churning of Butter, Factors affecting  .Churning of Butter, Factors affecting  .
Churning of Butter, Factors affecting .Satyam Kumar
 
Study on Air-Water & Water-Water Heat Exchange in a Finned Tube Exchanger
Study on Air-Water & Water-Water Heat Exchange in a Finned Tube ExchangerStudy on Air-Water & Water-Water Heat Exchange in a Finned Tube Exchanger
Study on Air-Water & Water-Water Heat Exchange in a Finned Tube ExchangerAnamika Sarkar
 
Arduino_CSE ece ppt for working and principal of arduino.ppt
Arduino_CSE ece ppt for working and principal of arduino.pptArduino_CSE ece ppt for working and principal of arduino.ppt
Arduino_CSE ece ppt for working and principal of arduino.pptSAURABHKUMAR892774
 
Heart Disease Prediction using machine learning.pptx
Heart Disease Prediction using machine learning.pptxHeart Disease Prediction using machine learning.pptx
Heart Disease Prediction using machine learning.pptxPoojaBan
 

Recently uploaded (20)

UNIT III ANALOG ELECTRONICS (BASIC ELECTRONICS)
UNIT III ANALOG ELECTRONICS (BASIC ELECTRONICS)UNIT III ANALOG ELECTRONICS (BASIC ELECTRONICS)
UNIT III ANALOG ELECTRONICS (BASIC ELECTRONICS)
 
Gfe Mayur Vihar Call Girls Service WhatsApp -> 9999965857 Available 24x7 ^ De...
Gfe Mayur Vihar Call Girls Service WhatsApp -> 9999965857 Available 24x7 ^ De...Gfe Mayur Vihar Call Girls Service WhatsApp -> 9999965857 Available 24x7 ^ De...
Gfe Mayur Vihar Call Girls Service WhatsApp -> 9999965857 Available 24x7 ^ De...
 
Introduction-To-Agricultural-Surveillance-Rover.pptx
Introduction-To-Agricultural-Surveillance-Rover.pptxIntroduction-To-Agricultural-Surveillance-Rover.pptx
Introduction-To-Agricultural-Surveillance-Rover.pptx
 
Software and Systems Engineering Standards: Verification and Validation of Sy...
Software and Systems Engineering Standards: Verification and Validation of Sy...Software and Systems Engineering Standards: Verification and Validation of Sy...
Software and Systems Engineering Standards: Verification and Validation of Sy...
 
IVE Industry Focused Event - Defence Sector 2024
IVE Industry Focused Event - Defence Sector 2024IVE Industry Focused Event - Defence Sector 2024
IVE Industry Focused Event - Defence Sector 2024
 
CCS355 Neural Network & Deep Learning UNIT III notes and Question bank .pdf
CCS355 Neural Network & Deep Learning UNIT III notes and Question bank .pdfCCS355 Neural Network & Deep Learning UNIT III notes and Question bank .pdf
CCS355 Neural Network & Deep Learning UNIT III notes and Question bank .pdf
 
Architect Hassan Khalil Portfolio for 2024
Architect Hassan Khalil Portfolio for 2024Architect Hassan Khalil Portfolio for 2024
Architect Hassan Khalil Portfolio for 2024
 
Artificial-Intelligence-in-Electronics (K).pptx
Artificial-Intelligence-in-Electronics (K).pptxArtificial-Intelligence-in-Electronics (K).pptx
Artificial-Intelligence-in-Electronics (K).pptx
 
INFLUENCE OF NANOSILICA ON THE PROPERTIES OF CONCRETE
INFLUENCE OF NANOSILICA ON THE PROPERTIES OF CONCRETEINFLUENCE OF NANOSILICA ON THE PROPERTIES OF CONCRETE
INFLUENCE OF NANOSILICA ON THE PROPERTIES OF CONCRETE
 
What are the advantages and disadvantages of membrane structures.pptx
What are the advantages and disadvantages of membrane structures.pptxWhat are the advantages and disadvantages of membrane structures.pptx
What are the advantages and disadvantages of membrane structures.pptx
 
main PPT.pptx of girls hostel security using rfid
main PPT.pptx of girls hostel security using rfidmain PPT.pptx of girls hostel security using rfid
main PPT.pptx of girls hostel security using rfid
 
Call Us ≽ 8377877756 ≼ Call Girls In Shastri Nagar (Delhi)
Call Us ≽ 8377877756 ≼ Call Girls In Shastri Nagar (Delhi)Call Us ≽ 8377877756 ≼ Call Girls In Shastri Nagar (Delhi)
Call Us ≽ 8377877756 ≼ Call Girls In Shastri Nagar (Delhi)
 
Concrete Mix Design - IS 10262-2019 - .pptx
Concrete Mix Design - IS 10262-2019 - .pptxConcrete Mix Design - IS 10262-2019 - .pptx
Concrete Mix Design - IS 10262-2019 - .pptx
 
Call Girls Narol 7397865700 Independent Call Girls
Call Girls Narol 7397865700 Independent Call GirlsCall Girls Narol 7397865700 Independent Call Girls
Call Girls Narol 7397865700 Independent Call Girls
 
Sachpazis Costas: Geotechnical Engineering: A student's Perspective Introduction
Sachpazis Costas: Geotechnical Engineering: A student's Perspective IntroductionSachpazis Costas: Geotechnical Engineering: A student's Perspective Introduction
Sachpazis Costas: Geotechnical Engineering: A student's Perspective Introduction
 
Biology for Computer Engineers Course Handout.pptx
Biology for Computer Engineers Course Handout.pptxBiology for Computer Engineers Course Handout.pptx
Biology for Computer Engineers Course Handout.pptx
 
Churning of Butter, Factors affecting .
Churning of Butter, Factors affecting  .Churning of Butter, Factors affecting  .
Churning of Butter, Factors affecting .
 
Study on Air-Water & Water-Water Heat Exchange in a Finned Tube Exchanger
Study on Air-Water & Water-Water Heat Exchange in a Finned Tube ExchangerStudy on Air-Water & Water-Water Heat Exchange in a Finned Tube Exchanger
Study on Air-Water & Water-Water Heat Exchange in a Finned Tube Exchanger
 
Arduino_CSE ece ppt for working and principal of arduino.ppt
Arduino_CSE ece ppt for working and principal of arduino.pptArduino_CSE ece ppt for working and principal of arduino.ppt
Arduino_CSE ece ppt for working and principal of arduino.ppt
 
Heart Disease Prediction using machine learning.pptx
Heart Disease Prediction using machine learning.pptxHeart Disease Prediction using machine learning.pptx
Heart Disease Prediction using machine learning.pptx
 

Mathematics 3.pdf civil engineering concrete

  • 1. Bas 111 Mathematics 3 Dr. Fatma Zedan El Emam 1 CHAPTER 1 Partial Derivatives 1. 1 Functions of Several Variables Many functions depend on more than one independent variable. The function h r V 2   calculates the volume of a circular cylinder from its radius and height. In this section, we define functions of more than one independent variable. Definition 1 A function f of two variables, x and y, is a rule that assigns a unique real number f (x, y) to each point (x, y) in some set D in the xy-plane. The set D is the domain of f and its range is the set of values that f takes on, that is, {f (x, y) (x, y)  D}. We often write z = f (x, y) to make explicit the value taken on by f at the general point (x, y). The
  • 2. Bas 111 Mathematics 3 Dr. Fatma Zedan El Emam 2 variables x and y are independent variables and z is the dependent variable. A function of two variables is just a function whose domain is a subset of 2 R and whose range is a subset of . R (see Fig. 1.1.1), where the domain D is represented as a subset Fig. 1.1.1 of the xy -plane and the range is a set of numbers on a real line, shown as a z-axis. Definition 2 A function f of three variables, x, y, and z, is a rule that assigns a unique real number f ( x, y, z ) to each point (x, y, z) in some set D in three dimensional space. Example 1 Let ). ( ln 1 ) , ( 2 y x y y x f     Find f (e, 0) and sketch the domain of f.
  • 3. Bas 111 Mathematics 3 Dr. Fatma Zedan El Emam 3 Solution. By substitution 3 2 1 ) ( ln 1 ) 0 ( ln 1 0 ) 0 , ( 2 2          e e e f To find the domain of f , we note that 1  y is defined only when y ≥ −1, Fig. 1.1.2 While ) ( ln 2 y x  is defined only when y x   2 0 or 2 x y  . Thus, the domain of f consists of all points in they-plane for which 2 1 x y    . To sketch the domain, we first sketch the parabola 2 x y  and the line y = −1. The domain of f is then the region lying above or on the line y = −1 and below the parabola 2 x y  (See Fig.1.1.2). Example 2 Let 1 1 ) , (     x y x y x f Find f (3, 2) and sketch the domain of f.
  • 4. Bas 111 Mathematics 3 Dr. Fatma Zedan El Emam 4 Solution. By substitution 2 6 1 3 1 2 3 ) 2 , 3 (      f The domain of f is Fig. 1.1.3 } 0 1 , 0 1 ) , {(       x y x y x D } 1 , 0 1 ) , {(      x y x y x . The inequality , 0 1   y x or , 1    x y describes the points that lie on or above the line , 1    x y while , 1  x means that the points on the line , 1  x must be excluded from the domain. (See Fig.1.1.3) Example 3 Find the domain and the range of . 9 ) , ( 2 2 y x y x g    Solution. The domain of g is } 9 ) , {( } 0 9 ) , {( 2 2 2 2        y x y x y x y x D
  • 5. Bas 111 Mathematics 3 Dr. Fatma Zedan El Emam 5 which is the disk with center (0,0) and radius 3. (See Fig. 1.1.4) The range of g is }. ) , ( , 9 { 2 2 D y x y x z z     Since z is a positive square root, 0  z . Also, because , 9 9 2 2    y x we have 3 9 2 2    y x . So the range is ] 3 , 0 [ } 3 0 {    z z Domain of ) , ( y x g Fig.1.1.4 Graph of ) , ( y x g Example 4 Let 2 2 2 1 ) , , ( z y x z y x f     . Find ) 2 1 , 2 1 , 0 (  f and the domain of f. Solution. By substitution
  • 6. Bas 111 Mathematics 3 Dr. Fatma Zedan El Emam 6 2 1 2 1 2 1 ) 0 ( 1 ) 2 1 , 2 1 , 0 ( 2 2 2                    f The domain of f is } 0 1 ) , , {( 2 2 2      z y x z y x D } 1 ) , , {( 2 2 2     z y x z y x , we see that the domain of f consists of all points on or within the sphere 1 2 2 2    z y x 1.2 Limits For a function of one variable there are two one- sided limits at a point 0 x , namely, ) ( lim 0 x f x x   and ) ( lim 0 x f x x   reflecting the fact that there are only two directions from which x can approach 0 x , the right or the left. If    ) ( lim 0 x f x x ) ( lim 0 x f x x   , then
  • 7. Bas 111 Mathematics 3 Dr. Fatma Zedan El Emam 7 ) ( lim 0 x f x x does not exist. For functions of two variables the situation is more complicated because there are infinitely many different curves (paths) along which (x, y) can Fig. 1.2.1 approach ) , ( 0 0 y x (See Fig. 1.2.1). Theorem 1 (a) If L y x f  ) , ( as ) , ( ) , ( 0 0 y x y x  , then L y x f  ) , ( as ) , ( ) , ( 0 0 y x y x  along any path (b) If 1 ) , ( L y x f  as ) , ( ) , ( 0 0 y x y x  along a path 1 C and 2 ) , ( L y x f  as ) , ( ) , ( 0 0 y x y x  along a path 2 C , where 2 1 L L  , then ) , ( lim ) , ( ) , ( 0 0 y x f y x y x  does not exist.
  • 8. Bas 111 Mathematics 3 Dr. Fatma Zedan El Emam 8 • To show a limit does not exist, it is still enough to find two paths along which the limits are not equal. We give some suggestions here. You can try the following paths: 1) Horizontal line through ) 0 , 0 ( , the equation of such a path is 0  y . 2) Vertical line through ) 0 , 0 ( , the equation of such a path is 0  x . 3) Any straight line through ) 0 , 0 ( , the equation of the line with slope m through ) 0 , 0 ( is . mx y  4) Quadratic paths. For example, a typical quadratic path through ) 0 , 0 ( is 2 x y  or 2 y x  Definition 3 We say that a function ƒ(x, y) approaches the limit L as (x, y) approaches ), , ( 0 0 y x and write ) , ( lim ) , ( ) , ( 0 0 y x f y x y x  = L if, for every number , 0   there exists a
  • 9. Bas 111 Mathematics 3 Dr. Fatma Zedan El Emam 9 corresponding number 0   such that for all (x, y) in the domain of ƒ,    L y x f ) , ( whenever       2 0 2 0 ) ( ) ( 0 y y x x . Remark: We remark that the usual limit theorems hold for limits of functions of two variables. For example ) , ( ) , ( 0 0 lim y x y x  [f (x, y)  g (x, y)] = ) , ( ) , ( 0 0 lim y x y x  f (x, y)  ) , ( ) , ( 0 0 lim y x y x  g (x, y) Example 5 Calculating Limits a) 3 2 ) 1 , 0 ( ) , ( 5 3 lim y xy y x xy x y x      b) 2 2 ) 4 , 3 ( ) , ( lim y x y x    Solution. Direct substitution gives: a) 3 ) 1 ( ) 1 )( 0 ( 5 ) 1 ( ) 0 ( 3 ) 1 )( 0 ( 0 5 3 lim 3 2 3 2 ) 1 , 0 ( ) , (             y xy y x xy x y x b) 5 25 ) 4 ( ) 3 ( lim 2 2 2 2 ) 4 , 3 ( ) , (         y x y x
  • 10. Bas 111 Mathematics 3 Dr. Fatma Zedan El Emam 10 Example 6 Find . lim 2 ) 0 , 0 ( ) , ( y x xy x y x    Solution. Since the denominator y x  approaches 0 as ) 0 , 0 ( ) , (  y x , we multiply numerator and denominator by , y x  however, we produce an equivalent fraction whose limit we can find: ) )( ( ) )( ( lim lim 2 ) 0 , 0 ( ) , ( 2 ) 0 , 0 ( ) , ( y x y x y x xy x y x xy x y x y x          y x y x y x x y x      ) )( ( lim ) 0 , 0 ( ) , ( ) ( lim ) 0 , 0 ( ) , ( y x x y x    0 ) 0 0 ( 0    Example 7 Show that 2 2 2 2 ) 0 , 0 ( ) , ( lim y x y x y x    does not exist. Solution. Direct substitution gives:
  • 11. Bas 111 Mathematics 3 Dr. Fatma Zedan El Emam 11 0 0 lim 2 2 2 2 ) 0 , 0 ( ) , (     y x y x y x (indeterminate). More investigation is needed! Let 2 2 2 2 ) , ( y x y x y x f    . First let’s approach (0, 0) along the x-axis. Then 0  y gives 1 ) 0 , ( 2 2   x x x f for all , 0  x so, 1 ) , (  y x f as ) 0 , 0 ( ) , (  y x along the x-axis. Next let’s approach (0, 0) along the y-axis by putting . 0  x Then 1 ) , 0 ( 2 2     y y y f for all , 0  y so 1 ) , (   y x f as ) 0 , 0 ( ) , (  y x along the y-axis. (See Fig. 1.2.2) Since f has two different limits along 2 different paths, the given limit does not exist. Fig. 1.2.2
  • 12. Bas 111 Mathematics 3 Dr. Fatma Zedan El Emam 12 Example 8 Show that 2 2 ) 0 , 0 ( ) , ( lim y x xy y x   does not exist. Solution. Let 2 2 ) , ( y x xy y x f   . First let’s approach (0, 0) along the x-axis. If , 0  y then , 0 0 ) 0 , ( 2   x x f so 0 ) , (  y x f as ) 0 , 0 ( ) , (  y x along the x-axis. Next let’s approach (0, 0) along the y-axis. If , 0  x then , 0 0 ) , 0 ( 2   y y f so 0 ) , (  y x f as ) 0 , 0 ( ) , (  y x along the y-axis. Although we have obtained identical limits along the axes, that does not show that the given limit is 0. Now let’s approach ) 0 , 0 ( along another line, say y=x. For all , 0  x . 2 1 ) , ( 2 2 2    x x x y x f So, 2 1 ) , (  y x f as ) 0 , 0 ( ) , (  y x along y=x.
  • 13. Bas 111 Mathematics 3 Dr. Fatma Zedan El Emam 13 Since f has two different limits along 2 different paths, the given limit does not exist. Example 9 If , ) , ( 4 2 2 y x xy y x f   does ) , ( lim ) 0 , 0 ( ) , ( y x f y x  exist? Solution. Let’s approach (0, 0) along the line y=mx where m is the slope, and . 0 1 ) ( ) ( ) , ( ) , ( 2 4 2 4 4 2 3 2 4 2 2          x m x m x m x x m mx x mx x mx x f y x f So 0 ) , (  y x f as ) 0 , 0 ( ) , (  y x along y=mx. Thus, the limit as (x, y) approaches to the origin along any straight line is zero. But that does not show that the given limit is 0, if we now let ) 0 , 0 ( ) , (  y x along the parabola 2 y x  , we have   2 1 2 ) , ( ) , ( 4 4 4 2 2 2 2 2       y y y y y y y y f y x f . So 2 1 ) , (  y x f as ) 0 , 0 ( ) , (  y x along 2 y x 
  • 14. Bas 111 Mathematics 3 Dr. Fatma Zedan El Emam 14 Since different paths lead to different limiting values, the given limit does not exist. Example 10 Find 2 2 2 ) 0 , 0 ( ) , ( 3 lim y x y x y x   if it exists Solution. We could show that the limit along any line through the origin is 0. This doesn’t prove that the given limit is 0, but the limits along the parabolas 2 x y  and 2 y x  also turn out to be 0, so we begin to suspect that the limit does exist and is equal to 0.
  • 15. Bas 111 Mathematics 3 Dr. Fatma Zedan El Emam 15 Hence, by Definition 3, 0 3 lim 2 2 2 ) 0 , 0 ( ) , (    y x y x y x 1. 3 Continuity Definition 4 A function ) , ( y x f is continuous at the point ) , ( 0 0 y x if 1. f is defined at ) , ( 0 0 y x , 2. ) , ( lim ) , ( ) , ( 0 0 y x f y x y x  exists, 3. ). , ( ) , ( lim 0 0 ) , ( ) , ( 0 0 y x f y x f y x y x   A function is continuous if it is continuous at every point of its domain. Example 11 Every polynomial in x, y is continuous on 2 R . Each rational function is continuous in its
  • 16. Bas 111 Mathematics 3 Dr. Fatma Zedan El Emam 16 domain. For instance, the rational function y x y x x y x f    3 2 ) , ( is continuous on }. 0 ) , {( 2     y x R y x D Example 12 let . 3 ) , ( 2 2 2 y x y x y x f   Where is f continuous? Example 13 let          ). 0 , 0 ( ) ( , 0 ) 0 , 0 ( ) ( , 3 ) , ( 2 2 2 x,y x,y y x y x y x f Show that f is continuous on 2 R . Example 14 Show that          ) 0 , 0 ( ) ( , 0 ) 0 , 0 ( ) ( , 2 ) , ( 2 2 x,y x,y y x xy y x f is continuous at every point except the origin. Solution. The function f is continuous at any point ) 0 , 0 ( ) (  x,y because its values are then
  • 17. Bas 111 Mathematics 3 Dr. Fatma Zedan El Emam 17 given by a rational function of x and y. At (0, 0), the value of f is defined, but f has no limit as ) 0 , 0 ( ) , (  y x , as we will now see. For every value of m, the function f has a constant value on the line y=mx, , 0  x because . 1 2 2 ) ( ) ( 2 2 ) , ( 2 2 2 2 2 2 2 2 2 m m x m x mx mx x mx x y x xy y x f mx y mx y            Therefore, f has this number as its limit as (x, y) approaches (0, 0) along the line:   . 1 2 ) , ( lim ) , ( lim 2 ) 0 , 0 ( ) , ( along ) 0 , 0 ( ) , ( m m y x f y x f mx y y x mx y y x        This limit changes with m. There is therefore no single number we may call the limit of f as (x, y) approaches the origin. Then, the limit does not exist and the function is not continuous.
  • 18. Bas 111 Mathematics 3 Dr. Fatma Zedan El Emam 18 Example 15 Where is the function 2 2 2 2 ) , ( y x y x y x f    continuous? Solution. The function is discontinuous at (0, 0) because it is not defined at (0, 0). Since f (x, y) is a rational function, it is continuous on its domain, which is the set )}. 0 , 0 ( ) , ( ) , {(   y x y x D Example 16 Let           ). 0 , 0 ( ) ( , 0 ) 0 , 0 ( ) ( , ) , ( 2 2 2 2 x,y x,y y x y x y x f Here, f is defined at (0,0) but is still discontinuous there because 2 2 2 2 ) 0 , 0 ( ) , ( lim y x y x y x    does not exist (see Example 7). Remark: One may compute limits using polar coordinates. This is especially convenient for limits at the origin and for those expressions that are independent of θ.
  • 19. Bas 111 Mathematics 3 Dr. Fatma Zedan El Emam 19 Example 17 Find ). ln( ) ( lim 2 2 2 2 ) 0 , 0 ( ) , ( y x y x y x    Solution. Let ) , (  r be polar coordinates of the point ) , ( y x with 0  r . Then we have 2 2 2 , sin , cos y x r r y r x       . Moreover, since 0  r we have , 2 2 y x r   so that   0 r if and only if ) 0 , 0 ( ) , (  y x . Thus, 0 ) ( lim rule) s Hopital’ L’ ( / 2 / 2 lim / 1 ln 2 lim ln lim ) ln( ) ( lim 2 0 3 0 2 0 2 2 0 2 2 2 2 ) 0 , 0 ( ) , (                   r r r r r r r y x y x r r r r y x Remark: The graph of ) ln( ) ( ) , ( 2 2 2 2 y x y x y x f    in Example 17 is a surface with a hole at the origin (Figure 1.3.1). We can remove this discontinuity by defining f (0, 0) to be 0. Figure 1.3.1
  • 20. Bas 111 Mathematics 3 Dr. Fatma Zedan El Emam 20 Exercise 1 1-5 Find and sketch the domain of the function. 6-11 Use limit laws to evaluate the 12-13 Show that the limit does not exist by considering the limits as (x, y)→(0, 0) along the coordinate axes. 14-17 Determine whether the limit exists. If so, find its value.
  • 21. Bas 111 Mathematics 3 Dr. Fatma Zedan El Emam 21 18-20 Evaluate the limit using the substitution 2 2 y x z   and observing that   0 z if and only if (x, y)→(0, 0). 21- 22 Evaluate the limits by converting to polar coordinates 24-27 Determine the set of points at which the function is continuous.
  • 22. Bas 111 Mathematics 3 Dr. Fatma Zedan El Emam 22 1. 4 Partial Derivatives Definition 5 If z = f (x, y), then the first partial derivative of f with respect to x (w.r.t. x) is denoted by x f   or x f and is given by h y x f y h x f x f h ) , ( ) , ( lim 0       Similarly, the partial derivative of f w.r.t. y is denoted by y f   or y f and is given by k y x f k y x f y f k ) , ( ) , ( lim 0       The slope of the curve ) , ( 0 y x f at the point )) , ( , , ( 0 0 0 0 y x f y x P in the plane 0 y y  is the value of the partial derivative of ƒ w.r.t. x at ) , ( 0 0 y x .
  • 23. Bas 111 Mathematics 3 Dr. Fatma Zedan El Emam 23 Rule for Finding Partial Derivatives of z = f (x, y) 1. To find x f , regard y as a constant and differentiate w.r.t. x. 2. To find y f , regard x as a constant and differentiate w.r.t. y. Example 1 Find the values of x f   and y f   at the point (4, -5) if . 1 3 ) , ( 2     y xy x y x f
  • 24. Bas 111 Mathematics 3 Dr. Fatma Zedan El Emam 24 Solution. To find x f   , we regard y as a constant and differentiate w.r.t. x: . 3 2 0 0 1 3 2 ) 1 3 ( 2 y x y x y xy x x x f                 the value of x f   at (4, -5) is 2(4) + 3(-5) = -7. To find y f   , we regard x as a constant and differentiate w.r.t. y: . 1 3 0 1 1 3 0 ) 1 3 ( 2                 x x y xy x y y f the value of y f   at (4, -5) is 3(4) + 1 = 13. Example 2 Find y f   if . sin ) , ( xy y y x f  Solution. We treat x as a constant and ƒ as a product of y and sin xy:
  • 25. Bas 111 Mathematics 3 Dr. Fatma Zedan El Emam 25 . sin cos ) (sin ) ( ) cos ( ) ( ) (sin sin ) sin ( xy xy xy xy xy y xy y y y xy xy y y xy y y y f                  Example 3 Find x f and y f if . cos 2 ) , ( x y y y x f   Solution. We treat x as a quotient. With y held constant, we get . ) cos ( sin 2 ) cos ( ) sin ( 2 ) 0 ( ) cos ( ) cos ( ) cos ( 2 ) 2 ( ) cos ( cos 2 2 2 2 x y x y x y x y x y x y x y x y y x x y x y y x fx                           With x held constant, we get 2 ) cos ( ) cos ( 2 ) 2 ( ) cos ( cos 2 x y x y y y y y x y x y y y fy                   
  • 26. Bas 111 Mathematics 3 Dr. Fatma Zedan El Emam 26 . ) cos ( cos 2 ) cos ( ) 1 ( 2 ) 2 ( ) cos ( 2 2 x y x x y y x y       Example 4 If , 1 sin ) , (         y x y x f calculate x f   and y f   . Solution. Using the Chain Rule for functions of one variable, we have y y x y x x y x x f                               1 1 1 cos 1 1 cos 2 ) 1 ( 1 cos 1 1 cos y x y x y x y y x y f                                • Implicit differentiation works for partial derivatives the way it works for ordinary derivatives, as the next example illustrates Example 5 Find x z   / if z is defined implicitly as a function of x and y by the equation y x z yz    ln
  • 27. Bas 111 Mathematics 3 Dr. Fatma Zedan El Emam 27 Solution. We differentiate both sides of the w.r.t. x, holding y constant and treating z as a differentiable function of x: . 1 1 1 0 1 1 ln ) (                                    yz z x z x z z y x z z x z y x y x x z x yz x Functions Of More Than Two Variables The definitions of the partial derivatives of functions of more than two independent variables are ordinary derivatives with respect to one variable, taken while the other independent variables are held constant. Example 6 Calculate z f   , if x, y, and z are independent variables and ). 3 ( sin ) , , ( z y x z y x f  
  • 28. Bas 111 Mathematics 3 Dr. Fatma Zedan El Emam 28 Solution. ). 3 ( cos 3 ) 3 ( ) 3 ( cos ) 3 ( sin )] 3 ( sin [ z y x z y z z y x z y z x z y x z z f                  Example 7 If , sin 3 2 xz e z y x w   find , / , / y w x w     and . / z w   Solution. xz ze z xy x w     sin 2 3 z y x y w sin 3 2 2    xz xe z y x x w     cos 3 2 Second-Order Partial Derivatives When we differentiate a function ƒ(x, y) twice, we produce its second-order derivatives. These derivatives are usually denoted by
  • 29. Bas 111 Mathematics 3 Dr. Fatma Zedan El Emam 29 2 2 ) ( x f x f x f f f x xx x x x                   x y f x f y f f f y xy y x x                    2 ) ( y x f y f x f f f x yx x y y                    2 ) ( 2 2 ) ( y f y f y f f f y yy y y y                   Theorem 2 Let f be a function of x and y If xy y x f f f f , , , and yx f are continuous on an open region R, then yx xy f f  throughout R. If w = f (x, y, z) and f has continuous second partial derivatives, then the following equalities hold for mixed partials: . ; ; 2 2 2 2 2 2 z y w y z w z x w x z w y x w x y w                     
  • 30. Bas 111 Mathematics 3 Dr. Fatma Zedan El Emam 30 Example 8 Find the Second Partial Derivatives of f if . 3 2 ) , ( 2 2 3 x y x y x y x f    Solution. Since the first Partial of f are 3 4 3 ) , ( 2 2    xy y x y x fx and , 2 2 ) , ( 2 3 x y x y x fy   then y xy xy y x x y x fxx 4 6 ) 3 4 3 ( ) , ( 2 2 2        x y x xy y x y y x fxy 4 6 ) 3 4 3 ( ) , ( 2 2 2        x y x x y x x y x fyx 4 6 ) 2 2 ( ) , ( 2 2 3       . 2 ) 2 2 ( ) , ( 3 2 3 x x y x y y x fyy      Example 9 Find the Second Partial Derivatives of f if . ) , ( 4 3 2 y x y x y x f   Solution. Since the first Partial of f are y x xy y x fx 3 3 4 2 ) , (   and , 3 ) , ( 4 2 2 x y x y x fy   then
  • 31. Bas 111 Mathematics 3 Dr. Fatma Zedan El Emam 31 y x y y x xy x y x fxx 2 3 3 3 12 2 ) 4 2 ( ) , (       3 2 3 3 4 6 ) 4 2 ( ) , ( x xy y x xy y y x fxy       3 2 4 2 2 4 6 ) 3 ( ) , ( x xy x y x x y x fyx       . 6 ) 3 ( ) , ( 2 4 2 2 y x x y x y y x fyy      Third and higher partial derivatives are defined in like manner. For example, xxx f x f x x f                 2 2 3 3 , yyyy f y f y y f                 3 3 4 4 xyy f x y f y x y f                   2 2 3 , xxyy f x y f y x y f                   2 3 2 2 4 .
  • 32. Bas 111 Mathematics 3 Dr. Fatma Zedan El Emam 32 and so on. If first, second, and third partial derivatives are continuous, then xxy yxx xyx f f f   and . yyx xyy yxy f f f   Example 9 Let . ) , ( 2 y e y y x f x   Find . xyy f Solution.   x xyy e y y x f y x y f f 2 2 2 2 2 2 3                     x x e ye y 2 2     1. 5 The Chain Rule Recall that the Chain Rule for functions of a single variable gives the rule for differentiating a composite function: If ) (x f y  and ) (t g x  , where f and g are differentiable functions, then y is indirectly a differentiable function of t and dt dx dx dy dt dy 
  • 33. Bas 111 Mathematics 3 Dr. Fatma Zedan El Emam 33 We will now derive a version of the chain rule for functions of two variables. The Chain Rule for Derivative Suppose that ) , ( y x f z  is a differentiable function of x and y, where ) (t g x  and ) (t h y  are both differentiable functions of t. Then z is a differentiable Fig. 1.5.1 function of t and dt dy y z dt dx x z dt dz       . See “tree diagram” Fig. 1.5.1 Example 11 Suppose that 3 2 2 , , t y t x y x z   
  • 34. Bas 111 Mathematics 3 Dr. Fatma Zedan El Emam 34 Use the chain rule to find dt dz/ , and check the result by expressing z as a function of t and differentiating directly. Solution. By the chain rule 6 2 4 5 2 2 7 ) 3 )( ( ) 2 )( 2 ( ) 3 )( ( ) 2 )( 2 ( t t t t t t x t xy dt dy y z dt dx x z dt dz            Alternatively, we can express z directly as a function of t , 7 3 2 2 2 ) ( ) ( t t t y x z    and then differentiate to obtain 6 7 / t dt dz  . Example 12 If , 3 4 2 xy y x z   where t x 2 sin  and , cost y  find dt dz when . 0  t Solution. The Chain Rule gives ) sin )( 12 ( ) 2 cos 2 )( 3 2 ( 3 2 4 t xy x t y xy dt dy y z dt dx x z dt dz           
  • 35. Bas 111 Mathematics 3 Dr. Fatma Zedan El Emam 35 when t=0, we have x = sin 0 = 0 and y = cos 0 = 1. Therefore 6 ) 0 sin )( 0 0 ( ) 0 cos 2 )( 3 0 ( 0        t dt dz The Chain Rules for Partial Derivatives Suppose that ) , ( y x f z  is a differentiable function of x and y, where ) , ( v u g x  and ) , ( v u h y  are both differentiable functions of u and v. Then u y y z u x x z u z             v y y z v x x z v z             Example 13 Given that v u y v u x e z xy / , 2 ,    
  • 36. Bas 111 Mathematics 3 Dr. Fatma Zedan El Emam 36 find ∂z/∂u and ∂z/∂v using the chain rule. Solution. , 1 4 2 2 2 1 ) ( ) 2 )( ( ) / )( 2 ( ) / )( 2 ( v u v u v u v u xy xy xy e v u e v v u v u e v x y v xe ye u y y z u x x z u z                                                ) / )( 2 ( 2 2 ) / )( 2 ( 2 2 2 2 ) 2 ( ) ( ) 1 )( ( v u v u v u v u xy xy xy e v u e v u v u v u e v u x y v u xe ye v y y z v x x z v z                                                       Example 14 If y e z x sin  , where 2 st x  and t s y 2  , find s z   / and t z   / using the chain rule Solution.
  • 37. Bas 111 Mathematics 3 Dr. Fatma Zedan El Emam 37 ), cos( 2 ) sin( ) 2 )( cos ( ) )( sin ( 2 2 2 2 2 2 t s ste t s e t st y e t y e s y y z s x x z s z st st x x                 ) cos( ) sin( 2 ) )( cos ( ) 2 )( sin ( 2 2 2 2 2 2 t s e s t s ste s y e st y e t y y z t x x z t z st st x x                 1.6 Implicit Differentiation Partial derivatives can be used to find derivatives of functions that are defined implicitly. Suppose, an equation F(x, y) = 0 defines a differentiable function f of one variable x such that F(x, f (x)) = 0 for all x in the domain D of f. If we let w = F(u, y), where u = x and y = f (x) then applying Chain Rule and using the fact that w =F(x, f(x)) =0 for all x in D, , 0        dx dy y w dx du u w dx dw
  • 38. Bas 111 Mathematics 3 Dr. Fatma Zedan El Emam 38 that is, . 0 ) ( ) 1 (        x f y w u w If , 0 /    y w then (since u = x) . ) , ( ) , ( / / ) ( y x F y x F y w x w x f y x          We may summarize this discussion as follows. Theorem 3 If the equation F(x, y) = 0 defines implicitly as a differentiable function x and if 0 ) , (  y x Fy one variable, then . ) , ( ) , ( y x F y x F dx dy y x   Example 15 Find dx dy if . 0 3 2 3    x y x Solution. Take . 3 ) , ( 2 3    x y x y x F Then . 2 3 2 2 yx y x F F dx dy y x      Alternatively, differentiating implicitly yields
  • 39. Bas 111 Mathematics 3 Dr. Fatma Zedan El Emam 39 0 0 2 3 2 2           dx dy y x y x or yx y x dx dy 2 3 2 2    which agrees with the result obtained first. Example 16 Find dx dy if . 0 2 sin 2    y y x Solution. Take . 2 sin ) , ( 2 y y x y x F    Then . 2 cos 2      y x F F dx dy y x Theorem 4 If the equation F(x, y, z) = 0 defines implicitly as a differentiable function of x and y, and if , 0 ) , (  y x Fz then z x F F x z     and . z y F F y z     Example 17 Consider the sphere . 1 2 2 2    z y x Find ∂z/∂x and ∂z/∂y at the point . 3 2 , 3 1 , 3 2       Solution. By Theorem 4 with
  • 40. Bas 111 Mathematics 3 Dr. Fatma Zedan El Emam 40 , 1 ) , , ( 2 2 2     z y x z y x F z x z x F F x z z x         2 2 and z y z y F F y z z y         2 2 At the point , 3 2 , 3 1 , 3 2       evaluating these derivatives gives 1 /     x z and 2 1 /     y z .
  • 41. Bas 111 Mathematics 3 Dr. Fatma Zedan El Emam 41 Exercise 2 22-26 Use the chain rule to find dz/dt.
  • 42. Bas 111 Mathematics 3 Dr. Fatma Zedan El Emam 42 Use appropriate forms of the chain rule to find ∂z/∂u and ∂z/∂v. 30. Find the value of x z   / at the point (1,1,1) if z is defined implicitly as a function of x and y by the equation 31. Find x z   / and y z   / if z is defined implicitly as a function of x and y by the equation 32. Find dx dy / if . 0 1 5 4 3 3 4      x x y y 33-35 Find dy/dx 36-37 Find ∂z/∂x and ∂z/∂y
  • 43. Bas 111 Mathematics 3 Dr. Fatma Zedan El Emam 43 1.7 Directional Derivatives and Gradient Vectors 1.7.1 The Directional Derivative Suppose that the function ƒ(x, y) is defined throughout a region R in the xy-plane, that ) , ( 0 0 0 y x P is a point in R, and that j u i u 2 1 u   is a unit vector. Then the equations 2 0 1 0 , hu y y hu x x     parameterize the line through 0 P parallel to u. If the parameter h measures arc length from 0 P in the direction of u, we find the rate of change of ƒ at 0 P in the direction of u by calculating ) , ( 0 0 y x f Du from the following definition (Fig. 1). Fig. 1
  • 44. Bas 111 Mathematics 3 Dr. Fatma Zedan El Emam 44 Definition 1 The directional derivative of f at ) , ( 0 0 y x in the direction of a unit vector j u i u 2 1 u   is h y x f hu y hu x f y x f D h u ) , ( ) , ( ) , ( 0 0 2 0 1 0 0 0 0 lim      (1) if this limit exists. Note If i  u ( 1 1  u and 0 2  u ), then Equation (1) gives ) , ( ) , ( ) , ( ) , ( 0 0 0 0 0 0 0 0 0 lim y x f h y x f y h x f y x f D x h i      That is, the directional derivative of f in the x- direction is the partial derivative of f in the x- direction, as expected. Similarly, we can show that ). , ( ) , ( 0 0 0 0 y x f y x f D y j 
  • 45. Bas 111 Mathematics 3 Dr. Fatma Zedan El Emam 45 If a is any vector having the same direction as u we shall also refer to ) , ( u y x f D as the directional derivative of f in the direction of a. Theorem 1 If f is a differentiable function of x and y, then f has a directional derivative in the direction of any unit vector j u i u 2 1 u   and 2 1 u ) , ( ) , ( ) , ( u y x f u y x f y x f D y x   (2) Example 1 Find the directional derivative of 2 3 ) , ( y x y x f  at the point ( - 1, 2) in the direction of the vector a = 4i - 3j. Solution We wish to find ) 2 , 1 ( u  f D , where u is a unit vector having the direction of a. Since j i j i a a 5 3 5 4 3 4 ) 3 4 ( u 2 2       , 2 2 3 ) , ( y x y x fx  and , 2 ) , ( 3 y x y x fy  it follows that 2 1 u ) , ( ) , ( ) , ( u y x f u y x f y x f D y x  
  • 46. Bas 111 Mathematics 3 Dr. Fatma Zedan El Emam 46 . 5 3 2 5 4 3 3 2 2                y x y x Hence . 12 5 12 5 48 5 3 ) 2 ( ) 1 ( 2 5 4 ) 2 ( ) 1 ( 3 ) 2 , 1 ( 3 2 2 u                      f D • If the unit vector u makes an angle  with the positive x-axis (as in Fig. 2), then we can write j i   sin cos u   and the formula in Theorem 1 becomes   sin ) , ( cos ) , ( ) , ( u y x f y x f y x f D y x   (3) Fig. 2
  • 47. Bas 111 Mathematics 3 Dr. Fatma Zedan El Emam 47 Example 2 Find the directional derivative of xy e y x f  ) , ( at t point ( - 2, 0) in the direction of the unit vector that makes an angle of 3 /  with the positive x-axis Solution The partial derivatives of f are The unit vector u that makes an angle of 3 /  with the positive x-axis is Thus, from (3) 1.7.2 The Gradient The directional derivative ) , ( u y x f D can be written as the dot product of the unit vector
  • 48. Bas 111 Mathematics 3 Dr. Fatma Zedan El Emam 48 j u i u 2 1 u   and the vector j y x f i y x f y x ) , ( ) , (  Thus, 2 1 2 1 u ) , ( ) , ( ] ) , ( ) , ( [ ) ( ) , ( u y x f u y x f j y x f i y x f j u i u y x f D y x y x       The vector j y x f i y x f y x ) , ( ) , (  plays an important role in many other computations and is given a special name. Definition 2 Let f be a function of x and y. The gradient of f is the vector function j y x f i y x f y x f y x ) , ( ) , ( ) , (    (4) The symbol f  (read “nabla”) Example 3 If , sin ) , ( xy e x y x f   then j xe i ye x j y x f i y x f y x f xy xy y x       ) (cos ) , ( ) , ( ) , (
  • 49. Bas 111 Mathematics 3 Dr. Fatma Zedan El Emam 49 and i f 2 ) 1 , 0 (   Example 4 Find the gradient of x y y x y x f ln sin ) , (   at the point ) , (  e . Solution Since we have So the gradient of f at ) , (  e is Formula (3) can now be written as u ) , ( ) , ( u    y x f y x f D (5) Example 5 Find the directional derivative of the function y y x y x f 4 ) , ( 3 2   at the point (2, -1) in the direction of the vector . 5 2 v j i  
  • 50. Bas 111 Mathematics 3 Dr. Fatma Zedan El Emam 50 Solution We first compute the gradient vector at (2, -1): j y x i xy y x f ) 4 3 ( 2 ) , ( 2 2 3     j i f 8 4 ) 1 , 2 (      Note that v is not a unit vector, but since 29 v  , the unit vector in the direction of v is j i j i 29 5 29 2 5 2 ) 5 2 ( v v u 2 2       Therefore, by Equation 5, we have         29 32 29 5 8 2 4 29 5 29 2 8 4 u ) 1 , 2 ( ) 1 , 2 ( u                     j i j i f f D • Properties of the Gradient Theorem 2 Suppose f is differentiable at the point (x, y). 1. If 0, ) , (   y x f then 0 ) , ( u  y x f D for every u.
  • 51. Bas 111 Mathematics 3 Dr. Fatma Zedan El Emam 51 2. The maximum value of ) , ( u y x f D is ) , ( y x f  , and this occurs when u has the same direction as ) , ( y x f  . 3. The minimum value of ) , ( u y x f D is ) , ( y x f   , and this occurs when u has the direction of ) , ( y x f   . Example 6 Let . ) , ( 2 y e x y x f  Find the maximum value of a directional derivative at (-2, 0) and find the unit vector in the direction in which the maximum value occurs. Solution Since j e x i xe j y x f i y x f y x f y y y x 2 2 ) , ( ) , ( ) , (      the gradient of f at (−2, 0) is j i f 4 4 ) 0 , 2 (      By Theorem 2, the maximum value of the directional derivative is 2 4 32 4 ) 4 ( ) 0 , 2 ( 2 2       f
  • 52. Bas 111 Mathematics 3 Dr. Fatma Zedan El Emam 52 This maximum occurs in the direction of ∇f (−2, 0). The unit vector in this direction is j i j i f f 2 1 2 1 ) 4 4 ( 2 4 1 0) 2, ( 0) 2, ( u           • Functions of Three Variables Theorem 3 If f is a differentiable function of x, y and z, then f has a directional derivative in the direction of any unit vector k u j u i u u 3 2 1    and 3 2 1 ) , , ( ) , , ( ) , , ( ) , , ( u z y x f u z y x f u z y x f z y x f D z y x u    (6) The gradient of f is k z y x f j z y x f i z y x f z y x f z y x ) , , ( ) , , ( ) , , ( ) , , (     We also write u ) , , ( ) , , ( u    z y x f z y x f D (7) Example 7 If z yz y x z y x f    3 2 ) , , ( (a) Find the gradient of f
  • 53. Bas 111 Mathematics 3 Dr. Fatma Zedan El Emam 53 (b) Find the directional derivative of f at the point ) 0 , 2 , 1 (  in the direction of the vector . 2 2 k j i a    Solution (a) The gradient of f is k yz j z x xyi k z y x f j z y x f i z y x f z y x f z y x ) 1 3 ( ) ( 2 ) , , ( ) , , ( ) , , ( ) , , ( 2 3 2           (b) At (1,−2, 0) we have k j i f       4 ) 0 , 2 , 1 ( Since a is not a unit vector, we normalize it, getting k j i k j i a a 3 2 3 1 3 2 ) 2 2 ( 9 1 u        Therefore Equation 7 gives     3 3 2 3 1 3 2 4 3 2 3 1 3 2 4 u ) 0 , 2 , 1 ( ) 0 , 2 , 1 ( u                               k j i k j i f f D
  • 54. Bas 111 Mathematics 3 Dr. Fatma Zedan El Emam 54 Exercise 3 1-4 Find f Du at P. 5-9 Find the directional derivative of f at P in the direction of a. 10-11 Find the directional derivative of f at P in the direction of a vector making the counterclockwise angle θ with the positive x-axis.
  • 55. Bas 111 Mathematics 3 Dr. Fatma Zedan El Emam 55 12-13 Find ∇z and ∇w. 14-15 Find the gradient of f at the indicated point. 16-18 Find a unit vector in the direction in which f increases most rapidly at P, and find the rate of change of f at P in that direction. 19-20 Find a unit vector in the direction in which f decreases most rapidly at P, and find the rate of change of f at P in that direction.
  • 56. Bas 111 Mathematics 3 Dr. Fatma Zedan El Emam 56 CHAPTER 2 Multiple Integrals The notion of a definite integral can be extended to functions of two or more variables. In this section we will discuss the double integral, which is the extension to functions of two variables. • Review of the Definite Integral We know that the area A between the graph of f and the interval [a, b] is            n i i i n x b a x x f dx x f A i 1 0 max ) ( lim ) ( 2.1 Volumes and Double Integrals over rectangles In a similar manner, let us consider a function
  • 57. Bas 111 Mathematics 3 Dr. Fatma Zedan El Emam 57 ) , ( y x f of two variable x and y that is continuous on a rectangular region R in the xy-plane given by R= {(x, y): d y c b x a     , } or ] , [ ] , [ d c b a R   . We imagine R to be covered by a net work of lines parallel to the x- and y-axes (Fig. 2.1). These lines divide the region R into small pieces of area y x A     . We number these in some order n A A A A     , , , , 3 2 1  , then the volume of the solid enclosed between the surface ) , ( y x f z  and the region R is defined by             n k k k k n A R A y x f dA y x f V k 1 0 ) , ( lim ) , ( Here,   n indicates the process of increasing the number of subrectangles of the rectangle R in such a way that both the lengths and the widths of the subrectangles approach zero.
  • 58. Bas 111 Mathematics 3 Dr. Fatma Zedan El Emam 58 Fig. 2.1  R dA y x f ) , ( is called the double integral of ) , ( y x f over R. • Properties Of Double Integrals If ƒ(x, y) and g(x, y) are continuous, then
  • 59. Bas 111 Mathematics 3 Dr. Fatma Zedan El Emam 59 (i) , ) , ( ) , (    R R dA y x f c dA y x cf if c is any real number.       R R R dA y x g dA y x f dA y x g y x f ) , ( ) , ( )] , ( ) , ( [ (ii) (iii) If R is the union of two nonoverlapping regions 1 R and 2 R      2 1 ) , ( ) , ( ) , ( R R R dA y x f dA y x f dA y x f (iv) 0 ) , (   R dA y x f if 0 ) , (  y x f on R. • Evaluating Double Integrals Fubini’s Theorem If ) , ( y x f is continuous on ] , [ ] , [ d c b a R   then, dy dx y x f dx dy y x f dA y x f d c b a b a d c R                    ) , ( ) , ( ) , (
  • 60. Bas 111 Mathematics 3 Dr. Fatma Zedan El Emam 60 These integrals are called iterated or repeated integrals. The symbols  b a dx y x f ) , ( and  d c dy y x f ) , ( denote partial definite integrals; the first integral, called the partial definite integral w.r.t. x, is evaluated by holding y fixed and integrating w.r.t. x, and the second integral, called the partial definite integral w.r.t. y, is evaluated by holding x fixed and integrating w.r.t. y. Example 1 Evaluate the iterated integrals. (a) dx dy y x   3 0 2 1 2 (b) dy dx y x   2 1 3 0 2 Solution (a) Here we first integrate w. r. t. y:
  • 61. Bas 111 Mathematics 3 Dr. Fatma Zedan El Emam 61 2 27 2 2 3 2 3 0 3 2 3 0 2 1 2 3 0 2 3 0 2 1 2 3 0 2 1 2                               x dx x dx y x dx dy y x dx dy y x y y (b) Here we first integrate w. r. t. x: 2 27 2 9 9 3 2 1 2 2 1 3 0 3 2 1 2 1 3 0 2 2 1 3 0 2                               y dy y dy x y dy dx y x dy dx y x x x Example 2 Evaluate the iterated integrals. (a) dx dy y x ) 2 40 ( 3 1 4 2    (b) dy dx y x ) 2 40 ( 4 2 3 1    Solution
  • 62. Bas 111 Mathematics 3 Dr. Fatma Zedan El Emam 62     112 6 80 ) 12 80 ( )] 4 80 ( ) 16 160 [( 40 ) 2 40 ( ) 2 40 ( (a) 3 1 2 3 1 3 1 3 1 4 2 2 3 1 4 2 3 1 4 2                             x x dx x dx x x dx xy y dx dy xy dx dy y x y     112 4 80 ] 8 80 [ )] 40 ( ) 9 120 [( 40 ) 2 40 ( ) 2 40 ( (b) 4 2 2 4 2 4 2 4 2 3 1 2 4 2 3 1 4 2 3 1                            y y dy y dy y y dy y x x dy dx xy dy dx y x x
  • 63. Bas 111 Mathematics 3 Dr. Fatma Zedan El Emam 63 Example 3 Calculate  R dA y x f ) , ( for y x y x f 2 6 1 ) , (   and R: . 1 1 , 2 0      y x Solution     . 4 8 2 ) 16 2 ( 2 ) 6 1 ( ) , ( 1 1 2 1 1 2 0 1 1 3 1 1 2 0 2                     y y dy y dy y x x dy dx y x dA y x f x x R Another Sol.   dx y x y dx dy y x dA y x f y y R 1 1 2 0 2 2 2 0 1 1 2 3 ) 6 1 ( ) , (            
  • 64. Bas 111 Mathematics 3 Dr. Fatma Zedan El Emam 64          2 0 2 0 2 2 4 2 )] 3 1 ( ) 3 1 [( dx dx x x Example 4 Use a double integral to find the volume of the solid that is bounded above by the plane y x z    4 and below by the rectangle ] 2 , 0 [ ] 1 , 0 [   R (Fig. 2.2). Solution The volume is the double integral of y x z    4 over R, this can be obtained from either of the iterated integrals     2 0 1 0 ) 4 ( dy dx y x or dx dy y x     1 0 2 0 ) 4 ( Using the first of these, we obtain
  • 65. Bas 111 Mathematics 3 Dr. Fatma Zedan El Emam 65          2 0 1 0 ) 4 ( ) 4 ( dy dx y x dA y x V R                     2 0 2 0 1 0 2 2 7 2 4 dy y dy xy x x x 5 2 2 7 2 0 2          y y You can check this result by evaluating the second integral. Fig. 2.2 Example 5 Evaluate  R dA x y2 ,     1 0 , 2 3 : ,       y x y x R .
  • 66. Bas 111 Mathematics 3 Dr. Fatma Zedan El Emam 66 Solution 6 5 6 5 2 5 2 1 1 0 3 1 0 2 1 0 2 3 2 2 1 0 2 3 2 2                                   y dy y dy x y dy dx x y dA x y x R Fact: If ) ( ) ( ) , ( y h x g y x f  and we are integrating over the rectangle ] , [ ] , [ d c b a R   then,                       dy y h dx x g dA y h x g dA y x f d c b a R R ) ( ) ( ) ( ) ( ) , ( Example 6 Evaluate  R dA y x 2 cos , ] 2 / , 0 [ ] 3 , 2 [     R . Solution                      2 / 0 2 3 2 2 cos cos  dy y dx x dA y x R
  • 67. Bas 111 Mathematics 3 Dr. Fatma Zedan El Emam 67 8 5 2 sin 2 1 2 1 2 5 ) 2 cos 1 ( 2 1 2 1 2 0 2 / 0 3 2 2                                         y y dy y x Example 7 Evaluate  R dA y xcos sin , where ] 2 / , 0 [ ] 2 / , 0 [     R Solution                     2 / 0 2 / 0 cos sin cos sin   dy y dx x dA y x R     1 1 1 sin cos 2 / 0 2 / 0        y x
  • 68. Bas 111 Mathematics 3 Dr. Fatma Zedan El Emam 68 Exercise 4 1-8 Evaluate the iterated integrals. 9-12 Evaluate the double integral over the rectangular region R.
  • 69. Bas 111 Mathematics 3 Dr. Fatma Zedan El Emam 69 13-15 Use a double integral to find the volume.
  • 70. Bas 111 Mathematics 3 Dr. Fatma Zedan El Emam 70 • Iterated Integrals With Nonconstant Limits Of Integration Example 8 Evaluate (a)    1 0 2 2 x x dx dy x y (b) dy dx y x y sin 3 / 0 cos 0    Solution 120 13 15 24 3 3 3 ) ( 1 0 5 8 1 0 4 7 1 0 3 1 0 2 1 0 2 2 2 2                                                    x x dx x x dx x y dx dy x y dx dy x y a x x y x x x x dy y x dy dx y x dy dx y x b y x y y cos 0 3 / 0 2 3 / 0 cos 0 3 / 0 cos 0 sin 2 sin sin ) (                       
  • 71. Bas 111 Mathematics 3 Dr. Fatma Zedan El Emam 71 48 7 cos 6 1 sin cos 2 1 3 / 0 3 3 / 0 2                    y dy y y 2.2 Double Integrals over General Regions D We will limit our study of double integrals to two basic types of regions, which we will call type I and type II; they are defined as follows. A plane region D is said to be of type I if it lies between the graphs of two continuous functions of x, that is,   ) ( ) ( , ) , ( 2 1 x g y x g b x a y x D     
  • 72. Bas 111 Mathematics 3 Dr. Fatma Zedan El Emam 72 Some type I regions Also, the plane region D is said to be of type II if it lies between the graphs of two continuous functions of y, that is,   ) ( ) ( , ) , ( 2 1 y h x y h d y c y x D      Some type II regions
  • 73. Bas 111 Mathematics 3 Dr. Fatma Zedan El Emam 73 Theorem 2 Fubini’s Theorem (Stronger form) Let ) , ( y x f is continuous on a regionD. 1. If D is defined by b x a   , ) ( ) ( 2 1 x g y x g   with 1 g and 2 g continuous on [a, b], (D is a type I region) then dx dy y x f dA y x f b a x g x g D     ) ( ) ( 2 1 ) , ( ) , ( . 2. If D is defined by , d y c   ) ( ) ( 2 1 y h x y h   with 1 h and 2 h continuous on [c, d], ], (D is a type II region) then dy dx y x f dA y x f d c y h y h D     ) ( ) ( 2 1 ) , ( ) , ( . Example 9 Evaluate   D dA y x ) 2 ( , whereD is the region bounded by the parabolas 2 2x y  and . 1 2 x y   Solution
  • 74. Bas 111 Mathematics 3 Dr. Fatma Zedan El Emam 74 The parabolas intersect when , 1 2 2 2 x x   that is, , 1 2  x so . 1   x We note that the region D, sketched in Fig.1, is a type I region but not a type II region and we can write   2 2 1 2 , 1 1 ) , ( x y x x y x D        Since the lower boundary is 2 2x y  and the upper boundary is 2 1 x y   , then   dx x x x x x x dx y xy dx dy y x dA y x x y x y x x D                        1 1 2 2 2 2 2 2 1 2 1 1 2 1 1 1 2 ] ) 2 ( ) 2 ( ) 1 ( ) 1 ( [ ) 2 ( ) 2 ( 2 2 2 2 dx x x x x         1 1 2 3 4 ) 1 2 3 ( 15 32 2 3 2 4 5 3 1 1 2 3 4 5               x x x x x .
  • 75. Bas 111 Mathematics 3 Dr. Fatma Zedan El Emam 75 Fig. 1 Example 10 Find the volume of the solid that lies under the paraboloid 2 2 y x z   and above the region D in the xy-plane bounded by the line x y 2  and the parabola 2 x y  . Solution 1 From Fig. 2 we see that D is a type I region and   x y x x y x D 2 , 2 0 ) , ( 2      Therefore the volume under 2 2 y x z   and above D is
  • 76. Bas 111 Mathematics 3 Dr. Fatma Zedan El Emam 76 35 216 6 7 5 21 3 14 3 3 ) ( 3 ) 2 ( ) 2 ( 3 ) ( ) ( 2 0 4 5 7 2 0 3 4 6 2 0 3 2 2 2 3 2 2 0 2 3 2 2 0 2 2 2 2 2 2 2                                                      x x x dx x x x dx x x x x x x dx y y x dx dy y x dA y x V x y x y x x D Fig. 2 D as a type I region
  • 77. Bas 111 Mathematics 3 Dr. Fatma Zedan El Emam 77 Solution 2 From Fig. 3 we see that D can also be written as a type II region and            y x y y y x D 2 1 , 4 0 ) , ( Therefore another expression for V is 35 216 96 13 7 2 15 2 2 24 3 3 ) ( ) ( 4 0 4 2 / 7 2 / 5 4 0 3 3 2 / 5 2 / 3 4 0 2 3 4 0 2 2 2 2 2 1 2 1                                        y y y dy y y y y dy x y x dy dx y x dA y x V y x y x y y D Fig. 3 D as a type II region
  • 78. Bas 111 Mathematics 3 Dr. Fatma Zedan El Emam 78 Example 11 Evaluate  R dA xy over the region R enclosed between , 2 1 x y  , x y  2  x and 4  x . Solution We view R as a type I region. dx xy dx dy xy dA xy x x y x x R              4 2 2 / 2 4 2 2 / 2 4 2 4 3 4 2 3 2 32 6 8 2                  x x dx x x 6 11 32 16 6 8 32 256 6 64                 
  • 79. Bas 111 Mathematics 3 Dr. Fatma Zedan El Emam 79 Example 12 Evaluate   R dA y x ) 2 ( 2 over the triangular region R enclosed between the lines , 1    x y 1   x y and 3  y Solution 1 We view R as a type II region     3 68 2 3 2 ) 2 2 ( ) 2 1 ( ) 2 2 1 ( ) 2 ( ) 2 ( 3 1 4 3 3 1 3 2 3 1 3 3 2 3 1 1 1 2 2 3 1 1 1 2 2                                     y y dy y y dy y y y y y dy x y x dy dx y x dA y x y x y x y y R
  • 80. Bas 111 Mathematics 3 Dr. Fatma Zedan El Emam 80 Solution 2 We view R as a type I region.                                                    2 0 3 0 2 3 2 0 3 1 3 0 2 3 1 3 2 0 3 1 2 0 2 3 1 2 2 2 2 )] 3 ) 1 ( ) 1 ( 2 ( ) 9 6 [( )] 3 ) 1 ( ) 1 ( 2 ( ) 9 6 [( ] 3 2 [ ] 3 2 [ ) 2 ( ) 2 ( ) 2 ( ) 2 ( ) 2 ( 2 1 dx x x x x dx x x x x dx y y x dx y y x dx dy y x dx dy y x dA y x dA y x dA y x y x y y x y x x R R R 3 68 6 52 6 84 )] ) 4 ( 3 ) 1 ( 3 2 9 3 [ ] ) 4 ( 3 ) 1 ( 3 2 9 3 [ 4 2 3 2 0 2 4 2 3 2                    x x x x x x x x x x
  • 81. Bas 111 Mathematics 3 Dr. Fatma Zedan El Emam 81 Example 13 Use a double integral to find the volume of the tetrahedron bounded by the coordinate planes and the plane . 2 4 4 y x z    Solution The tetrahedron is bounded above by the plane y x z 2 4 4    (1) and below by the triangular region R shown in Fig. 4. Thus, the volume is given by dA y x V R     ) 2 4 4 (
  • 82. Bas 111 Mathematics 3 Dr. Fatma Zedan El Emam 82 The region R is bounded by the x-axis, the y-axis, and the line y = 2 − 2x [set z = 0 in (1)], so that treating R as a type I region yields   3 4 ) 4 8 4 ( 4 4 ) 2 4 4 ( ) 2 4 4 ( 1 0 2 1 0 2 2 0 2 1 0 2 2 0                      dx x x dx y xy y dx dy y x dA y x V x y x R Fig. 4
  • 83. Bas 111 Mathematics 3 Dr. Fatma Zedan El Emam 83 • Reversing The Order Of Integration Sometimes the evaluation of an iterated integral can be simplified by reversing the order of integration. The next example illustrates how this is done. Example 14 Evaluate   2 0 1 2 / 2 y x dy dx e Solution [Since there is no elementary antiderivative of 2 x e , the integral cannot be evaluated by performing the x-integration first. Evaluate this integral by expressing it as an equivalent iterated integral with the order of integration reversed, so we must change the order of integration.] To reverse the order of integration, we treat R as a type I region, which enables us to write the given integral as
  • 84. Bas 111 Mathematics 3 Dr. Fatma Zedan El Emam 84     1 2 1 0 1 0 1 0 2 0 1 0 2 0 2 0 1 2 / 2 2 2 2 2 2                e e dx e x dx y e dx dy e dA e dy dx e x x x y x x x R x y x Example 15 Evaluate   1 0 1 2 . sin x dx dy y Solution Here, we must change the order of integration (why). So, we treat D as a type II region
  • 85. Bas 111 Mathematics 3 Dr. Fatma Zedan El Emam 85   ) 1 cos 1 ( 2 1 cos 2 1 sin sin sin sin sin 1 0 2 1 0 2 1 0 0 2 1 0 0 2 1 0 2 1 2                        y dy y y dy y x dy dx y dA y dx dy y y x x y D x D as a type I region D as a type II region • Area Calculated As A Double Integral Definition: The area of a closed, bounded plane region R is   R dA A . (2) To evaluate the integral in (2), we integrate the constant function f (x, y)=1 over R.
  • 86. Bas 111 Mathematics 3 Dr. Fatma Zedan El Emam 86 Example 16 Find the area of the region R bounded by x y  and 2 x y  in the first quadrant Solution We sketch the region (Fig. 5) and calculate the area as   . 6 1 3 2 ) ( 1 0 3 2 1 0 2 1 0 1 0 2 2                  x x dx x x dx y dx dy A x x x x Fig. 5 Example 17 Use a double integral to find the area of the region R enclosed between the parabola 2 2 1 x y  and the line y = 2x.
  • 87. Bas 111 Mathematics 3 Dr. Fatma Zedan El Emam 87 Solution The region R may be treated equally well as type I (Fig. 6a) or type II (Fig. 6b). Treating R as type I yields   . 3 16 6 2 1 2 of area 4 0 3 2 4 0 2 4 0 2 2 / 4 0 2 2 / 2 2                           x x dx x x dx y dx dy dA R x x y x x R Treating R as type II yields   . 3 16 4 3 2 2 2 1 2 of area 8 0 2 2 / 3 8 0 8 0 2 2 / 8 0 2 2 /                           y y dy y y dy x dy dx dA R y y x y y R
  • 88. Bas 111 Mathematics 3 Dr. Fatma Zedan El Emam 88 Exercises 5 1–8 Evaluate the iterated integral. 9-10 Evaluate the double integral in two ways using iterated integrals: (a) viewing R as a type I region, and (b) viewing R as a type II region. 11–12 Evaluate the double integral.
  • 89. Bas 111 Mathematics 3 Dr. Fatma Zedan El Emam 89 13–14 Use double integration to find the area of the plane region enclosed by the given curves. 15 Use double integration to find the volume of the solid. 16–17 Evaluate the integral by first reversing the order of integration.
  • 90. Bas 111 Mathematics 3 Dr. Fatma Zedan El Emam 90 2.3 Double Integrals in Polar Coordinates •Converting Double Integrals From Rectangular To Polar Coordinate Sometimes a double integral that is difficult to evaluate in rectangular coordinates can be evaluated more easily in polar coordinates by making the substitution x = r cos θ, y = r sin θ and expressing the region of integration in polar form; that is, we rewrite the double integral in rectangular coordinates as      d dr r r r f dA r r f dA y x f R R      limits e appropriat ) sin , cos ( ) sin , cos ( ) , (
  • 91. Bas 111 Mathematics 3 Dr. Fatma Zedan El Emam 91 Notice that d x d y is not replaced by dr dθ but by r dr dθ. Example 1 Use polar coordinates to evaluate      1 1 1 0 2 / 3 2 2 2 ) ( x dx dy y x . Solution 5 5 1 ) ( ) ( ) ( 0 0 1 0 3 2 / 3 2 2 1 1 1 0 2 / 3 2 2 2                    d d dr r r dA y x dx dy y x R x Fig. 1
  • 92. Bas 111 Mathematics 3 Dr. Fatma Zedan El Emam 92 Example 2 Use polar coordinates to evaluate , ) 4 3 ( 2   R dA y x where R is the region in the upper half-plane bounded by the circles 1 2 2   y x and . 4 2 2   y x Solution The region R can be described as   4 1 , 0 ) , ( 2 2      y x y y x R It is the half-ring shown in Figure 2(b), and in polar coordinates it is given by . 0 , 2 1       r Therefore,                              0 2 1 2 4 3 0 2 1 2 3 2 0 2 1 2 2 2 sin cos ) sin 4 cos 3 ( ) sin 4 cos 3 ( ) 4 3 ( d r r d dr r r d dr r r r dA y x r r R
  • 93. Bas 111 Mathematics 3 Dr. Fatma Zedan El Emam 93 2 15 ) 2 sin 4 15 2 15 sin 7 ) 2 cos 1 ( 2 15 cos 7 ) sin 15 cos 7 ( 0 0 0 2                                     d d      2 0 , 1 0 ) , ( ) (      r r R a           0 , 2 1 ) , ( ) ( r r R b Fig. 2
  • 94. Bas 111 Mathematics 3 Dr. Fatma Zedan El Emam 94 If ) , (  r f is the constant function whose value is 1, then the integral of f over R is the area of R. Area in Polar Coordinates The area of a closed and bounded region R in the polar coordinate plane is , .   R d dr r A  Example 3 Use a polar double integral to find the area enclosed by the three-petaled rose r = sin 3θ.
  • 95. Bas 111 Mathematics 3 Dr. Fatma Zedan El Emam 95 Solution We will calculate the area of the petal R in the first quadrant and multiply by 3.              4 1 6 6 sin 4 3 ) 6 cos 1 ( 4 3 3 sin 2 3 3 3 3 / 0 3 / 0 3 / 0 2 3 / 0 3 sin 0                    d d d dr r dA A R 2.4 Triple Integral As in the case of double integrals, a triple integral may be found by evaluating an appropriate iterated integral. Theorem 1 Let G be the rectangular box defined by the inequalities a ≤ x ≤ b, c ≤ y ≤ d, k ≤ z ≤ l If f is continuous on the region G, then
  • 96. Bas 111 Mathematics 3 Dr. Fatma Zedan El Emam 96 Moreover, the iterated integral on the right can be replaced with any of the five other iterated integrals that result by altering the order of integration. Example 4 Evaluate the triple integral  G dV z xy 3 2 12 over the rectangular box G defined by the inequalities −1 ≤ x ≤ 2, 0 ≤ y ≤ 3, 0 ≤z ≤ 2. Solution. Of the six possible iterated integrals we might use, we will choose the one in (1). Thus, we will first integrate with respect to z , holding x and y fixed, then with respect to y, holding x fixed, and finally with respect to x.
  • 97. Bas 111 Mathematics 3 Dr. Fatma Zedan El Emam 97 Example 5 Evaluate the triple integral where B is the rectangular box given by Solution. We could use any of the six possible orders of integration. If we choose to integrate with respect to x, then y, and then z, we obtain
  • 98. Bas 111 Mathematics 3 Dr. Fatma Zedan El Emam 98 • Evaluating Triple Integrals Over More General Regions Theorem 2 Let G be a simple xy-solid with upper surface ) , ( 2 y x g z  and lower surface ) , ( 1 y x g z  , and let R be the projection of G on the xy -plane. If f(x, y, z) is continuous on G, then
  • 99. Bas 111 Mathematics 3 Dr. Fatma Zedan El Emam 99 Example 6 Let G be the wedge in the first octant that is cut from the cylindrical solid 1 2 2   z y by the planes y = x and x = 0. Evaluate  G dV z Solution. The solid G and its projection R on the xy-plane are shown in Figure 4. The upper surface of the solid is formed by the cylinder and the lower surface by the xy-plane. Since the portion of the cylinder 1 2 2   z y that lies above the xy-plane has the equation , 1 2 y z   and the xy-plane has the equation z = 0, it follows from (2) that For the double integral over R, the x- and y- integrations can be performed in either order, since R is both a type I and type II region. We will integrate with respect to x first. With this choice, (3) yields
  • 100. Bas 111 Mathematics 3 Dr. Fatma Zedan El Emam 100 Fig. 4 Example 7 Evaluate ,  E dV z where E is the solid tetrahedron bounded by the four planes x = 0, y = 0, z = 0, and . 1    z y x
  • 101. Bas 111 Mathematics 3 Dr. Fatma Zedan El Emam 101 Solution. When we set up a triple integral it’s wise to draw two diagrams: one of the solid region E (see Figure 5) and one of its projection D on the xy-plane (see Figure 6). The lower boundary of the tetrahedron is the plane z = 0 and the upper boundary is the plane 1    z y x (or y x z   1 ). Notice that the planes 1    z y x and z= 0 intersect in the line 1   y x (or x y  1 ) in the xy-plane. So the projection of E is the triangular region shown in Figure 6, and
  • 102. Bas 111 Mathematics 3 Dr. Fatma Zedan El Emam 102 • Volume Calculated As A Triple Integral of volume   G dV G (4) Example 8 Use a triple integral to find the volume of the solid within the cylinder 9 2 2   y x and between the planes z = 1 and x + z = 5. Solution. The solid G and its projection R on the xy-plane are shown in Fig. 7. The lower surface of the solid is the plane z = 1 and the upper surface
  • 103. Bas 111 Mathematics 3 Dr. Fatma Zedan El Emam 103 is the plane x + z = 5 or, equivalently, z = 5 − x. Thus, from (2) and (4) For the double integral over R, we will integrate with respect to y first. Thus, (5) yields Where,
  • 104. Bas 111 Mathematics 3 Dr. Fatma Zedan El Emam 104 and the second integral is 0 because the integrand is an odd function. Fig. 7 Example 9 Find the volume of the solid enclosed between the paraboloids 2 2 5 5 y x z   and 2 2 7 6 y x z    . Solution. The solid G and its projection R on the xy-plane are shown in Figure 7. The projection R is obtained by solving the given equations simultaneously to determine where the paraboloids intersect. We obtain
  • 105. Bas 111 Mathematics 3 Dr. Fatma Zedan El Emam 105 or which tells us that the paraboloids intersect in a curve on the elliptic cylinder given by (6). The projection of this intersection on the xy-plane is an ellipse with this same equation. Therefore,
  • 106. Bas 111 Mathematics 3 Dr. Fatma Zedan El Emam 106 Fig. 7 • Integration In Other Orders In Formula (2) for integrating over a simple xy- solid, the  z integration was performed first. However, there are situations in which it is preferable to integrate in a different order. For example, Figure 8a shows a simple xz-solid, and Figure 8b shows a simple yz -solid. For a simple xz -solid it is usually best to integrate with respect to y first, and for a simple yz -solid it is usually best to integrate with respect to x first:
  • 107. Bas 111 Mathematics 3 Dr. Fatma Zedan El Emam 107 A simple xz-solid A simple yz -solid (a) Fig. 8 (b) Example 10 In Example 6 we evaluated  G dV z over the wedge in Fig. 4 by integrating first with respect to z. Evaluate this integral by integrating first with respect to x. Solution. The solid is bounded in the back by the plane x = 0 and in the front by the plane y x  , so
  • 108. Bas 111 Mathematics 3 Dr. Fatma Zedan El Emam 108 where R is the projection of G on the yz -plane (Fig. 9). The integration over R can be performed first with respect to z and then y or vice versa. Performing the z-integration first yields which agrees with the result in Example 6. Fig. 8
  • 109. Bas 111 Mathematics 3 Dr. Fatma Zedan El Emam 109 Exercises 6 1-4 Evaluate the iterated integral. 5-7 Evaluate the iterated integral by converting to polar coordinates. 8-11 Evaluate the iterated integral.
  • 110. Bas 111 Mathematics 3 Dr. Fatma Zedan El Emam 110 12-13 Evaluate the triple integral. 14-16 Use a triple integral to find the volume of the solid.
  • 111. Bas 111 Mathematics 3 Dr. Fatma Zedan El Emam 111 CHAPTER 3 First-Order Differential Equation 3.1 Introduction A differential equation (DE) is any equation which contains derivatives, either ordinary derivatives or partial derivatives. we shall classify differential equations by type, order, and linearity. Classification By Type: An equation involving only ordinary derivatives w.r.to a single independent variable is ordinary differential equation (ODE). For example, An equation involving partial derivatives w.r.to more than one independent variables is called a partial differential equation (PDE). For example,
  • 112. Bas 111 Mathematics 3 Dr. Fatma Zedan El Emam 112 Classification By Order: The order of a differential equation (either ODE or PDE) is the order of the highest derivative in the equation. For example, is a second-order ODE. Classification By Linearity: An nth-order ODE ). ( ) ( ) ( ) ( ) ( 0 1 1 1 1 x g y x a dx dy x a dx y d x a dx y d x a n n n n n n          is said to be linear since it satisfies the following two conditions 1) It is linear in ) ( , , , n y y y   . This means that, the dependent variable y and all its derivatives ) ( , , , n y y y   are of the first degree, that is, the power of each term involving y is 1. 2) The coefficients n a a a , , , 1 0  of ) ( , , , n y y y   depend at most on the independent variable x.
  • 113. Bas 111 Mathematics 3 Dr. Fatma Zedan El Emam 113 For example, the equations Are linear first, second, and third-order ODE s, where the first equation can be written of the form x y y x    4 . A nonlinear ODE is simply one that is not linear. Nonlinear functions of the dependent variable or its derivatives, such as sin y or , cannot appear in a linear equation. Therefore nonlinear term: coefficient depends on y x e y y y      2 ) 1 ( , nonlinear term: nonlinear function of y 0 sin 2 2    y dx y d and nonlinear term: power not 1 0 2 4 4    y dx y d
  • 114. Bas 111 Mathematics 3 Dr. Fatma Zedan El Emam 114 are examples of nonlinear first, second, and fourth-order ordinary differential equations, respectively. • The degree of a DE is the degree of the highest derivative after removing the radical sign and fraction. The solution of DE An equation containing dependent variable y and independent variable x and free from derivative, which satisfies the DE, is called the solution of the DE.
  • 115. Bas 111 Mathematics 3 Dr. Fatma Zedan El Emam 115 Note: The number of arbitrary constants in the solution is equal to the order of the differential equation. • Initial Condition(s) initial conditions are values of the solution and/or its derivative(s) at specific points of the form, 0 0 ) ( y x y  and/or k k y x y  ) ( 0 ) ( • Initial Value Problem An Initial Value Problem ( IVP) is a differential equation along with an appropriate number of initial conditions. For example 0 0 ) ( ), , ( y x y y x f dx dy   and 1 0 0 0 2 2 ) ( , ) ( ), , , ( y x y y x y y y x f dx y d      are first and second-order initial-value problems, respectively.
  • 116. Bas 111 Mathematics 3 Dr. Fatma Zedan El Emam 116 Note: The number of initial conditions required will depend on the order of the differential equation. 3.2 Solutions Of First ODEs 3.2.1 Separable Equations Definition 3.2.1 A first-order differential equation of the form dx x g dy y h ) ( ) (  is said to be separable or to have separable variables. The solution to the first-order separable differential equation dx x g dy y h ) ( ) (  is C dx x g dy y h     ) ( ) (
  • 117. Bas 111 Mathematics 3 Dr. Fatma Zedan El Emam 117 Example 2 Solve the differential equation 0 ) 4 ( 6 2 2      y x x xy . Solution. The equation may be written . 0 ) 4 ( ) 3 ( 2 2     dx dy x y x Assuming that , 0 ) 4 )( 3 ( 2    x y we may divide both sides by this product, then 0 3 1 4 2 2     dy y dx x x . Thus the given equation is separable, and integration of each term gives us . 3 4 ) 3 )( 4 ( ) 3 )( 4 ( ) 3 )( 4 ( ln 3 ln 4 ln 3 1 4 2 2 2 2 2 1 2 1 2 1 2 1                             x C y C y x C e y x C y x C y x C dy y dx x x C
  • 118. Bas 111 Mathematics 3 Dr. Fatma Zedan El Emam 118 Example 3 Solve the differential equation . 0 2 4   dx dy e y x Solution. Rewrite the equation as . 3 2 6 where 2 3 6 by sides both multiply 3 1 2 1 1 0 1 0 3 / 1 2 1 3 2 1 3 2 1 4 2 4 2 2 4                             C e y C C C y e C y e C dy y dx e dy y dx e dy dx e y x x x x x x Example 4 Solve . 4 2   y dx dy Solution. We put the equation in the form
  • 119. Bas 111 Mathematics 3 Dr. Fatma Zedan El Emam 119 . 4 2 dx y dy   By using partial fractions, we get dx dy y y               2 4 1 2 4 1 . 1 1 2 by replacing 2 2 2 2 2 2 4 2 2 ln 4 2 ln 2 ln 4 2 1 2 1 4 4 4 4 4 1 1 1 1 1 1 1 x x C x C x C C x Ce Ce y C e e e y y e e y y e y y C x y y C x y y C dx dy y y                                            
  • 120. Bas 111 Mathematics 3 Dr. Fatma Zedan El Emam 120 Example 5 Solve the IVP . 0 ) 0 ( , 2 sin cos ) ( 2    y x e dx dy x y e y y Solution. Dividing the equation by x ey cos gives dx x x dy e y e y y cos 2 sin 2                 xdx dy ye e xdx dy ye e dx x x x dy ye e y y y y y y sin 2 ) ( sin 2 ) ( cos cos sin 2 ) ( , cos 2 C x e ye e y y y         [ where y y y e ye dy ye        by integrating by parts]. The initial condition y = 0 when x = 0 implies C= 4. Thus a solution of the IVP is . cos 2 4 x e ye e y y y      
  • 121. Bas 111 Mathematics 3 Dr. Fatma Zedan El Emam 121 3.2.2 Exact Differential Equation Definition 3.2.2 A first-order differential equation of the form 0 ) , ( ) , (   dy y x N dx y x M is said to an exact differential equation provided . x N y M      The solution to the first-order exact differential equation 0 ) , ( ) , (   dy y x N dx y x M is C dy x N dx M y     ) containing not of terms ( const. as Example 6 Solve 0 ) 5 3 2 ( ) 2 3 5 ( 4 2 2 3 3 2 2 4       dy y y x y x dx xy y x x Solution. Here,
  • 122. Bas 111 Mathematics 3 Dr. Fatma Zedan El Emam 122 . 6 6 , 6 6 5 3 2 , 2 3 5 2 2 2 2 4 2 2 3 3 2 2 4 x N y M xy y x x N xy y x y M y y x y x N xy y x x M                     Then, the given equation is exact and the solution is C dy x N dx M y     ) containing not of terms ( const. as C y y x y x x C dy y dx xy y x x y             5 3 2 2 3 5 4 3 2 2 4 5 ) 2 3 5 ( const. as Example 7 Solve 0 ) 3 (sin ) 1 2 cos 2 ( 2 2 2       dy x x dx xy x xy Solution. Here,
  • 123. Bas 111 Mathematics 3 Dr. Fatma Zedan El Emam 123 . 2 cos 2 , 2 cos 2 3 sin , 1 2 cos 2 2 2 2 2 2 x N y M x x x x N x x x y M x x N xy x xy M                     So the given differential equation is exact differential equation and the solution is C dy x N dx M y     ) containing not of terms ( const. as C dy dx xy x xy y        3 ) 1 2 cos 2 ( 2 const. as . 3 sin 3 1 2 cos 2 2 2 2 C y x yx x y C dy y dx dx x y dx x x y               Example 8 Solve 0 1 ) 1 ( / /           dx dy y x e e y x y x Solution. We have,
  • 124. Bas 111 Mathematics 3 Dr. Fatma Zedan El Emam 124 . 1 1 , , 1 0 1 0 1 1 2 2 2 x N y M e y x e y x e y e y x N e y x y M y x e e N e M dy y x e e dx e dx dy y x e e y x y x y x y x y x y x y x y x y x y x y x y x y x                                                              Then the given equation is exact and its solution is C dy x N dx M y     ) containing not of terms ( const. as . 0 1 C ye x C dy dx e y x y y x                  const. as
  • 125. Bas 111 Mathematics 3 Dr. Fatma Zedan El Emam 125 Equations Reducible to Exact - Integrating Factor Sometimes a differential equation which is not exact may become so, on multiplication by a suitable function known as the integrating factor I.F. Rule 1. If N N M x y / ) (  is a function of x alone, say f (x), then   dx x f e ) ( I.F. Rule 2. If M M N y x / ) (  is a function of y alone, say ) (y f , then   dy y f e ) ( I.F. Example 9 Solve 0 2 ) ln 2 (    dx y dy xy x x (1) Solution. xy x x N y M dy xy x x dx y       ln 2 , 2 0 ) ln 2 ( 2
  • 126. Bas 111 Mathematics 3 Dr. Fatma Zedan El Emam 126 . ) ln 1 ( 2 , 2 y x x N y M         So, equation (1) not exact. Here, ). ( 1 ) ln 2 ( ) ln 2 ( ln 2 ln 2 2 2 x f x y x x y x xy x x y x N x N y M                  x x e e e e x x dx x dx x f 1 I.F. 1 ln ln 1 ) ( 1             On multiplying equation (1) by x 1 , we get the exact DE. . 2 1 ln 2 2 0 ) ln 2 ( 2 2 C y x y C dy y dx x y dy y x dx x y y             const. as
  • 127. Bas 111 Mathematics 3 Dr. Fatma Zedan El Emam 127 Example 10 Solve 0 ) 4 2 ( ) 2 ( 4 3 4      dy x y xy dx y y (2) Solution. Here, . 4 , 2 4 4 2 , 2 3 3 4 3 4              y x N y y M x y xy N y y M 3 3 ln ln 3 3 ) ( 3 3 4 3 3 1 I.F. ) ( 3 ) 2 ( ) 2 ( 3 2 ) 2 4 ( ) 4 ( 3 y y e e e e y f y y y y y y y y M y M x N x x dx y dx y f                              On multiplying equation (2) by 3 / 1 y , we get the exact DE. 0 ) 4 2 ( ) 2 ( 3 2      dy y x y x dx y y C y y y x C dy y dx y y y           2 2 2 ) 2 ( 2 ) 2 ( const. as
  • 128. Bas 111 Mathematics 3 Dr. Fatma Zedan El Emam 128 Exercises 7 1-4 prove that y is a solution of the indicated differential equation. Solve the differential equations Find the particular solution of the IVP. Solve the differential equations
  • 129. Bas 111 Mathematics 3 Dr. Fatma Zedan El Emam 129 Find the particular solution of the IVP. Solve the differential equations
  • 130. Bas 111 Mathematics 3 Dr. Fatma Zedan El Emam 130 3.2.3 Homogeneous Differential Equation Definition 3.2.3 A homogeneous DE is a first order DE that can be written in the form ) ( x y f dx dy  To solve the homogeneous DE, we put x y v  (or vx y  ) and dx dv x v dx dy   . The reduced differential equation involves v and x only and it can be solved by variables separable method. Another Definition A DE of the form ) , ( ) , ( y x y x f dx dy   is called a homogeneous equation if each term of f (x, y) and (x, y) is of the same degree.
  • 131. Bas 111 Mathematics 3 Dr. Fatma Zedan El Emam 131 Example 11 Solve the following differential equation xy y y x xy 2 3 ) 2 ( 2 2     Solution. We have, 2 2 2 2 2 2 3 2 3 ) 2 ( x xy xy y dx dy xy y dx dy x xy        . Divide numerator and denominator on the right by 2 x to obtain 1 2 2 ) ( 3 2    x y x y x y dx dy . Put x y v  vx y   and dx dv x v dx dy   . On substituting, the given equation becomes 1 2 2 2 3 1 2 2 3 2 2 2           v v v v v dx dv x v v v dx dv x v
  • 132. Bas 111 Mathematics 3 Dr. Fatma Zedan El Emam 132 x dx dv v v v v v v dx dv x               2 2 1 2 1 2 C x v v x dx dv v v v ln ln ) ( ln 1 2 2 2                 Cx v v    2 Replacing v with x y we get, 3 2 2 2 Cx xy y Cx x y x y      Example 12 Solve the differential equation . 0 cot cot          dx y x y dy y x x y Solution. We have, y x y y y x x dy dx y y x x dy dx y x y cot cot cot ) cot (      .
  • 133. Bas 111 Mathematics 3 Dr. Fatma Zedan El Emam 133 Divide numerator and denominator on the right by y to obtain y x y x y x dy dx cot 1 cot   . Put y x v  vy x   and dy dv y v dy dx   . On substituting, the given equation becomes v v v v v dy dv y v v v dy dv y v cot cot 1 cot cot 1 cot        y dy vdv v dy dv y       cot cot 1 C y v y dy dv v ln ln sin ln cot          C v y   sin Replacing v with y x we get, . sin C y x y 
  • 134. Bas 111 Mathematics 3 Dr. Fatma Zedan El Emam 134 Example 13 Solve the differential equation y x y dx dy x   2 Solution. Rewrite the equation as x y x y dx dy         2 Put x y v  vx y   and dx dv x v dx dy   . x dx v dv v dx dv x v v dx dv x v         2 2 2 C x v C x v x dx v dv             ln 1 ln 1 2 Replacing v with x y we get, C x x y    ln
  • 135. Bas 111 Mathematics 3 Dr. Fatma Zedan El Emam 135 3.2.4 Linear Differential Equations Definition 3.2.3 A first-order linear DE is an equation of the form ) ( ) ( x Q y x P dx dy   where P and Q are continuous functions. The solution to the first-order linear DE ) ( ) ( x Q y x P dx dy   is       c dx x Q x x y ) ( ) ( ) ( 1   where,   dx x P e x ) ( ) (  Note.   dx x P e x ) ( ) (  is called the integrating factor.
  • 136. Bas 111 Mathematics 3 Dr. Fatma Zedan El Emam 136 Example 14 Solve the differential equation 2 ) 1 ( ) 1 (     x e y dx dy x x Solution. Rewrite the equation as ) 1 ( 1 1     x e y x dx dy x . Here, 1 1 ) (    x x P and ) 1 ( ) (   x e x Q x . 1 1 ) ( 1 ) 1 ( ln ) 1 ( ln 1 1 ) ( x e e e e x x x dx x dx x P               Then the solution is       C e x C dx e x C dx x e x x C dx x Q x x y x x x                         ) 1 ( ) 1 ( ) 1 ( 1 1 ) 1 ( ) ( ) ( ) ( 1   Example 15 Solve the differential equation 0 3 5 5 2     x xy y x
  • 137. Bas 111 Mathematics 3 Dr. Fatma Zedan El Emam 137 Solution. Rewrite the equation as 3 3 5 x y x dx dy    . Here, x x P 5 ) (  and 3 3 ) ( x x Q   . ) ( 5 5 ln ln 5 5 ) ( x e e e e x x x dx x dx x P         Then the solution is                             C x x C dx x x C dx x x x C dx x Q x x y 3 1 3 1 ) 3 ( 1 ) ( ) ( ) ( 1 9 5 8 5 3 5 5   • Equations Reducible To The Linear Form (Bernoulli Equation) A Bernoulli differential equation has the form n y x Q y x P dx dy ) ( ) (   (1)
  • 138. Bas 111 Mathematics 3 Dr. Fatma Zedan El Emam 138 where n is a real number. The Bernoulli DE can be reduced to the linear form on dividing by n y and substituting z yn  1 1 . On dividing both sides of (1) by n y , we get ) ( 1 ) ( 1 1 x Q y x P dx dy y n n    (2) Put z yn  1 1 , so that dx dz n dx dy y dx dz dx dy y n n n ) 1 ( 1 1 ) 1 (      . Then (2) becomes ) ( ) ( ) 1 ( 1 x Q z x P dx dz n    or ) ( ) 1 ( ) ( ) 1 ( x Q n z x P n dx dz     which is a linear equation in z and can be solved easily by the previous method. Note; Equation (1) is linear if n =0 and separable if n =1.
  • 139. Bas 111 Mathematics 3 Dr. Fatma Zedan El Emam 139 Example 16 Solve the differential equation 0 ) ( 2    dx y x y dy x Solution. Rewrite the equation as 2 2 1 1 y x y x dx dy    (Bernoulli DE, n =2) 2 2 1 1 1 1 2 x y x dx dy y y      Put z y  1 , so that dx dz dx dy y dx dz dx dy y      2 2 1 1 The given equation reduces to a linear differential equation in z. 2 1 1 x z x dx dz   . Here, x x P 1 ) (   and 2 1 ) ( x x Q  . 1 ) ( 1 ln ln 1 ) ( x e e e e x x x dx x dx x P          
  • 140. Bas 111 Mathematics 3 Dr. Fatma Zedan El Emam 140 Then the solution is     ) 2 1 ( 1 2 1 1 ) ( ) ( ) ( 1 2 2 3 2 C x x y C x x C dx x x C dx x x x C dx x Q x x z                                  Example 17 Solve the differential equation 3 6 1 2 y x y x y     Solution. Rewrite the equation as 3 6 2 y x y x dx dy   (Bernoulli DE, n =3) 6 2 3 1 2 1 3 x y x dx dy y y     Put z y  2 1 , so that
  • 141. Bas 111 Mathematics 3 Dr. Fatma Zedan El Emam 141 dx dz dx dy y dx dz dx dy y 2 1 1 1 2 2 3      The given equation reduces to a linear differential equation in z. 6 ) 2 ( 6 2 4 2 2 1 x z x dx dz x z x dx dz          . Here, x x P 4 ) (   and 6 2 ) ( x x Q   . 1 ) ( 4 4 ln ln 4 1 4 ) ( x e e e e x x x dx x dx x P            Then the solution is                                 C x x C dx x x C dx x x x C dx x Q x x z 3 2 2 ) 2 ( 1 ) ( ) ( ) ( 1 3 4 2 4 6 4 4   ) 3 2 ( 1 3 4 2 C x x y    
  • 142. Bas 111 Mathematics 3 Dr. Fatma Zedan El Emam 142 or 1 ) 3 2 ( 2 4 7    y Cx x Example 18 Solve the following IVP 0     y x y y , y(1) = 0 Solution. Rewrite the equation as 2 1 1 y y x dx dy   (Bernoulli DE, 2 1  n ) 1 1 2 1 2 1 2 1      y x dx dy y y . Put z y  2 1 , so that dx dz dx dy y dx dz dx dy y 2 2 1 2 1 2 1      The given equation reduces to a linear differential equation in z. 2 1 2 1 1 1 2 2       z x dx dz z x dx dz . Here, x x P 2 1 ) (  and 2 1 ) (  x Q
  • 143. Bas 111 Mathematics 3 Dr. Fatma Zedan El Emam 143 . ) ( 2 1 2 1 ln 1 2 1 ) ( x e e e x x dx x dx x P        Then the solution is   2 1 2 1 2 3 2 1 2 1 2 1 3 1 3 1 3 2 1 ) ( ) ( ) ( 1 2 1                                Cx x y Cx x C x x C dx x x C dx x Q x x z   Applying the initial condition and solving for C gives, 3 1 3 1 0      C C . Thus a solution of the IVP is ) ( 3 1 2 1 2 1    x x y or 2 ) 1 ( 9 1   x x x y
  • 144. Bas 111 Mathematics 3 Dr. Fatma Zedan El Emam 144 Exercises 8 1-5 Solve the following differential equations 6-7 Solve the following differential equations by using the Substitution x = vy. 8 Show that the following differential equation is both homogeneous and exact and solve it. 9-12 Solve the following differential equations
  • 145. Bas 111 Mathematics 3 Dr. Fatma Zedan El Emam 145 13 The value of  so that 2 y e is an integrating factor of the differential equation In Problems 15–20 solve the given differential equation by using an appropriate substitution. In Problems 21 and 22 solve the given initial-value problem.
  • 146. Bas 111 Mathematics 3 Dr. Fatma Zedan El Emam 146 CHAPTER 4 Second-Order Linear Differential Equation 4.1. Basic concepts Definition 4.1.1 The second-order linear differential equation with constant coefficients has the form ) ( 2 2 x f cy dx dy b dx y d a    . (1) If 0 ) (  x f for all x, the equation is said to be homogeneous. Notice that the meaning of the word homogeneous is different from that in Section 3.2.3. If 0 ) (  x f for some x, the equation is said to be nonhomogeneous. Thus the form of a second-order linear homogeneous differential equation is 0       cy y b y a (2)
  • 147. Bas 111 Mathematics 3 Dr. Fatma Zedan El Emam 147 Theorem 4.3.1. If 1 y and 2 y are both solutions of equation (2), then the linear combination 2 2 1 1 y c y c y   where 1 c and 2 c are arbitrary constants, is also a solution. • Linear Dependence And Linear Independence Of Solutions There is a simple test to determine whether two solutions of equation (2) are linearly independent or dependent on an open interval I . Define the Wronskian ) , ( 2 1 y y W of two solutions 1 y and 2 y to be the 2×2 determinant . ) , ( 1 2 2 1 2 2 1 1 2 1 y y y y y y y y y y W       
  • 148. Bas 111 Mathematics 3 Dr. Fatma Zedan El Emam 148 Often we denote this Wronskian as just W(x). The solutions 1 y and 2 y are linearly independent on I if and only if 0 ) (  x W on I. Example 1 Determine whether the functions x e2 and x xe2 are linearly independent Solution. 0 2 2 2 2 ) ( 4 4 4 4 2 2 2 2 2        x x x x x x x x x e xe xe e xe e xe e e x W , so x e2 and x xe2 are linearly independent . Theorem 4.3.2. If 1 y and 2 y are linearly independent solutions of equation (2) on an interval, and a is never 0, then the general solution is given by 2 2 1 1 y c y c y   where 1 c and 2 c are arbitrary constants.
  • 149. Bas 111 Mathematics 3 Dr. Fatma Zedan El Emam 149 • The general solution of Eq. (1) has the form p c y y y   where the complementary function ) (x yc is general solution of associated homogeneous Eq. (2) and ) (x yp is a particular solution of Eq. (1) i.e. p y y c y c y    2 2 1 1 • Differential Operator Symbol D stands for the operation of differential i.e., 2 2 2 , dx y d y D dx dy Dy   . For example x x x x x x e e dx d e D e e dx d e D 3 3 2 2 3 2 3 3 3 9 , 3     ,
  • 150. Bas 111 Mathematics 3 Dr. Fatma Zedan El Emam 150 D 1 stands for the operation of integration and 2 1 D stands for the operation of integration twice. Some properties of operator D is ) ( ) ( )] ( ) ( [ ) 1 2 1 2 1 x f D x f D x f x f D    ) ( )] ( [ ) 2 x Df k x f k D  4.2 Method For Finding The Complementary Function (C.F.) c y Steps for solving 1) Starting from: 0       cy y b y a 2) Rewrite this Eq. as: 0 ) ( 2    y c bD aD 3) Write the auxiliary equation A.E. (characteristic equation): 0 2    c b a   4) Solve the auxiliary equation:
  • 151. Bas 111 Mathematics 3 Dr. Fatma Zedan El Emam 151 a ac b b 2 4 , 2 2 1       Then form the solution according to the following possibilities: Solutions to the A.E. general solution Two real distinct roots 2 1,  Real repeated roots      2 1 Complex roots   i  x x e C e C y 2 1 2 1     x x x e x C C xe C e C y    ) ( 2 1 2 1       x C x C e y x    sin cos 2 1   Finally, solve for constants if you have been given initial conditions. Example 2 Solve
  • 152. Bas 111 Mathematics 3 Dr. Fatma Zedan El Emam 152 . 0 15 8 2 2    y dx dy dx y d Solution. Given equation can be written as 0 ) 15 8 ( 2    y D D Here auxiliary equation is 5 , 3 0 ) 5 )( 3 ( 0 15 8 2 1 2                 Hence, the required solution is x x e C e C y 5 2 3 1   Example 3 Solve . 0 3 2 2    y dx dy dx y d Solution. Given equation can be written as 0 ) 1 3 ( 2    y D D Here auxiliary equation is 0 1 3 2      6 13 1 , 2 1       The general solution is
  • 153. Bas 111 Mathematics 3 Dr. Fatma Zedan El Emam 153 x x e C e C y 6 ) 13 1 ( 2 6 ) 13 1 ( 1       Example 4 Solve . 0 9 6 2 2    y dx dy dx y d Solution. Given equation can be written as 0 ) 9 6 ( 2    y D D A.E. is 0 9 6 2      3 0 ) 3 ( 2 1 2          Then, the general solution is x x xe C e C y 3 2 3 1   Example 5 Solve the equation . 0 9 12 4       y y y Solution. Given equation can be written as 0 ) 9 12 4 ( 2    y D D A.E. is 0 9 12 4 2      0 ) 3 2 ( 2    
  • 154. Bas 111 Mathematics 3 Dr. Fatma Zedan El Emam 154 2 3 2 1       Then, the general solution is x x xe C e C y 2 3 2 2 3 1     Example 6 Solve . 0 5 4 2 2    y dx dy dx y d Solution. Here the auxiliary equation is 0 5 4 2      i i            2 2 2 4 2 20 16 4 , 2 1   Then, the general solution is ) sin cos ( 2 1 2 x C x C e y x    Example 7 Solve the initial-value problem 0     y y 2 ) 0 (  y 3 ) 0 (   y
  • 155. Bas 111 Mathematics 3 Dr. Fatma Zedan El Emam 155 Solution. The auxiliary equation is    0 1 2  1 2    i    2 1,  . Thus , 1 , 0     and since , 1 0  x e the general solution is x C x C x y sin cos ) ( 2 1   x C x C x y cos sin ) ( 2 1     The initial conditions become 3 ) 0 ( , 2 ) 0 ( 2 1      C y C y Therefore the solution of the initial-value problem is x x x y sin 3 cos 2 ) (  
  • 156. Bas 111 Mathematics 3 Dr. Fatma Zedan El Emam 156 Exercises 9 1-4 Determine whether the given set of functions are linearly independent on the interval ) , (   . 1. x e x f  ) ( 1 and x e x f   ) ( 2 2. x x f 3 sin ) ( 1  and x x f 3 cos ) ( 2  3. x e x f   ) ( 1 and x e x f   5 ) ( 2 4. 3 1 ) ( x x f  and 4 2 ) ( x x f  5-12 Find the general solution of the given second- order differential equation.
  • 157. Bas 111 Mathematics 3 Dr. Fatma Zedan El Emam 157 4.3. Methods for Finding The Particular Solution p y 4.3.1 Inverse Differential Operator Method BY using the differential operator D, we can write the equation ) ( 2 2 x f cy dx dy b dx y d a    in the form ) ( ) ( 2 x f y c bD aD    or more simply, ) ( ) ( x f y D   where c bD aD D     2 ) ( . So, ) ( ) ( 1 x f D yp   . • Rules To Find p y 1) If m x f  ) ( (m is constant) a) 0 ) 0 ( , ) 0 ( 1 ) ( 1        c m m m D yp
  • 158. Bas 111 Mathematics 3 Dr. Fatma Zedan El Emam 158 0 ) 0 ( , 1 ) ( 1 1 ) ( 1 b) 2           x b m b m D m b aD D m bD aD m D yp Example 8 Find the particular solution of the following DE 2 8 2       y y y Solution. Given equation can be written as 2 ) 8 2 ( 2    y D D     . 4 1 8 2 2 8 0 0 1 2 8 2 1 2          D D yp Example 9 Find the particular solution of the following D.E. 2 2      y y Solution. Given equation can be written as 2 ) 2 ( 2   y D D
  • 159. Bas 111 Mathematics 3 Dr. Fatma Zedan El Emam 159       . ) 1 ( 1 2 2 0 1 1 2 2 1 1 2 2 1 2               x dx D D D D D D yp 2) If ) (x f is a polynomial of degree n and ) (D  take any form of , , ) 1 ( ), 1 ( ), 1 ( 2  D D D    then ) ( ] ) 1 ( 1 [ ) ( ] 1 [ ) ( 1 1 2 1 x f D D D x f D x f D n n            ) ( ] 1 [ ) ( ] 1 [ ) ( 1 1 2 1 x f D D D x f D x f D n           ) ( ] ) 1 ( 3 2 1 [ ) ( ] 1 [ ) ( ) 1 ( 1 2 2 2 x f D n D D x f D x f D n            Here we expand 1 )] ( [   D by the Binomial theorem in ascending powers of D as far as the result of operation on n x is zero.
  • 160. Bas 111 Mathematics 3 Dr. Fatma Zedan El Emam 160 Example 10 Solve the following DE 2 x y y      Solution. Given equation can be written as 2 2 ) ( x y D D   A.E. is 0 ) 1 ( 0 2          , 1 , 0 , 2 1 2 1 x c e C C y          . 2 3 ) 2 2 ( 1 ) 1 ( 1 ) 1 ( 1 ) 1 ( 1 1 1 2 3 2 2 2 2 1 2 2 2 x x x x x D x D D D x D D x D D x D D yp                 Then the general solution is x x x e C C y y y x p c 2 3 2 3 2 1         Example 11 Find the particular solution of the following DE 2 3 x y y y      
  • 161. Bas 111 Mathematics 3 Dr. Fatma Zedan El Emam 161 Solution. Given equation can be written as 2 2 ) 1 3 ( x y D D    2 1 2 2 2 )] 3 ( 1 [ 1 3 1 x D D x D D yp         2 2 2 2 2 ] 8 3 1 [ ] 9 ) 3 ( 1 [ x D D x D D D        16 6 2    x x 3) Exponential shift: If ) ( ) ( x v e x f mx  , then ) ( ) ( 1 ) ( ) ( 1 x v m D e x v e D mx mx     Example 12 Solve the DE x e x y y y 2 3 4 4       Solution. Given equation can be written as x e x y D D 2 3 2 ) 4 4 (    A.E. is 0 ) 2 ( 0 4 4 2 2          , 2 2 2 2 1 2 1 x x c xe C e C y        
  • 162. Bas 111 Mathematics 3 Dr. Fatma Zedan El Emam 162     x x p e x D e x D D y 2 3 2 2 3 2 ) 2 ( 1 4 4 1      3 2 2 3 2 2 1 ] 2 ) 2 [( 1 x D e x D e x x     20 4 1 5 2 4 2 x e x D e x x   Then the general solution is 5 2 2 2 2 1 20 1 x e xe C e C y y y x x x p c      Example 13 Solve the DE 3 3 9 6 x e y y y x        Solution. Given equation can be written as 3 3 2 ) 9 6 ( x e y D D x     A.E. is 0 ) 3 ( 0 9 6 2 2          , 3 3 2 3 1 2 1 x x c xe C e C y           
  • 163. Bas 111 Mathematics 3 Dr. Fatma Zedan El Emam 163                    3 3 2 3 3 2 ) 3 ( 1 9 6 1 x e D x e D D y x x p   3 2 3 3 2 3 1 1 ] 3 ) 3 [( 1        x D e x D e x x x e x e x D e x x x 2 ) 1 )( 2 ( 2 1 3 1 3 2 3                  Then the general solution is x e xe C e C y y y x x x p c 2 3 3 2 3 1         Example 14 Solve the IVP. 4 1 ) 0 ( , 4 11 ) 0 ( , 6 8 3 4            y y xe y y y x Solution. Given equation can be written as 6 8 ) 3 4 ( 2     x xe y D D A.E. is 0 ) 3 )( 1 ( 0 3 4 2            , 3 , 1 , 3 2 1 2 1 x x c e C e C y              6 8 3 4 1 2     x p xe D D y
  • 164. Bas 111 Mathematics 3 Dr. Fatma Zedan El Emam 164     2 1 2 2 6 3 4 1 8 3 4 1 I I D D xe D D x                 x D D e x D D e x D D e xe D D I x x x x ) 4 1 )( 2 1 )( 4 )( 2 ( 1 8 ) 4 )( 2 ( 1 8 ] 3 ) 1 ][( 1 ) 1 [( 1 8 8 ) 3 )( 1 ( 1 1                   x D D e x D D e x x                                 4 1 2 1 4 1 2 1 1 1 ) 2 1 4 1 ( 4 1 2 1                   x e x D e x x ) 3 4 ( 4 1 ) 4 3 (     x e x e x x ,     2 6 3 0 0 1 6 3 4 1 2 2        D D I .
  • 165. Bas 111 Mathematics 3 Dr. Fatma Zedan El Emam 165 Then 2 ) 3 4 ( 4 1 2 1      x e I I y x p and the general solution is 2 ) 3 4 ( 4 1 3 2 1          x e e C e C y y y x x x p c , ) 3 4 ( 4 1 ) 4 ( 4 1 3 3 2 1          x e e e C e C y x x x x ) 1 4 ( 4 1 3 3 2 1        x e e C e C x x x , 2 ) 3 ( 4 1 4 11 4 11 ) 0 ( 2 1          C C y  0 2 1    C C (1) 4 1 3 4 1 4 1 ) 0 ( 2 1        C C y  0 3 2 1    C C (2) From (1) and (2) we get 0 , 0 2 1   C C , then the required solution is 2 ) 3 4 ( 4 1    x e y x
  • 166. Bas 111 Mathematics 3 Dr. Fatma Zedan El Emam 166 4) If mx e x f  ) ( , then a) 0 ) ( , ) ( 1 ) ( 1       m e m e D y mx mx p b) 0 ) ( , ) ( 1 ) ( 1        m e m x e D y mx mx p and 0 ) (   m . If    0 ) (m mx mx p e m x e D y ) ( 1 . ) ( 1 2       Example 15 Solve the DE x e y y y 4 42 3 2       Solution. Given equation can be written as x e y D D 4 2 42 ) 3 2 (    A.E. is 0 ) 3 )( 1 ( 0 3 2 2            , 3 , 1 , 3 2 1 2 1 x x c e C e C y            x p e D D y 4 2 42 3 2 1   
  • 167. Bas 111 Mathematics 3 Dr. Fatma Zedan El Emam 167 x x x x e e e e D D 4 4 4 4 2 ) 7 )( 3 ( 1 42 ) 3 4 )( 1 4 ( 1 42 ) 3 )( 1 ( 1 42         Then the general solution is x x x p c e e C e C y y y 4 3 2 1 2       Example 16 Find the particular solution of the following DE x e y y       Solution. x p x e D D y e y D D        2 2 1 ) ( We note that 0 ) ( 1 2     D D D , then x x x D x D x p xe e x e D x e D D x e D D y                      ) 1 2 ( 1 ) 1 2 ( 1 ) ( 1 1 1 1 2 2